MICROBIOLOGY

Pataasin ang iyong marka sa homework at exams ngayon gamit ang Quizwiz!

Routes and Mode of infection

-Direct contact with bodily fluids and/or an infected source. -Vertical transmission (transplacental—mother to child). -Direct inoculation through injections or trauma; organ transplant. -Vectors, such as insects and small rodents. Humans can become infected through bites of vector species or inhalation of viral particles from the vector's hair, fur, or feces. --Arthropod-borne viruses—arboviruses: Mainly belong to four families of viruses—Flaviviridae, Togaviridae, Bunyaviridae, and Reoviridae. *The most common arthropod vectors are mosquitoes and ticks. --Rodent-borne viruses—roboviruses: Belong to the Arenavirdiae and Bunyaviridae families. These viruses are generally transmitted by rats and mice.

Why is the incidence of fungal infections on the rise?

-Immunosuppression -Diseases - HIV, cancer, diabetes mellitus -Anticancer chemotherapeutic agents -Radiotherapy -Organ transplant recipients on immunosuppressants (therefore more susceptible to infections with fungi) -Long term antibiotic use -Indwelling lines - central lines, total parenteral nutrition

Capsule/Slime Layer

-Important virulence factor; encapsulated bacteria resist phagocytosis -Antigenic—induces Ab production in the host -Capsule used in the preparation of vaccines *A vaccine that contains capsular material is going to elicit the production of antibodies in a host cell

Acid-fast staining *MYCOBACTERIUM is a prime example of acid fast bacterium -Acid Fast = PINK -Non acid fast = blue -Carbofuscin stain used here

-Multi-layer of peptidoglycan (as in G+ve bacteria) + mycolic acids + wax D + cord factor (trehalose) + other waxes -Waxy mycolic acids = 50% of the cell wall -Resistant to many dyes, chemicals, acids, antibiotics -Acid-fast bacteria stain pink, non acid-fast bacteria stain blue

Identification features of Pseudomonas -S. viridans produces green colonies when it grows on blood or chocolate agar -Pseudomonas produces green colonies on nutrient agar Oxidase test: -You take a filter paper and add an oxidase reagent. Pick up the colony and touch it to the filter paper. -If it turns deep purple: Oxidase positive

-Nutrient agar showing green pigment colonies -Oxidase positive -Mac Conkey's agar showing NLF colonies -Antibiotic sensitivity test showing resistance to a number of drugs

Uncoating

-Once internalized, the capsid must be delivered to the site of replication and the capsid and/or envelope removed -Occurs concomitantly or shortly after penetration May be initiated by attachment to the receptor or promoted by the acidic environment or proteases in the endosome/lysosome. -May be partial as in reoviridae and poxviridae -Why should we learn about virus uncoating? *Viral uncoating proceses are a target for antiviral drugs - Amantadine, Rimantidine

Exotoxin A: An important virulence factor of Pseudomonas

-One of the important virulence factors is the EXOTOXIN A (resembles C. diphtheriae toxin in MOA) -It acts by inhibiting the Elongation Factor 2 and in turn inhibits protein synthesis.

Parasitism, mutualism, symbiosis

-Parasitism - an association between two organisms wherein one is benefitted to the detriment of the other -Mutualism - an association between two organisms wherein both benefit from the relationship -Symbiosis - an extreme form of mutualism wherein the association between the two organisms is compulsory

Antigenic mimicry

-Pathogen has surface molecules that resemble host cell antigens -Group A streptococci and cardiac muscle myosin (rheumatic fever) -Group A streptococci and hyaluronic acid capsule (hyaluronic acid is also a component of connective tissue) -Treponema pallidum and cardiolipin -Mycoplasma pneumoniae and erythrocytes

Support of viral replication by host cells

-Permissive cells: allow productive infection because has all the biochemical machinery needed for a virus to enter and replicate -Non permissive cells: may lack a receptor, important enzyme pathway, or transcriptional activator or express an antiviral mechanism that will not allow replication of a particular type or strain of virus -Semipermissive cells: allow some viral functions to occur or support low levels of replication

Generalised Transduction

-Phage infects bacterial cell and destroys host DNA -Phage undergo genome replication and protein biosynthesis -Some bacterial DNA erroneously packaged into phage heads -Cells lysed, progeny phage released -Progeny phage infect another bacterial cell -If contains bacterial DNA, phage cannot replicate -DNA may integrate into host genome

Specialised Transduction

-Phage infects bacterial cell and integrates into host chromosome -Imprecise exit of phage from host chromosome -Phage replication -> some progeny contain bacterial DNA -Progeny phage infect other bacteria -DNA from initial host may integrate into chromosome of new host

Phenotypic Mixing and Masking

-Phenotypic mixing is known to occur among members of the Picornaviridae family -Can potentially occur between DNA and RNA viruses if both are enveloped.

Blood and tissue protozoans

-Plasmodium: P. falciparum, P. vivax, P. ovale, P. malariae -Babesia: B. microti -Toxoplasma: Toxoplasma gondii -Leishmania: L. donovani, L. tropica, L. major -Trypanosoma: T. cruzi, T. brucei rhodesiense, T. brucei gambiense

Candida albicans Aspergillus

-Poor oral hygiene or Dental manipulations -> Bacteremia due to organisms in the oral cavity -Rarely HACEK group -Commonly Streptococcus viridans group

Mother to baby

-Prepartum -Intra partum -Post partum -Rubella virus, CMV, Varicella-zoster virus, Parvovirus B19, HIV, LCM, ECHO virus

Invasion - other mechanisms

-Promote self uptake into the cell by using a TYPE III SECRETION DEVICE. Shigella, Salmonella, and Yersinia. -POLYMERISATON OF INTRACELLULAR ACTIN to push Shigella into adjacent cells. -Secrete proteins that promote attachment - Enteropathogenic Escherichia coli -Secretes proteins that promote attachment and uptake Salmonella -Listeria monocytogenes causes the POLYMERISATION OF ACTIN at its rear to propel the bacteria around the cell and into an adjacent cell

Subcutaneous fungus (cont)

-Sporotrichosis caused by Sporothrix schenckii -Eumycetoma caused by the phaeoid fungi -Uleronodular lesions of sporotrichosis. Note the distribution of the lesions. What do you think?

Prions

-They are protein in nature and do not have any nucleic acid, hence the name prion (proteinaceous infectious particle). -They are experimentally transmissible to a variety of mammals. -They are the causative agent of transmissible spongiform encephalopathies or prion diseases - a unique group of fatal neurodegenerative disorders occurring in humans and animals. -No evidence of an immune reaction has been found in these diseases. -Neuronal death and a reactive proliferation of microglia occur, and an abnormal form of prion protein accumulates in the CNS as diffuse deposits and in the form of amyloid plaques.

Strongyloides

-This is a helminth -Sexes are separate. Male and female worms, both are cylindrical. They infect by penetrating the skin.

Candida -Most of the candida infections are endogenous because we have candida naturally on our skin and mucous -Candida can flourish in hospital environments, also change in hormones and taking antibiotics predisposes you for candidal infections

-Candida is normal flora in the oral cavity, female genital tract and GIT -Overgrowth of Candida leads to a variety of infections; mostly from endogenous source -One of the most common fungi encountered in infections seen in the community as well as in the hospital set up. -Classified as a yeast like fungus

Underlying conditions and possible etiology (Cont)

-Cardiac surgery and valve replacement Common causes: -S. aureus (mostly) -Staph epidermidis (mostly) -Pseudomonas -Gram negative enteric bacilli -Strep viridans -Enterococci -Fungi: Candida.

Dimorphism

-Dimorphic fungi undergo morphological transformation to a more invasive form (which is the yeast form). -All of the primary systemic fungal pathogens are agents of respiratory infections, and none are obligate parasites. -They exist in two distinct forms --a saprobic phase *filamentous septate hyphae *found in soil or decaying vegetation *produces conidia --A parasitic phase adapted to grow at 37°C and to reproduce asexually in the host respiratory mucosa -This dimorphism allows these fungi to cope with the hostile environmental conditions of the host *Dimorphism is a virilence factor itself

Adaptive

ACTIVE Natural -Host gets infected -> immune system overcomes infection -> resolution Artificial -vaccination PASSIVE Natural -Transplacental IgG transfer IgA and IgG in breast milk Artificial -Give antibodies raised in another host

Source of infection

Exogenous (environmental) -Inhalation of conidia from molds -Traumatic inoculation of spores Endogenous (members of normal flora!) -Few yeasts like Candida albicans

Bacterial replication

DNA replication regulated by DNA-DEPENDENT DNA POLYMERASE -Starts at ORI and ends at TER Topoisomerases alter supercoiling of DNA strands -Unique to prokaryotes -Antibiotic target (fluoroquinolones - are made to target specifically topoisomerases that are only found in bacteria and not in eukaryotes)

Treatment

- Ceftazidime/ Cefepime/ Imipenem + Amikacin/ Tobramycin Several strains may be multidrug resistant due to production of extended-spectrum β-lactamases; metallo-betalactamases; carbapenemases and aminoglycoside modifying enzyme.

Intracellular bacteria

-Bacteria that grow intracellularly include mycobacteria, francisellae, brucellae, chlamydiae, and rickettsiae -They are inaccessible to neutralizing antibody (humoral immunity) -Cell mediated immunity via cytotoxic T-cells is important in resolving these infections

Release

-Cell lysis -Exocytosis -Budding *most enveloped viruses acquire an envelope from the host cell *Virus is released without killing the cell, can cause chronic infections -Can we target the release process with drugs? *Oseltamivir - a neuraminidase inhibitor used to treat influenza patients

stages of infection

-Entry into the human body -Adhesion, Colonisation, and Invasion -Pathogenic actions of bacteria -Mechanisms for escaping host defences

Axial filament

-Flagella-like structures wrapped around the body of the bacteria; endogenous flagella -Seen in spirochetes Clinical significance: -Corkscrew rotational movement to bore through tissue

Albert's stain

-For visualizing volutin granules (metachromatic), a key feature of Corynebacterium diphtheriae *Granules stain purple-black while the cytoplasm stains green

Leishmania

-Leishmania spp. amastigotes in a Giemsa-stained tissue scraping -Here you see organisms inside of the macrophage

Sporozoa and Microsporidia

-Microsporidia -Cryptosporidium -Cyclospora -Isospora

Microfilaria

-Parasite seen outside of the RBCs and is found in the blood and the tissues. Worm-like

A patient presents to the OPD (outpatient department) for an annual health check-up. The nurse on duty greets the patient and washes her hands using a liquid from a bottle shown above. 1. What is the most probable active ingredient in this liquid? 2. What is the mechanism of action of the contents?

1. Alcohol -70% alcohol, 30% water -Either ethanol or isopropyl 2. Disrupts membranes, denatures proteins

1. Describe 3 strategies by which Neisseria might exploit their pili for evasion of the host immune system? 2. Explain how capsules aid in evasion of the host immune system 3. Explain what is meant by the term "fastidious" 4. Explain what is meant by quorum sensing

1. Antigenic Variation, Phasic Variation, Soluble Pilli (like Neisseria) 2. Capsules resist phagocytosis 3. Needs specialized environment to grow (like co-factors or NAD in addition to the normal nutrients) 4. Bacteria in a biofilm communicate with each other, they then produce small molecules and the bacteria begin to sense this once the small molecules concentration is increased. This will then cause genes to be activated that will help with virilence

Bacterial Anatomy

1. Bacterial cell wall/cell envelope 2. Structures external to the cell wall (appendages) 3. Structures enclosed by the cell wall

Bacterial Anatomy 2. Structures external to the bacterial cell wall

1. Capsule; glycocalyx 2. Fimbriae/pili 3. Flagella

A 60 yo male presents to the ER with complaint of fever, cough, and breathlessness for the past two days. On examination, his vitals are as follows: -Temperature - 101 degree F -B.P - 140/86 mmHg -Pulse - 90/minute -Respiratory rate - 16/minute -Respiratory system - decreased breath sounds in the left lower lobe with increased vocal fremitus and resonance -While in the ER he expectorates purulent sputum. Gram stain of the sputum shows GRAM POSITIVE bacteria with a large CAPSULE surrounding it. 1. What is the biochemical nature of the CAPSULE and the CELL WALL. 2. Does the capsule and cell wall contribute to the virulence? Give

1. Capsules are polysaccharide with the exception of the capsule of BACILLUS ANTHRACIS (which is polypeptide). The gram positive cell wall is composed of peptidoglycan 2. YES! The capsule resist phagocytosis because it is slippery and slimy (like catching a bar of soap). It also resist compliment activation and can also act as a ligand for adhesion.

Select the appropriate steps for the management of these conditions

1. Stabilize the patient. 2. Clear the bloodstream infection with empiric antimicrobial therapy. 3. Remove the underlying focus of infection such as purulent exudate, gangrenous tissue, infected organs or foreign bodies. 4. In the cases of toxemia, specific anti-toxin can be administered.

Toxoplasma Bradyzoites in tissue cyst

2 forms, Tachyzoites and bradyzoites Tachyzoites in the RBCs and Bradyzoites in the tissue cyst

Toxoplasma Tachyzoites

2 forms, Tachyzoites and bradyzoites Tachyzoites in the RBCs and Bradyzoites in the tissue cyst

Coxsackie virus

A Entero-virus that causes myocarditis and pericarditis -When a pt comes to you with acute CP, you can think of MI. But you can also think of pericardiits and myocarditis

Which of the following best describes the growth rate of bacteria during the exponential phase of growth? A. Constant B. Decreasing C. Increasing D. Zero

A. Constant

Chagas Disease

Agent: Trypanosoma cruzi Armadillo under huts -> Reduviid bug -> bug infects man

Breakdown

All Life: 1. Archea 2. Eubacteria -> Monera 3. Eukarya -> Protista, FUNGI, Plantae, Animalia

Construction of cell wall

Amount/thickness of peptidoglycan varies in -Gram-positive bacteria—many layers thick -Gram-negative bacteria—few layers Mycoplasma do not have a cell wall (sterols); no defined shape; not susceptible to beta-lactam drugs *Instead of peptidoglycan they have STEROLS *The Beta-lactam drugs only target the penicillin binding proteins that are important for building the bacterial cell wall (involving peptidoglycan), and MYCOPLASMA DOES NOT HAVE PEPTIDOGLYCAN!

Trematodes

An example of intestinal trematodes is: -Fasciolopsis buski ("porcine liver fluke") Examples of tissue trematodes include: -Fasciola hepatica ("sheep liver fluke") -Clonorchis sinensis ("Chinese liver fluke") -Paragonimus westermani ("oriental lung fluke") Examples of blood trematodes include: -Schistosoma mansoni (intestinal bilharziasis) -Schistosoma japonicum ("oriental blood fluke") -Schistosoma haematobium (urinary bilharziasis)

A 34 yo man habitually addicted to IV drugs, presents with a high fever. O/E the physician notes a raised body temperature and presence of painful nodules on his palms. A fundoscopy demonstrates roth spots. Two blood cultures yield a gram positive cocci that are catalase positive and coagulase negative. Which of the following features is mainly responsible as the causative agent of this condition? A. Expression of Protein A B. Glycocalyx giving rise to biofilms C. Pili promoting adhesion D. Coagulase production E. Hyaluronidase production

B. Glycocalyx giving rise to biofilms -Catalase positive = Staph -Coagulase negative = not staph aureus -This is clearly speaking of S. epidermidis A - Protein A is a feature of S. aureus C - Many gram negatives produce pili (like pseudomonas) D - This is a distractor as the vignette already says that it is does not produce coagulase E - Hyaluronidase production is by staph aureus and streptoccus pyogenes. Staph epidermidis never produces pyogenes If given an IV drug abuse question, you must differentiate between S. AUREUS, S. EPIDERMIDIS, PSEUDOMONAS, CANDIDA as they are commonly associated with it

Babesia microti

Babeisa in a thin blood smear, Giemsa stain -pathogen found in the RBCs

MODE OF INFECTIOUS DISEASE TRANSMISSION

Contact transmission Human to human -Direct (horizontal transmission) *Touching *Kissing *Sexual intercourse *Mother to child (vertical transmission) **Transplacental/ within birth canal/ breast milk, fingers -Indirect *Fomites (linen, paper tissue, needles, tooth brush, toys, money, diapers, drinking glasses, medical equipment, door knobs, flush handle) -Blood borne *Transfusion *Iv drug use -Droplet transmission *Droplet nuclei through coughing, sneezing

Cyrptosporidium parvum Causes intractable diarrhea

Cryptosporidium parvum oocysts stained with modified acid-fast. Against a blue-green background, the oocysts stand out in a bright red stain.

Enatmoeba histolytica cyst

Enatmoeba histolytica consist of both trophozoite and cyst form

Mechanism of action of diphtheria toxin produced by Corynebacterium diphtheriae

For example: -Diptheriae produces and EXOTOXIN -The B portion of the exotoxin binds to the cell, the A portion gets internalized -The A portion of the exotoxin goes on to inactivated ELONGATION FACTOR 2 which results in the inhibition of protein synthesis on the ribosomes and then cell death

Giardia lamblia Cyst

Giardia lamblia has both trophozoite and cyst form

Giardia lamblia Trophozoite

Giardia lamblia has both trophozoite and cyst form

Treatment, Prophylaxis

I.E due to S.viridans: -Penicillin G/ Ceftriaxone. -Gentamicin may be added. -In cases of RHD cases undergoing dental treatment, antibiotic prophylaxis is given with Amoxicillin prior to the dental procedure

Innate

Individual resistance -Seen in individual members of a species -Sickle cell trait - resistant to malaria Anatomic barriers -Eye - tears, lysozyme, blink -Skin - structural, sweat, sebum, propionic acid, normal flora -Urogenital tract - flushing, acid pH, lysozyme, vaginal lactic acid Anatomic barriers -Respiratory tract - cough, sneeze, cilia, mucus, phagocytes -GIT - acid, bile, normal flora, peristalsis Complement -C3a, C5a - proinflammatory -C3b -MAC

What must you be able to demonstrate in the laboratory, in order to prove that the fungus is dimorphic?

Interconversion between the two phases (yeast to mold and mold to yeast)

Note about Transfection

Introduction of nucleic acids (DNA or RNA) into eukaryotic cells Useful for gene therapy (ie. thyroid cancer)

Surface components: S.aureus

MSCRAMM: Microbial Surface Components Recognizing Adhesive Matrix Molecules are surface components bound to peptidoglycan that help in adherence to host proteins/ tissues. Eg: -Protein A binds to Fc receptor of Immunoglobulin G. -Binds Fc component of IgG and inhibits phagocytosis -Bound coagulase binds to fibrinogen, converts it to fibrin and helps to clump the bacteria.

Entry - Enveloped viruses

Mediated by two mechanisms -Direct fusion -Receptor mediated endocytosis

Factors which influence the resistance of microorganisms to various procedures employed (cont....)

Microbial load -All other conditions remaining constant, the larger the number of microbes, the more time a germicide needs to destroy all of them. Physical and chemical factors -temperature, pH, relative humidity, and water hardness *pH and water hardness particularly affect chemical disinfectants *Relative humidity - gases Biofilms -Dental unit water lines, whirlpools, medical devices (contact lenses, pacemakers, heamodialysis systems, central venous catheters) -Bacteria within biofilms are up to 1,000 times more resistant to antimicrobials than are the same bacteria in suspension

Acid-Fast Stains - example..

Mycobacterium tuberculosis in lung, Ziehl-Neelsen Acid-fast stain

A 27 yo health care worker gets a needle stick injury. The source is found to be positive for a viral pathogen that is sensitive to drying. As the workers vaccinations are up to date, what is the mechanism by which she is able to resist infection? If she was not vaccinated, what would be the immune mechanism(s) by which she would be able to clear the infection?

Neutrolizing Antibodies (humoral immunity) -This is an enveloped virus as it is sensitive to drying! When you hear needle stick injury of a health care worker, you can many times raise concerns for HEP B VIRUS! However, this pt got the vaccine. The vaccine only contains purified heptatitis B surface antigens so you get good protective neutralizing antibodies. Therefore there is a humoral immunity in this person which will protect her against hep B. Now Hep B vaccine produces neutralizing antibodies which neutralize the target by binding the VAPs and not allowing the hep B virus to get into the hepatocyte

Acid-Fast Stains - example

Nocardia asteroides filaments stained by the modified Kinyoun acid-fast method

Bacterial Metabolism (cont..)

O2 generates two extremely toxic substances -Superoxide anion (O2-)—detoxified by superoxide dismutase -H2O2—broken down by catalase Anaerobes cannot make superoxide dismutase and catalase *This is why they are killed easily in the presence of oxygen -Whether they can grow anaerobically depends on their ability to ferment -Obligate anaerobes extremely sensitive to O2 and grow anaerobically using fermentation, exclusively Aerobes—bacteria that grow in presence of O2 and can make superoxide dismutase and catalase -Obligate aerobes CANNOT FERMENT

Additional cell wall components in Gram negative bacteria -VERY THIN PEPTIDOGLYCAN LAYER -NO LIPOTECHOIC ACIDS, NO TECHOIC ACIDS -ENDOTOXIN = Lipopolysaccharides

Outer membrane—asymmetrical -Inner leaflet: phospholipids, proteins, etc. -Outer leaflet: mainly LPS LPS (endotoxin) Porins -Transmembrane -Allow diffusion of hydrophilic molecules < 700 Da. Excludes large molecules -Allow communication and passage of molecules between the bacteria and its external environment Thin peptidoglycan layer located in PERIPLASM *The space between the cytoplasmic layer and the outer layer is called the PERIPLASM! *Gram positive has no outer membrane and has no periplasmic place -Gram-negative bacteria (eg. E. coli) more resistant to penicillins *This is because abx that wants to kill a gram neg bacteria has to cross the OUTER MEMBRANE FIRST to get to the PBPs (This is not a problem in Gram + as they do not have an outer membrane) *Beta lactum abx are not effective in those that wont let it pass through

Additional cell wall components in Gram-negative bacteria

Periplasm -Located between CM and OM -Site for proteins (translocation proteins) and enzymes (eg. lactamases, nucleases, proteases), secreted by cell which have important role in transport, breakdown of nutrients and of antibiotics

Disinfection

Physical vs Chemical agents

Factors which influence the resistance of microorganisms to various procedures employed

Presence of organic matter -Serum, blood, pus, or fecal or lubricant material *chemical reaction between the germicide and the organic matter -> resulting complex that is less germicidal or nongermicidal, -> less active germicide available for attacking microorganisms *Physical barrier Duration of exposure -Use as directed on label

Role of protein A in immune evasion

Protein A is covalently linked to the peptidoglycan and has an affinity for the Fc part of IgG . It binds this portion making it unavailable to attach to Fc receptors on macrophages and hence prevents phagocytosis. Only S.aureus has Protein A

Morphology, staining: Wall structure

Quellung rxn for identification of S. pneumoniae -Anti-sera + bacteria + methylene blue *Methylene is able to stain the body of the bacterium *The anti-sera (contains antibody) can bind to the capsule (antigenic) and create a larger structure that produces a greater refractiveness that allows microscopic ID to be easier -Increased refractiveness of capsule for microscopic identification

Prevention

Regular hand washing by hospital personnel helps to break the chain of transmission. Regular screening of health care personnel and preoperative screening of patients for MRSA carriage. Swabs: nasal/ cutaneous swabs Use of Mupirocin ointment for carrier state. Vancomycin therapy for persistent carriers.

R plasmid

Resistance transfer factors Partly responsible for high frequency spread of antibiotic resistance genes among some bacteria

Laboratory diagnosis

Sample: Blood culture THEN Microscopy Culture Anti fungal susceptibility test Identification - Germ Tube Test

Spores

Sexual Spores -Ascopores -Basidiospores -Zygospores Asexual Spores 1. Conidis (ABC) -Arthroconidia -Blastoconidia -Chlamydoconidia 2. Sporangiospores

Additional cell wall components of Gram-positive bacteria

TECHOIC ACIDS (wall TA) -Inserted into peptidoglycan layer; covalent linkages -Major component of their cell wall -Indicated by the extensions of the cell wall LIPOTECHOIC ACIDS (membrane TA) -Extend through peptidoglycan and link to CM glycolipids -These EXTEND through the cell wall

Mechanisms of Bacterial Horizontal Gene Transfer Vertical gene transfer - what happens when the bacteria divides by binary fision where all the parent genes go to daughter cell

Transformation Conjugation Transduction

Physical requirements for growth (continued...)

pH -Most prefer neutral pH of 7 -Acidophiles: optimum pH ~3 for growth -Note that Helicobacter pylori is not an acidophile *It has urease which gives it an alkali surrounding, not effected by the acid

Phenotypic mixing and masking

-When a cell is infected with two related virus there are two possibilities -Phenotypic mixing: genome of one virus type takes on the surface protein of the second virus also Virus no. 1 & 3 -Phenotypic masking (transcapsidation): entire surface protein of one virus covers that of the second virus genome Virus no. 2 & 4; pseudotype

In the OR, the patient is operated on. 1. What is the role of the UV lights in OR disinfection? 2. What are the health hazards of operating under UV lights and what precautions can the personnel take to mitigate these risks?

1. The lights in the OR are UV NON-IONIZING RADIATION! 2. Surgeons/Nurses/Techs must wear protective gear and eye wear to protect themselves from the UV light. -UV is a SURFACE DISINFECTANT! UV has a good level of disinfection but it has poor penetrating power -In the OR it is disinfecting the operating field. One of the big problems we have today is multi-drug resistant pathogens, hospital acquired infections, etc, and they found that focusing the UV on the operating field it goes a long way in reducing nosocomial infections -Going outside is also a disinfectant because sunlight is a natural UV light/heat disinfectant. This is why hanging clothes outside is good. Also in many countries, many hospitals have large windows to get into the hospital to disinfect the surfaces and the air. When you make a hospital with air-conditioning and tinted windows, you get recirculated air, must be filtered, etc. This is why hospital design is a whole different field in itself.

Two mutants of polio virus (picornaviridae) one mutated at gene X and the other mutated at gene Y have been isolated. If a cell is infected with each mutant alone no virus is produced. In a cell infected with both mutants which of the following is most likely to occur? A. Complementation between the mutant gene products may occur and if so both X and Y progeny viruses will be made B. Phenotypic mixing may occur and if so both X and Y progeny viruses will be made C. Re-assortment of the genome segments may occur and if so both X and Y progeny viruses will be made D. The genome may be transcirbed into DNA and if so both X and Y progeny viruses will be made E. Recombination between the genome segments may occur and both will kill the infected cell

A. Complementation between the mutant gene products may occur and if so both X and Y progeny viruses will be made -If virus 1 has a mutation in X and virus 2 has a mutation in Y, in order to be successful, they must help each other out. *An exception to this rule is that HEP B gives help to HEP D without needing help back. HEP D is dependent on HEP B - which is another example of complementation B - This is essentially "wolf in sheep's clothing" as inside is something, when outside has another covering. VAPs determine the host range of the virus, therefore if there is masking - you are potentially altering the tropism of the virus. If there is phenotypic mixing, the tropism is extended (because the host range is still present + the other viurs' host range). However with mixing and masking, the core of the genome is still the same! So once the genome gets inside of the cell, it will be as the original viral genome. C - This is not a member of BOAR! *Bunyaviridae *Orthomyoxoviridae *Arenaviridae *Reoviridae -This is why the family is given in parenthesis D - Still will have 2 separate viruses with a mutation in X or Y. There must be some sort of genetic intervention. They have to help each other out. E- Recombination is like Meiosis 2 crossover. There is crossing over between 2 similar virus and exchange of genetic segments. If one is lacking gene x and the other is lacking gene y, its not a good idea to crossover because the same issue will arise

A 65 yo man presents with fever, swelling on his finger tips and streaks on nails. He had undergone knee replacement surgery a month ago. He recollects having suffered from repeated sore throats when he was a teenager. On examination, the patient is febrile, and has tenderness over his joints. In addition, he has tender lesions on his plams, a murmur is detected in the 5th intercostal space. What most likely will be isolated from blood culture samples taken from this patient? A. A gram positive coccus that is alpha hemolytic B. A gram positive coccus that is novobiocin resistant C. A gram positive coccus that can grow in high salt concentration D. A gram negative bacillus which is oxidase negative E. A gram negative bacillus which is very fastidious

C. A gram positive coccus that can grow in high salt concentration -Finger swelling on the fingertips (painful-oslers, non-painful-janeway), streaks on nails (splinter hemorrhages) -Recent knee surgery, predisposed for organisms that are found on the skin -Repeated sore throats - possible could result in RHD -Joints tender, new murmur in 5th intercostal space (MITRAL VALVE) - RHD affects the MITRAL VALVE! -The dx now is infective endocarditis. By the time RHD develops, most of the time, you will not be able to isolate the organism from the throat or the heart. The RHD caused a non-infective endocarditis, caused by immune mediated mechanism. Now this left the valves susceptible to organisms -So now, the organisms you would think of first A- Strep viridans, strep bovis. Unlikely unless there was a tooth extraction which pushed those into the blood B - Staph saprophytius - no. But if it were sensitive to the test, then yes, since staph epidirmidis is C - Another gram + coccus that can grow in high salt concentration is staph aureus. So if you have both staph epidermidis and staph aureus in the options, always pick aureus when it comes to salt concentration because it is much more salt loving. Enterococcus is also salt loving but the predisposing condition is genital-urinary manipulations, gut manipulations or catherizations. D - This is not pseudomonas, pseudomonas is OXIDASE POSITIVE but this is not that E - HACEK group are very fastidious but this is not that

To investigate how a species of gut bacteria acquires resistance to antibiotics, the antibiotic-sensitive bacteria are mixed with fragments of DNA wherein each fragment includes the gene for amoxicillin resistance. If the bacteria subsequently become resistant to this antibiotic, which mechanism was most likely responsible? A. Conjugation B. Generalized transduction C. Specialized transduction D. Transformation

D. Transformation

Urinary Tract Infections are another common cause of sepsis -Many of the pt's admitted in the hospitals. Many have urinary catheters and it can help to introduce the organisms into the bladder from the fecal flora. Biofilms are the virilence factor that helps the organisms to stick to the catheter and rise up. Organisms can actually accend up, cause pyelonephritis and from there it can get to the blood stream and cause sepsis

Escherichia coli Klebsiella Pseudomonas Proteus Enterococcus spp

Differences between prokaryotic and eukaryotic pathogens (continued..)

Eukaryote (eg. yeast) -Organelles: Present -Cytoplasmic membrane: Contains sterol -Cell wall: Absent; (seen only in in fungi) -Reproduction: Sexual, asexual -Respiration: Via mitochondria Prokaryote (bacteria) -Organelles: Absent -Cytoplasmic membrane: Does not contain sterol* -Reproduction: Binary fission -Respiration: Via cytoplasmic membrane

Differences between prokaryotic and eukaryotic pathogens (continued)

Eukaryote (eg. yeast) -Regulation of gene expression: Single genes; mono cistronic -Type and level: Post-translational regulation --Transcription -Relation of transcription and translation: Separate: transcription in nucleus, translation in cytoplasm -mRNA processing: Poly A at 3' end; cap at 5' end, splicing --Translation of mRNA -First amino acid: Methionine -Ribosomes: 40S + 60S = 80S Prokaryote (bacteria) -Regulation of gene expression: Operon-polycistronic mRNA -Type and level: Mostly transcriptional --Transcription -Relation of transcription and translation: Separate: Concurrent -mRNA processing: Rare --Translation of mRNA -First amino acid: Formylated methionine -Ribosomes: 30S + 50S= 70S *This feature is what is exploited during antibiotic creation

Differences between prokaryotic and eukaryotic pathogens

Eukaryote (eg. yeast) -Size: > 5µm *Eukaryote microbes are much much larger then bacteria -Nuclear Structure: Nucleus, nuclear membrane -Chromosomes: Many -Haploid (Ie. Sex cells)/Diploid -Genome organization: Single gene copies; repetitive DNA -Co-linearity of gene with RNA: Introns within gene Prokaryote (bacteria) -Size: 0.5-3.0 µm -Chromosomes: One (some >1); circular or linear -Haploid (1 copy) -Genome organization: Single gene copies -Co-linearity of gene with RNA: No introns; precise sequence *Bacteria don't have the space for extra introns

Physical methods of Disinfection

HEAT -Method: Boiling -Activity Level: High -Spectrum: Most pathogens, some spores -Uses/Comments: General -Method: Pasteurization -Activity Level: Intermediate -Spectrum: Vegetative bacteria -Uses/Comments: Beverage, plastic inhalation therapy equipment -Method: Microwave -Activity Level: High -Spectrum: Vegetative forms -Uses/Comments: General RADIATION -Method: UV -Activity Level: High -Spectrum: All -Uses/Comments: Poor penetration FILTRATION -Method: Cellulose acetate -Activity Level: High/intermediate -Spectrum: Vegetative forms -Uses/Comments: Heat labile liquids

Physical requirements for growth

Temperature -Depending on temperature range at which they grow best bacteria classified as: -Psychrophiles: growth at cold temperatures (≤ 15-20 C) -Mesophiles: (30 - 37 C ) most pathogenic bacteria -Thermophiles: growth at warm temperatures (optimum ~50-60 C) -Optimum temperature: that at which maximum growth occurs

Terminology in viral taxonomy

VIral Family: -viridae e.g. Herperviridae Viral Subfamily: -virinae e.g. alphaherepesvirinae Viral Genus: -virus e.g. Herpesvirus -strains

Pathogenesis of Infective endocarditis

What are cardiac vegetations? What are their implications in : a. The laboratory diagnosis of the disease? b. In the radio-diagnosis of the disease? c. In the treatment of the disease including the choice of antibiotics, route of administration, and duration of treatment?

Chaga's disease/ South American Trypanosomiasis

-A flagellate parasite affecting bloodstream and tissues -This is a protozoa - it is SINGLE CELLED!

Gram Positive Cocci

-Staphylococcus aureus -CoNS (Coagulative negative staph): Staphylococcus epidermidis S.lugdunensis -Streptococcus viridans group -Enterococcus spp; S.bovis Fungus: Yeast like cell that is GRAM POSITIVE! -Candida

Clinical Classification of mycoses

-Superficial mycoses -Cutaneous mycoses -Subcutaneous mycoses -Systemic mycoses -Opportunistic mycoses

Subcutaneous mycoses

-The subcutaneous mycoses are fungal infections of the subcutaneous tissue and only rarely spread systemically. -They usually form deep, ulcerated skin lesions or fungating masses, most commonly involving the lower extremities. *Found in the lower extremities mostly because they are most likely to be injured in the environment to where the spores of this fungi are incoulated -The causative organisms are soil saprophytes which are introduced through trauma to the feet or legs.

A few general facts....

-naked genomes of DNA viruses (except poxvirus) and the positive sense RNA viruses (except retroviruses) are said to be infectious -Most RNA viruses replicate in the cytoplasm (except orthomyxoviruses and retrovirus) *(+) ss RNA viruses - genome serves as mRNA for early protein synthesis *(-) ss RNA viruses carry a virion associated RNA dependent RNA polymerase to produce initial mRNAs *Most viral RNA polymerases work at a fast pace but are also error prone, causing mutations.

Growth characteristics

Oxygen requirement -Aerobic (Need AIR/OXYGEN) *Obligate aerobe -Anaerobic *Obligate - if oxygen is present, they will die *Facultative: use anaerobic or aerobic metabolism -Microaerophilic: low oxygen tension/concentration. These are more successful under low concentration of oxygen -Aerotolerant: grow in presence or absence of oxygen; Host cell? -Obligate intracellular - bacteria that can only reproduce in the host cell (chlamydia, rickettsia) *Can't provide ATP by themselves so need the host cell -Facultative intracellular

Treatment of I.E

First line of antibiotics: -Nafcillin / Cephazolin/ Ceftriaxone + Gentamicin *These are *These are the first line because you can give without knowing exactly what the organism is and you must start treatment! Second line of antibiotics: -In case the isolate is MRSA : Vancomycin

Pathogenic actions of bacteria

-Tissue destruction -Toxin production

Transduction

-Transfer of DNA between bacteria via phage -Generalised vs specialised

Spectrum of diseases: Pseudomonas

-Burns -Malignant Otitis externa -Hot tub folliculitis -Endophthalmitis -Septicemia -Infective endocarditis -Ventilator associated pneumonia -Nosocomial UTI -Biofilms in Cystic Fibrosis (Biofilms are involved in numerous diseases. In cystic fibrosis patients have pseudomonas infections that result in antibiotic resistant biofilms

Cytoplasmic Membrane

-Phospholipid bilayer -There are a number of integral proteins that allow substances to be passed in and out of the cytoplasm but this is SELECTIVELY PERMEABLE -Performs functions of mitochondria and other organelles in eukaryotes *Energy production, oxidative metabolism -Mg2+ and Ca2+ important for membrane integrity *In research this is exploited if you want to disrupt a bacterial cell membrane

A few general facts.... (cont..)

-Picornaviruses, Togaviruses, Flaviviruses, Caliciviruses, Coronaviruses -Genome is in mRNA sense and is translated -(+)RNA genomes are produced via a (-)RNA intermediate. -Reoviruses -(+/-) ds segmented RNA genome acts as template for mRNA -Orthomyxoviruses, Paramyxoviruses, Filoviruses, Bunyaviruses -(-)RNA genome transcribed to mRNA -(-)RNA genomes are produced via a (+)RNA intermediate -Retroviruses -(+)RNA genome is converted to DNA which is integrated into the host chromatin and transcribed as a cellular gene

Escaping phagocytes and phagocytosis

-Some produce enzymes capable of lysing phagocytic cells (e.g., the streptolysin produced by Streptococcus pyogenes or the alpha-toxin produced by Clostridium perfringens). -Some can inhibit phagocytosis (e.g., the effects of the capsule and the M protein produced by Streptococcus pyogenes) or block intracellular killing. -Some can circumvent killing (see table) -Production of catalase by staphylococci can break down the hydrogen peroxide produced by the myeloperoxidase system -Many of the bacteria that are internalized but survive phagocytosis can use the cell as a place to grow and hide from immune responses and as a means of being disseminated throughout the body.

Spread of infection/Viraemia

-Some remain localised: localised at the site of entry; after entry spreads locally to adjacent cells and does not spread systemically *E.g., rhinovirus, influenza (respiratory tract), rotavirus (intestinal tract), molluscum contagiosum and HPV to the skin -Some spread beyond the primary site of replication to multiple organs; *Primary viremia *Secondary viremia

Molecular mimicry

-An oestrogen-binding protein has been isolated from cytosolic fractions of Coccidioides immitis. -Physiologic concentrations of progesterone and 17-β-estradiol stimulate the rate of Coccidioides immitis growth and endospore release. -This information coincides with the recognition of pregnancy, especially during the third trimester, as a major risk factor for disseminated coccidioidomycosis. -In contrast, oestrogen inhibits the conversion of conidial phase to yeast phase of Paracoccicioides brasiliensis

Blastomyces dermatitidis These in the tissue form usually have broad based budding seen in the image

-Antigen shedding - yeast cells shed their immunodominant antigen, WI-1 -Modify their cell wall composition -Cryptococcus neoformans, produce substances such as melanin, which interfere with oxidative killing by phagocytes. -Invasive aspergillosis is HIGHLY ASSOCIATED with neutropenia and impaired neutrophil function -Aspergilli produce catalase, an enzyme that breaks down hydrogen peroxide. (No surprise that there is a strong association of aspergillosis with chronic granulomatous disease)

Adaptive response

-Antigen specific responses: -Adaptive immunity consist of humoral and cell mediated immunity to clear the virus and resolve the infection -Humoral immunity: *secreted antibodies have a major role in resolving lytic infections *Antibody neutralise or inactivate or eliminate (opsonise) free virions - block VAPs *Neutralising antibodies to viral surface proteins prevents infections, targets the virus for destruction by phagocytosis, ADCC or complement system *IgM is a marker of recent or current infection *Secretory IgA protects the mucosal surfaces

Viral escape from immune responses

-Antigenic shift/drift/ variation: HIV, influenza, hepatitis C, rotavirus -Establish latent infection/immunologically silent: Herpes virus, -Hide in immunologically privileged sites: herpes viruses, JC viruses, polyoma, papilloma -Infect immune cells and suppress their function: HIV, measles -Inhibit viral antigen presentation: adenovirus and herpes virus interfere with the presentation of MHC class I proteins for recognition by cytotoxic T cells -Express extracellular immunomodulatory proteins: Pox and Herpes express cytokine receptors that sabotage host response -Inhibition of IFN antiviral activity: adenovirus, Hepatitis B, herpes, polio, influenza -Inactivation of complement: HSV -Release from infected cells of antigen that block antibody neutralisation: Hepatitis B -Syncytia formation: direct cell to cell spread; HSV, VZV, paramyxovirus, retrovirus

Endotoxin (cont)

-At low concentrations, endotoxin stimulates the development of protective responses, such as fever, vasodilation, and the activation of immune and inflammatory responses -In high concentrations seen in gram-negative bacterial sepsis, the systemic response to these can be overpowering, resulting in shock and possibly death. -High concentrations of endotoxin can also activate the alternative pathway of complement and production of anaphylotoxins (C3a, C5a), contributing to vasodilation and capillary leakage. In combination with TNF-α and IL-1, this can lead to hypotension and shock. -Disseminated intravascular coagulation (DIC) can also result from the activation of blood coagulation pathways.

Fungal dimorphism

-Can grow as yeast or molds -If this variation in form is subject to variation in the growth temperature - thermal dimorphism -YEAST form is the PARASITIC FORM *Seen in the tissues of the human body *Culture at 37 degree C (because this is close to human body temp) -Mold form *Saprophytic form *Culture at 25 deg C

Gram-Negative Summary

-Capsule/polysaccharide Outer Membrane -Hydrophobic phospholipid bilayer with OM proteins and porins -Porin Proteins Outermembrane -Proteins (OMP) Periplasmice space -Space between (IM & CM) -Peptidoglycan Inner Membrane (IM) -Hydrophobic phospholipid bilayer -Antiphagocytic & immunogenic -Hydrophobic; LPS; Oantigenic & lipid toxic -Passive transport of aqueous minerals -Virulence, attachment -Extracellular enzymes; reduces osmotic pressure -Rigid support of cell; protection from osmotic damage -Membranous matrix for enzymes, respiration, cell wall synthesis; PBP

Schistosoma hematobium

-Casues Schistosomiasis of the bladder -The spine (in the L pic, helps us identify the egg and is responsible for causing trauma to the bladder wall). -As a result, you get terminal hematuria (or presence of blood in the urine towards the end of the act of micturation)

Helminths

-Cylindrical worms (Class nematoda) -Flatworms *Tapeworms (Class cestoda) *Flukes (Class trematoda)

Instrumentation-Cystoscopy/ Colonoscopy

-Cystoscopy -Colonoscopy -Long standing urinary catheterization -Biliary tract surgery *All the above can lead to: Enterococcus fecalis or E. faecium -Colon carcinoma/ GIT malignancies: predisposes for bacteremia with S. bovis gp. (Streptococcus gallolyticus) -> Infective endocarditis due to streptococcus bovis

Types of infections

-Cytolytic infections -Persistent infections: --Chronic infections --Latent infections --Immortalizing/Transforming infections: *DNA tumour viruses *RNA tumour viruses -Slow infections

Types of infections (cont...)

-Cytolytic infections: *viral replication leads to cell death; rapid antibody mediated immune response; e.g., polioviruses, Togaviruses, Herpes viruses, pox viruses. Expression of viral antigens on the cell surface and *disruption of the cytoskeleton can change cell-to-cell interactions and the cell's appearance, making the cell a target for immune cytolysis. -Abortive infections (failed infection): viral mutants cause failed infections; cell survives -Persistent infection (replication without cell death) *Chronic infections: non lytic and productive; continuous production of virions or viral components occurs without cell lysis or immune resolution; e.g., hepatitis B, hepatitis C, HIV, HTLV; cell senescence

Bacterial Anatomy 3. Structures enclosed by cell wall

-Cytoplasmic membrane -Nuclear region (nucleoid) -Plasmids -Ribosomes -Inclusion bodies -Endospores

More Definitions

-Germicide: An agent that can kill microorganisms, particularly pathogens. -Antiseptics: Germicides applied to living tissue and skin; disinfectants are antimicrobials applied only to inanimate objects. -Cleaning: The removal of visible soil (e.g., organic and inorganic material) from objects and surfaces and normally is accomplished manually or mechanically using water with detergents or enzymatic products. -Decontamination: Removes pathogenic microorganisms from objects so they are safe to handle, use, or discard

Virulence factors and pathogenesis of Candida

-Glucomannan receptor on yeast binds to fibronectin on epithelial cell/ to extracellular matrix -Formation of hyphae that invade (they start digging deeper and deeper) + Production of proteinases -> Digestion of tissue and destruction

Some important Gram negative bacilli causing sepsis

-Gram negative organism: E.coli -Common underlying cause: Urinary tract Infection -Gram negative organism: Klebsiella -Common underlying cause: UTI/ Pneumonia -Gram negative organism: Pseudomonas -Common underlying cause: Burns infections/ UTI/ Pneumonia -Gram negative organism: Proteus -Common underlying cause: UTI

Interferons

-IFN-alpha and IFN-beta secreted by infected cells induce an antiviral state in neighboring non infected cells, thus limiting spread of infection within host tissues -Host species specific not virus specific -IFN-alpha produced by leukocytes -IFN-beta produced by fibroblasts and epithelial cells -IFN-gamma produced by NK cell and activated T cells -Ds RNA potent inducer of interferons

Nature of Bacteria

-In the body, out-number human cells 10:1 *Many are pathogens but do not cause disease -Human genome includes 22,000 protein-coding genes. -Human microbiome contributes ~8 million protein-coding genes -Bacteria essential for our survival

Clinical course of viral diseases

-Incubation period -Acute versus persistent infecions -Slow infections

So we have our NAM's and our NAG's. What might happen if the PENTA GLYCINE BRIDGE were not present?

-It will compromise the integrity of the wall. You can have many layers but if they are not connecting each other, you can technically peel each layer off one by one. Having all the layers there together and connected will make it a stronger wall

Viral Cytopathogenesis (cont)

-Disruption of cytoskeleton: accumulation of non enveloped viruses; HSV -Alteration of cell permeability: Togavirus, Herpes viruses -Inclusion bodies: some viral infections cause characteristic changes in the appearance and properties of target cell *Negri bodies (intracytoplasmic); Rabies *Owl's eye (intranuclear); CMV *Cowdry type A (intranuclear); HSV, SSPE (measles) *Intranuclear basophilic; adenovirus *Intracytoplasmic acidophilic; Poxvirus *Perinuclear cytoplasmic acidophilic; reovirus

Skin

-A good water repellant barrier to infection -What happens if you get a cut in the skin? *For example, Staphyloccocus epidermidis, which is a part of the normal flora on skin, can cause catheter or prosthesis infections

Entry into the human body 1. Adhesion 2. Colonisation 3. Invasion (some of them)

-A good water repellent barrier to infection -What happens if you get a cut on the skin? *For example, Staphylococcus epidermidis, which is a part of the normal flora on skin, can cause catheter or prosthesis infections

Injury Mycotoxins are produced in the environment and NOT IN THE BODY!

-A number of fungi do produce exotoxins, called mycotoxins, in the environment but not in vivo. -The circulating products of Cryptococcus neoformans have been shown to down-regulate immune functions. -The injury caused by fungal infections seems to be due primarily to the destructive aspects of delayed-type hypersensitivity (DTH) responses as a result of the inability of the immune system to clear the fungus. -In this respect, fungal infections resemble tuberculosis more than any other disease.

Bacterial Growth Curve

-Lag period: growth not detectable, cells adjusting to the new environment (there is growth but it is just not detectable!) *This is where it is "waking up" and getting ready to start growing -Logarithmic/exponential growth: phase of constant, maximal growth rate, generation time is constant; cell number, mass increase at an exponential rate -Stationary phase: nutrients depleted, waste accumulates; growth limited *Ie bacterium starving at this point *No more multiplcation *After this phase the bacteria can recover or the death phase -Death

Types of virus mutants

-Lethal mutants - mutation inactivates essential genes, virus cannot replicate -Deletion mutant - loss or selective removal of a portion of the genome and the function it encodes for -Plaque mutants - differ from the wild-type in the size or appearance of the infected cells -Host range mutants - differ in the tissue type or species of target cell that can be infected -Attenuated mutants - variants that cause less serious disease in animals and humans

What are parasites?

-Living organisms that obtain food and shelter by living on or within another living organism -The parasite derives all benefits from association and the host is harmed → a parasite disease -Obligate parasites - can live only in association with a host -Facultative - can live both in or on a host as well as in a free form

Bacterial Anatomy 1. Cell wall/cell envelope

-Major component: peptidoglycan/murein *Mureoin is a type of peptidoglycan and sometimes used interchangeably *Peptidogylcan has NAM, NAG repeating layers -Functions: *Protection from osmotic stress, environmental stress (innate immune response). Bacteria are single cells and have no control over their environment, so the cell wall helps them survive with this *Maintains cell shape, rigidity *Role in cell division -Includes an outer membrane in GRAM-NEGATIVE bacteria -Exception: Mycoplasma lacks a cell wall - therefore they have no peptidoglycan

Tissue destruction

-Mediated by *Enzymes *Metabolic By-products - acids, gas, and other substances -For example, Clostridium perfringens, the causative agent of gas gangrene, establishes infection in oxygen-depleted tissues -These anaerobic bacteria produce enzymes like phospholipase C, collagenase, protease, and hyaluronidase, several toxins, and acid and gas from bacterial metabolism, which destroy the tissue.

Strep viridans gp: Role in health and disease and pathogenesis

-Members of the viridans group are normal oral flora and are relatively avirulent -S. mutans produces dextran (biofilm) Glues the S. mutans to teeth forming plaque which can lead to dental caries. -Virulence factor: Production of Biofilms: dextrans -Any of these can spread into blood THEN Blood stream infection -Strep viridans group: -S. mutans -S. salivarius -S. mitis -S. sanguis -S. anginosus -Predisposing Activities: Tooth extraction Dental work up w/o antibiotics -Rarely HACEK (which is gram negative). Also HACEK is fastidious, and takes longer to culture in the lab) THEN Subacute Endocarditis of abnormal or damaged heart valves -Patients with RHD should be given antibiotic prophylaxis before dental manipulations

Laboratory diagnosis of Sepsis

-Microbiology laboratory tests: -Blood cultures: Aseptically withdrawn blood should be cultured *Blood is being used as a sample from the patient to grow an organisms *While with urine samples you only take about 3-5mls, and wound samples you just dab the wound, in blood cultures you must get about 10-15ml of blood. You have to then add that blood to 2 sets of blood cultures, taken from different sites perferably, over a period of 24 hours. You then add it to bottles, like broth. Then you incubate the broth, then you notice there is growth -> then you subculture it to the appropriate agar to see the colonies. You add so much blood because you want to increases the chances of recovering the organism from the bloodstream, also small quantities of blood, the compliments of that area might be enough to kill the pathogen in that area diminishing the chances of recovering the organism in the blood culture. So therefore you must get large samples! -At least two sets of blood cultures should be obtained over a 24-h period -During intermittent fever, bacteremia is most prominent 0.5 h before the spike. Blood drawn at this time is more likely to contain detectable bacteria. -Culture of Sputum/ Urine/ CSF etc. to determine the source of sepsis. Other tests to evaluate a case of sepsis: -Serum electrolytes -CBC with differential -Procalcitonin (PCT) -C-reactive protein -BUN and creatinine -Arterial blood gas

Paracoccidioides braziliensis

-Microscopy - look for Mariner's wheel (Pilot wheel) - Yeast with multiple buds -You see multiple buds coming circumferentially aorund the yeast looking like a mariners wheel

Fungi

-On an average thousands of fungal spores are inhaled or ingested every day. -Despite this ubiquity, clinically apparent systemic fungal infections are uncommon, even among persons living within the geographic habitat of the more pathogenic species. -Relatively few fungi are sufficiently virulent to be considered primary pathogens *Coccidioides immitis *Blastomyces dermatitidis *Histoplasma capsulatum *Paracoccidioides brasiliensis *Sporothrix schenckii -Others are usually cause opportunistic systemic mycoses *Candida spp. *Crytococcus neoformans *Aspergillus spp.

Basic Mycology

-Mykes - mushroom (Greek) -Medical mycology - a study of fungal epidemiology, ecology, pathogenesis, laboratory diagnosis and treatment in humans. -Most fungi are saprophytes (any organism that grows on dead and decaying matter, Ie. bread) -Usually not transmissible from person to person -Exception? DERMATOPHITES! Which cause RING WORM

Diseases associated with Coxsackie B virus

-Myocarditis, Pericarditis. -Bornholm disease/ Devil's grip/ Epidemic myalgia -Aseptic meningitis -Severe systemic disease of newborns All of the above can cause: COXSACKIE B

Cell wall construction—Peptidoglycan -Repeating units of NAG and NAM are joined by B-1,4 glycosidic linkage. Not only do we have single layers, but layers are attached to each other by a PENTAPEPTIDE Each of the NAMs have a PENTAPEPTIDE (5 amino acids) and they are joined to the NAMS, never to the NAGs! -A peptapeptide in one layer is joined with the pentapeptide in the other layer by the PENTA-GLYCINE BRIDGE - via the transpeptidation reaction

-NAM-NAG repeating units -NAM, NAG joined by β- 1,4-glycosidic linkage *Transglycosylation -Pentapeptide attached to NAM *L-Ala, E, Lys or DAP, A, A *DAP = Diamino promolic acid - not found in proteins. Just unique to bacterial peptidoglycan *The terminal amino acid is also lost in the transpeptidation reaction -Overlying pentapeptides joined by 5G bridge *Transpeptidation DAP = diaminopimelic acid; a diamino aa unique to bacterial peptidoglycan; not found in proteins

Innate responses

-Natural killer cells produce IFN- gamma, activates macrophages and inflammatory response -NK cells act to control viral infection till acquired immunity is induced -NK cells kill virus infected cells by natural killer cell cytotoxicity. -Macrophages and NK cells have antibody receptors on their surface that bind to antibodies on infected cells and eliminate them in a process called ADCC -Complement component C3b binds to virions and are digested by macrophages with receptors for C3b by opsonisation

Assembly

-Newly synthesized viral genomes and capsid polypeptides assemble together (watch encapsidation.swf) -Site and mechanism of viral assembly depends on where the genome has been replicated and whether the final structure is a naked capsid or an enveloped virus -Capsids may be assembled as empty structures (procapsids) or may be assembled around the genome (watch encapsidation.swf) -Enveloped viruses - virus surface proteins are inserted into the host nuclear or plasma membrane, virus buds through it acquiring the envelope (watch egress.swf) -Site of budding varies with the virus - flaviviruses, coronaviruses, and bunyaviruses bud into ER and golgi -HSV is assembled in the nucleus and then buds into and out of the ER -In viral replication, errors are made by viral polymerase and during viral assembly -This results in empty virus particles or viruses with defective genomes -Defective or dud viruses can occupy cellular machinery and prevent virus production -These are referred to as defective interfering particles.

Replication

-Next logical step -Most protozoan parasites replicate intracellularly or extracellularly in the human host -Temperature may also play an important role in the ability of parasites to infect a host and cause disease. -For example, Leishmania donovani replicates well at 37°C and causes visceral leishmaniasis, involving the bone marrow, liver, and spleen. -In contrast, Leishmania tropica grows well at 25°C to 30°C but poorly at 37°C and causes an infection of the skin without involvement of deeper organs.

Nuclear region/nucleoid

-No nucleus -Single chromosome, circular, supercoiled ds DNA, more than 1mm long (1000 times the length of the cell); 4000 genes -Haploid

Macromolecular synthesis - Expression of Viral genomes and synthesis of viral components

-Once inside the cell, the genome must direct the synthesis of viral mRNA and protein and generate identical copies of itself -This depends on the structure of the genome and the site of replication -The first step would be transcription, unless it is a (+) ss RNA virus (Baltimore Class IV)

Cell & tissue damage

-Parasitic disease can be established by the elaboration of toxic products, mechanical tissue damage, and immunopathologic reactions -See next two slides

Virulence factors and pathogenesis

-Pili -Slime (because it can have capsules) *Can form biofilms by FORUM SENSING THEN (Attachment and colonization of host tissue) -EXOTOXIN A *Inhibits protein synthesis by inhibiting elongation factor 2 *Diphtheria toxin has similar activity -Proteases -Cytotoxin -Hemolysin -Pyocyanin THEN (Local invasion and tissue damage) -Due to: -Dissemination -Antiphagocytic capsule -Endotoxin (when it dies - which can lead to endotoxin shock) -Exotoxin (even though mostly gram + mostly produce exotoxin)

The role of Super-antigens in shock syndrome Superantigens don't undergo the usual presentation, and is presented differently because it is outside of the groove. As a result, it activates many lymphocytes. Usually, with normal MHC groove processing, you got 0.001% lymphocytes are activated. With superantigens, 20% of lymphocytes get activated -> Many T cells activated -> cytokine storm!

-Some exotoxins formed by gram positive organisms also lead to shock. Eg: Toxic Shock Syndrome -TSST-1 is a super-antigen -Massive non specific T cell activation creating a cytokine storm -Widespread systemic effects

Nutrient requirements for growth

-Non-fastidious bacteria have simple nutrient requirements: source of C, N, energy, H2O, ions -FASTIDIOUS bacteria have very specific growth requirements *Eg. Add cofactors, such as NAD -Salt: source of ions *Note: Halophiles: Marine bacteria, eg. Vibrio species, Staphylococcus (present on skin), Enterococcus (in the gut) *Most bacteria cannot grow/survive in high salt -Siderophores: small molecules used to chelate Fe in 'environment'

T cell immunity

-TH1 inflammatory and cell mediated cytolytic response are necessary for resolution of nonlytic and enveloped viral infection since virus does not kill infected cells -Cytotoxic T lymphocytes (CTLs) recognize viral peptide antigens presented on MHC class I molecules on infected cells and destroy them -As CTLs kill infected cells the source of new virus is eliminated. -Follicular T cells promote the B cell response.

Identification of Enterococci

-Test: Growth in bile and Esculin hydrolysis *Strep bovis: Positive *Enterococcus: Positive *Observation: Black colonies on Bile Esculin agar -Test: Growth in 6.5 % salt *Strep bovis: Negative *Enterococcus: Positive *Observation: Turbidity in test tube with saline solution -Test: PYR hydrolysis *Strep bovis: Negative *Enterococcus: Positive *Observation: Cherry red color on addition of PYR agent

The gamma hemolytic streptococci -Enterococcus fecalis -Enterococcus fecium -Staph is usually the bad when it compared to strep. Usually, you dont hear much about strep antibiotic resistance. However, ENTEROCCOCI (gamma hemolytic) tend to be very resistant!

-Normal fecal flora-most infections are endogenous -Most resistant among all the streptococci -GRAM POSITIVE oval or spherical cocci in pairs/ short chains arranged at acute angles *They look like Ray-bans -On blood agar: usually gamma hemolytic

Receptor mediated endocytosis (Viropexis)

-Some enveloped viruses require an acid pH for fusion to occur and are unable to fuse directly with the plasma membrane. -These viruses are taken up by invagination of the membrane into endosomes. -As the endosomes become acidified, the latent fusion activity of the virus proteins becomes activated by the fall in pH and the virion membrane fuses with the endosome membrane. -The nucleocapsid is then released into the cytoplasm -Orthomyxoviridae, Togaviridae, Rhabdoviridae

Adherence Like both bacterial and viral infections, this is the first step of infection

-The ability to adhere to buccal or vaginal epithelial cells is associated with colonization and virulence. -Adherence usually requires a surface adhesin on the microbe and a receptor on the epithelial cell. -Candida albicans -> *adhesion - mannoprotein components extending from the cell wall *Receptors - fibronectin and components of the extracellular matrix. -Paracoccidiodes brasiliensis produces gp43 which binds to laminin-1 on the host basement membrane

INTERFERONS (cont)

-The best inducer of IFN-α and IFN-β production is dsRNA, produced as the replicative intermediates of RNA viruses or from the interaction of sense/antisense messenger RNAs (mRNAs) for some DNA viruses -Alternatively, inhibition of protein synthesis in a virally infected cell can decrease the production of a repressor protein of the interferon gene, allowing production of interferon. -Nonviral interferon inducers include the following: *Intracellular microorganisms (e.g., mycobacteria, fungi, protozoa) *Activators of certain TLRs or mitogens (e.g., endotoxins, phytohemagglutinin) *Double-stranded polynucleotides (e.g., poly I:C, poly dA:dT) *Synthetic polyanion polymers (e.g., polysulfates, polyphosphates, pyran) *Antibiotics (e.g., kanamycin, cycloheximide) *Low-molecular-weight synthetic compounds (e.g., tilorone, acridine dyes)

Look at the image and tell me everything you know about it?

-This is a alpha hemolysis (partial hemolysis) -Could involve organisms such as Strep viridens, Strep bovis, Strep pneumoniae -Turns the blood agar green -S. Viridans produces DEXTRANS to cause infection. It is gram + cocci in chains. If you want to test this you would first do Catalase (which it will be neg), then bile solubility (where it is insoluble), optochin test (where it is resistant)

Look at the image and tell me everything you know about it?

-This is beta hemolysis -Most common one is S. aureus -It is Gram + cocci in clusters -Catalse positive -Coagulase positive -Has protein A which binds the Fc portion and evades immune evasion and inhibits phagocytosis -Has hemolycins and Leukocydins -Can also produce TSST-1 which can be seen by some of the infections that are caused by staph -Media that can grow staph: *Blood agar *Nutrient agar: golden yellow colonies *Mannitol colonies: yellow colonies *Can grow in salt medium

Infection

-To produce a disease, viruses must enter the host, interact with susceptible cells/tissues, replicate, and produce cell injury. -Humans are infected with viruses by the same basic mechanisms that allow the spread of other microorganisms.

Case -As part of a research project by the CDC, a team of medical interns are posted in Chile to study a cohort of children who developed an acute febrile illness followed by regional lymphadenopathy, myalgia and swelling of the eyelid. -A few children were reported to have developed arrythmias and two had died due to complete heart block. 1. What most likely led to this disease? 2. Which countries have a high prevalence of such infections? Describe the morphology of the parasite 3. Briefly describe its life cycle 4. What is the typical clinical presentation and complications ? 5. How is it diagnosed? 6. What is the treatment and preventive measures to be taken.

1.

Immunity

Innate -Individual resistance -Anatomical barriers -Inflammation -Phagocytosis -Normal flora -NK cells Acquired -T cells -B cells

A patient is brought into the ER after a road traffic accident. After stabilizing him, he is moved to the OR for emergency surgery. The nurse and nursing attendant survey the scene. Suggest and justify the most appropriate method of disinfection or sterilization of the bed linen.

It depends! On healthy people (who are not immunocompromised) you generally can go without disinfecting because a BP cuff is considered as a low level risk device. -If you have a soiled BP cuff from a sweaty pt, it needs to be disinfected.

MODE OF INFECTIOUS DISEASE TRANSMISSION (cont)

Non human to human -Vehicle transmission -Spread of pathogens via air, drinking water, food -Airborne : spread of pathogens more than a meter to the respiratory mucus membrane of a new host via an aerosol *Cough, sneeze, sweeping, mopping, changing clothes, bedlinen, air conditioners -Waterborne : fecal oral infection -Foodborne -Body fluids -Vector transmission -Animals or insects that transmit disease from one host to the other *Biological vectors: not only transmit, also serve as hosts *Biting arthropods, mosquitoes, ticks, lice, fleas, mites *Mechanical vectors **Passively carry pathogens to new hosts on their body parts ***Domestic fly, cockroach ***Food borne infections

RNA, Non-enveloped virus Rhinovirus (common cold) HEPARNA virus (HAV) Picorna viruses P - Polio E - Enterovirus E - Echovirus Co - coxsackie virus

Non-enveloped RNA 1. PicoRNA virus -ENTEROVIRUSES -Poliovirus -Echovirus -Coxsackie virus -Apart from the enteroviruses in the picoRNA there is also: Rhinovirus (common cold) HEPARNA virus (HAV) 2. Calicivirus -NORO virus/ Norwalk virus 3. Reovirus (ds RNA) -ROTA virus

Epidemiology and pathogenesis Enterococcus can adhere well to host, and it also secretes a number of cytolysins and proteases sufficient enough to damage the tissue. Its hard to treat to because of the presence of antibiotic resistance

Normal Gut flora Can colonize urethera, perineum can do 2 things: 1: -UTI -Diabetic foot infections/ wound infections -Biliary tract infections 2: -Activities such as catheterization/ bladder instrumentation/ cystoscopy/ colonoscopy lead to blood stream infections -Endocarditis by Enterococcus fecalis -E. faecium -Virulence factors: *adherence to host surfaces. *Secretion of cytolysins and proteases -Many strains of enterococci can be multi drug resistant and can cause nosocomial infections

Transformation Recipient bacteria takes up naked DNA from the environment

Occurs in 3 steps. The loose DNA must go through: 1. Adsorption 2. Penetration 3. Recombination -Uptake of naked DNA from the environment; may or may not be integrated into the bacterial genome -Competence: ability of bacteria to be transformed *The recipient DNA must have holes in its wall to be able to take up the free DNA *This is very transient *DNA hydrophobic so 'holes' needed in cell wall for uptake -Natural competence genetically and physiologically determined -Received DNA can be assimilated into host genome by—RECOMBINATION (recA)

Inapparent infections

Result if -The infected tissue is undamaged -the infection is controlled before the virus reaches its target tissue -The target tissue is expendable -the damaged tissue is rapidly repaired -the extent of damage is below a functional threshold for that particular tissue. Inapparent or asymptomatic infections are major sources of contagion.

Disease spectrum and virulence factor of Coagulase Negative Staphylococci (CoNS)

S. epidermidis/S. lugdunensis: 1. Catheter and related infections (Introduction of bacteria during an IV infusion) 2. Prosthetic Joint infection 3. Endocarditis (Introduction of bacteria at the time of valve replacement/ via bloodstream infections) *BIOFILMS are formed by S. epidermidis with the help of slime, a polysaccharide that bonds staphylococci to catheters / valves/ shunts, protecting them from antibiotics and inflammatory cells. *Though they are not very virulent like staph aureus, they biofilms help them infect! *Biofilm also known as slime or glycocalyx *Another organism that produces biofilm by strep viridans called DEXTRAN S. saprophyticus -Urinary tract infection in young females *DONT NEED TO KNOW RIGHT NOW!

A 28 yo man presents with high fever with petechiae on the mucus membranes and conjunctivae. On examination, the patient is febrile, and has tenderness over his joints. In addition to a few maculo-papular lesions on his hands, a tricuspid murmur is detected on his chest auscultation. Two blood cultures grow bacteria as shown in picture. Which of the following best describes the growth characteristics of this pathongen? (Picture is clusters of grapes) A. Greenish colonies on blood agar B. Black colonies on Bile Esculin agar C. Creamish colonies on SDA D. Pale colonies on MacConkey's agar

THERE IS NO CORRECT ANSWER HERE! -Tricuspid murmur- could think possible IV drug abuse -We know the image is staphyloccous (unsure of if its epidirmidis or aureus) A - S. aurus would produce BETA HEMOLYTIC colonies on blood agar. S. epidirmidis would produce whitish colonies on blood agar. Neither of them alpha hemolysis (green colonies) B - The organisms that do this is strep bovis and enterococcus (not staph) C - This is not fungi! Only fungi grow on Sabouraud Dextrose Agar D - This agar is used for gram negative. Pale colonies mean no lactos fermentation which would have been correct for pseudomonas

True pathogens vs. opportunistic pathogens

TRUE -Organisms not ordinarily found in the host -Precipitating factors *Exposure to the organisms in other hosts or environment OPPOTURNISTIC -Organisms ordinarily in contact with the host or in the environment -When these organisms get the opportunity to invade, they take it -Precipitating factors *Mechanical injury *Immunosuppressive disease *Immunosuppressive therapy

So what is the utility of knowing whether a virus encodes/carries its own polymerase and or other enzymes?

Target for antiviral agents - for example, -acyclovir, ganciclovir - targets the viral DNA polymerase -zidovudine - targets the HIV reverse transcriptase -Integrase inhibitor - raltegravir

A 46 yo woman presents with fever, nausea, painful and frequent micturition. Urine and blood samples were collected and sent to the laboratory for analysis. What should the lab do with the urine sample and why?

You can look at it, put it on a slide, do a urinalysis. Then you can take a urine culture and see the type of bacteria because many bacteria can cause UTIs

Chlamydospores

round thick walled spores produced by Candida albicans

Zygomycetes -Bread mold

e.g. Rhizopus , Mucor -The fruiting body is called the sporangium which is filled with sporangiosphores

Zygomycetes (continued)

-In the tissues you would see broad, aseptate hyphae

Ring-form trophozoites of P. falciparum in a thin blood smear

-Multiple rings inside an RBCs

Biofilms

-A special bacterial adaptation that facilitates colonization -Especially seen on surgical appliances such as artificial valves or indwelling catheters -Bacteria in biofilms are bound within a polysaccharide derived from slime that binds the cells together and to the surface -Production of a biofilm requires sufficient numbers of bacteria (quorum) *When Pseudomonas aeruginosa determine that the colony size is large enough (quorum sensing) they produce a biofilm. *Streptococcus mutans causing dental plaque is another example of a biofilm. -The biofilm matrix can also protect the bacteria from host defences and antibiotics. *When bacteria form a biofilm on a prosthetic surface like a catheter, you can give so much abx but the bacteria will still be present. If someone has a prosthetic heart valve that is infected leading to prosthetic valve endocarditis, you can give the pt so many abx but the bacteria is being hid by the layers of biofilm so won't be affected. Sometimes you have to replace the valve

Exotoxins

-AB toxins -Many toxins are dimeric with A and B subunits (A-B toxins). -The B (binding) portion of the A-B toxins binds to a specific cell surface receptor -The A (active) subunit is transferred into the interior of the cell, where it acts to promote cell injury

Adaptive contd...

-APCs - what are the APCs, what do they do? -T cells *CD4 - helper T cells, differentiate into TH1 cells and TH2 cells *CD8 - cytotoxic T cells -TH1 cells direct cytotoxic T cell activity, important in immunity against intracellular bacterial pathogens -Follicular T cells direct antibody production -TH17 cells, produce IL-17, IL-21 and IL-22, fight extracellular bacterial infections

Conidia

-ASEXUAL SPORES -They are found on structures called CONIDIOSPHORES -The shape, colour and arrangement of conidia aid in identification of fungi -Size: *Macroconidia are large sized conidia and *microconidia are smaller

Myocarditis and pericarditis

-Abrupt onset, fever with chest pain, malaise, pericardial friction rub. Pain reduces while leaning forwards. ECG shows characteristic diffuse ST segment elevation. -WHenever there is myocarditis, you won't just get pain, but you can also get changes in the electrical signals of the heart

Acute versus persistent infections

-Acute infections: host immune response leads to complete resolution of disease -Persistent (recurrent infection): incomplete immune resolution -Latent infection: preceded by acute infection in different tissue -Chronic infections -The ability and speed with which a person's immune system controls and resolves a viral infection usually determine whether acute or chronic disease ensues, as well as the severity of the symptoms

Capsule/Slime Layer Clinical significance

-Adhesion to epithelium of the respiratory and urogenital tract *particularly when normal features of the anatomy are unable to flush these out -Slime-producing bacteria (Streptococcus mutans) associated with dental caries; forms biofilm that adheres to teeth -(Pseudomonas aeruginosa) form biofilm and adheres to catheters *This is why many catheters these days are single use -(Staphylococcus epidermidis) adherence to prosthetic devices cause device-related infections

Adaptive immune response

-After neutrophil responses of the innate immune system, TH1-mediated immune responses have been found to be of primary importance in resolution of fungal infection -Small role for antibody mediated immunity *Cryptococcus neoformans, is an example of a fungus against which antibody plays a role in controlling infection. *Otherwise simulation of a TH2 response can result in failure to clear the infection or in some cases, be associated with worsening of the disease!

Viral protein synthesis

-All viruses depend on the host cell ribosomes, tRNA, and mechanisms for posttranslational modification to produce their proteins. -Most but not all viral mRNA have a polyadenosine (polyA) tail, like eukaryotic mRNAs. -The eukaryotic ribosome that binds to mRNA and can make only one continuous protein, and then it falls off the mRNA. -So how do viruses process this large protein into constituent structural proteins? *It is broken down by cellular and viral proteases. *Viral proteases serve as targets for anti viral agents - for example, the protease inhibitors like indinavir inhibit HIV protease -DNA viruses, retroviruses, and most negative-strand RNA viruses transcribe separate mRNA for smaller polyproteins or individual proteins. -The orthomyxovirus and reovirus genomes are segmented, and most of the segments code for single proteins for this reason.

Streptococcus bovis (Strep gallolyticus) You must differentiate this from enterococcus because bovis can be gamma hemolytic as well (no hemolysis) -Bovis will not grow in salt while enterococcus will -Bovis does not do PYR hydrolysis will enterococcus does

-Also called non-enterococcal Group D streptococci. -Constitute fecal flora and are alpha or non hemolytic -Associated with endocarditis in patients with Colon Carcinoma/ polyps. Identification feature: -Test: Growth in bile and cause Esculin hydrolysis -On the bile esculin agar both of them can produce black colored colonies *S. bovis: Positive *Enterococcus: Positive -Test: Growth in 6.5 % salt *S. bovis: Negative *Enterococcus: Positive -Test: PYR hydrolysis *S. bovis: Negative *Enterococcus: Positive

Treatment of Infective Endocarditis caused by Candida

-Amphotericin B: 6-8 weeks *Azoles: fluconazole -Newer drugs: Echinocandins: Caspofungin And-- -Valvular surgery

Chaga's disease: Global importance and prevalence

-An endemic disease of South America. -Caused by Trypansoma cruzi- a flagellate parasite residing in blood stream and tissues. -First described by Carlos Chagas in 1909. More than 300,000 people in the United States are infected with Trypanosoma cruzi, the parasite that causes Chagas disease - and most don't know it

Bacterial Metabolism (continued...) Review fermentation, ETC, glycolysis, etc.

-Bacteria undergo fermentation and respiration to generate energy from carbon sources -Fermentation: direct transfer of proton and electron to organic acceptor *Not efficient in generating ATP -Aerobic Respiration: transport of electrons through chain of carriers to ultimate acceptor which may be oxygen -Anaerobic respiration: inorganic compounds other than oxygen electron acceptor

Bacterial Metabolism

-Bacteria undergo fermentation and respiration to generate energy from carbon sources -Fermentation: direct transfer of proton and electron to organic acceptor *Not efficient in generating ATP -Aerobic Respiration: transport of electrons through chain of carriers to ultimate acceptor which may be oxygen -Anaerobic respiration: inorganic compounds other than oxygen electron acceptor -Based on response to oxygen; whether they can ferment, respire or protect from deleterious effects of oxygen

Bacterial Growth

-Bacterial grow in an ordered process followed by cell division by binary fission -Increase in cell mass and number of ribosomes -Duplication of the bacterial chromosome -Synthesis of new cell wall and plasma membrane -Partitioning of the two chromosomes -Septum formation, and cell division

Abdominal sepsis How do these pathogens get into the peritoneum? There must be a devitilization of the tissue somewhere. It could also be due to a car crash/accident so all these organisms spill out into the peritoneum. Abd surgery, etc. This can lead to sepsis because the organisms can go into the bloodstream from here

-Bacteroides spp and other anaerobes -E.coli -Enterococci

Lab diagnosis of Infective Endocarditis You want to do abx sensitivity test on the cultures after you get the growth, to see what antibiotic the pt can be put on for treatment.

-Blood culture is a procedure where blood is aseptically withdrawn from a suspected case of Infective endocarditis and inoculated into bottles with culture media to determine if microbes are present in the blood. -3 blood samples taken at intervals of 30 mins- 1 hr and added to blood culture bottles -Samples sub-cultured to culture media -For fastidious organisms, special tests like Direct Fluorescence antibody tests/ PCR may be used -Further identification by Grams stain, tests for identification and antibiotic sensitivity test

What is Blood culture?

-Blood culture is a procedure where blood is aseptically withdrawn from a suspected case of sepsis and inoculated into bottles with culture media (containing broth) to determine if microbes are present in the blood. -10-15 ml of blood is withdrawn and added to the broth bottles to increase the chances of growing the bacteria and to dilute out the effect of antibacterial substances like complement in the blood -Further identification by Grams stain, tests for identification and antibiotic sensitivity test -For fastidious organisms, special tests like Direct Fluorescence antibody tests/ PCR may be used -Refer to lab ID of individual organisms

Skin

-Breaks in the skin -Largest target but the most difficult barrier -Outer keratinocytes do not support viral growth -Gain entry when skin is cut through abrasions, cuts, bite of arthropod vector, animal bites or needle prick Mechanical abrasions: -HSV -Papilloma virus -Pox virus Animal bite: -Rabies IV drug abuse: -Hepatitis B -Hepatitis C -HIV Arthropod bites: -Toga virus -Flavi virus -Bunya virus -Reovirus

Stain

-By GIEMSA or WRIGHT STAIN -Look for yeast form with macrophages -Tissue form

On November 25, 2013 at 9:02 am, a hospital notified the Kansas Department of Health and Environment's Infectious Disease Epidemiology and Response section (KDHE) of a possible outbreak of Clostridium difficile among hospitalized patients. Beginning in early November, six patients had been diagnosed with infections caused by Clostridium difficile that were likely acquired during hospitalization. The facility reported that the increase in cases was above their normal baseline rate for hospital-onset C. difficile. Recommend appropriate prevention and control measures for this outbreak, with a focus on measures in sterilization and disinfection.

-C. diff is gram positive, anaerobic with spores. -Sterilization works with spores but there are other things to consider, ie. how would you sterilize a bed or bedside table? These are practical problems but you could do the best you can by using high grade sterilizers like ORTHOPHTHALALDEHYDE OR GLUTARALDEHYDE. You must use a sporeicidal. -How do the spores pass from person to person? Spores do not get up and move, they move via us by not washing hands. So you must implement a vigorous hand-washing program to make sure there is no transmission from one person to the next. -These are two things you can do to break the chain of transmission

Importance of Pseudomonas

-Common organism found in the environment (soil, plants and water) -Often associated with Nosocomial infections as it can colonize: *Hospital environment and surfaces *Ventilators, dialysis units, water tubings *Diluted disinfectants placed in jars, soaps -Very hardy and can develop drug resistance.

Tests for identification of Staphylococci

-Catalase : Enzyme which has the ability to break down H2O2 into water and oxygen. -Positive catalase test: effervescence; Negative: no effervescence -Staphylococci are Catalase positive; Strep are negative -Coagulase test: based on detection of coagulase Enzyme which converts fibrinogen into fibrin. -Incubate S.aureus with rabbit plasma to look for clot formation -S.aureus is coagulase positive while S.epidermidis & S.saprophyticus are negative -Novobiocin disc identification test: Used to differentiate between S.epidermidis and S.saprophyticus -Novobiocin sensitive: Staph epidermidis -Novobiocin resistant: S.saprophyticus

Cell wall construction: formation of peptidoglycan -Abx like BACITRACIN & the enzyme LYSOZYME target B-1,4-glycosidic linkage *By targeting the glycosidic linkage they will be breaking that linkage between the NAGs and the NAMs of a particular layer to compromise the bacterial wall *Lysozme is found in tears

-Cell wall construction mechanisms are targets for antibiotics and antibiotic resistance -When cell divides, cell wall refurbished, new wall added -NAM and NAG held together by β-1,4-glycosidic linkage *Target for BACITRACIN, LYSOZYME -Binding together of peptidoglycan through peptide linkages to form mesh *Catalysed by enzymes, eg. PBP, that have transpeptidase or transglycosylase activity *PBPs are targets for beta-lactam antibiotics -PBPs = Penicillin Binding Protein - catalyze reactions such as transpeptidase and transglycosylase -They are important for the formation of the bacterial cell wall -These proteins got their names because they found that Penicillin specifically targets these enzymes

Bacterial Genetic Material -Housekeeping genes (genes essential for bacterial growth and survival) - found on CHROMOSOME -Genes associated with specialized functions (Ie. virulence genes) - found on PLASMIDS

-Chromosomal, plasmid, bacteriophage DNA -Genes may be organized into operons *Recall lac operon -Plasmid: extra-chromosomal DNA that contains genes non-essential for life *If these are lost, the cell still survives, it just may lose the means of survival in extreme condtions, or virulence *Cannot re-integrate *Autonomous replication - can make more copies of themselves in the absence of bacterial division -Episome—extrachromosal DNA that can integrate into the host chromosome via insertion sequences *Insertion sequences are short sequences of DNA that display homology with some part of the DNA on the host chromosome. Can match up with host chromosome and undergo recombination -Plasmids result from inaccurate excision of episomes from the host chromosome *PLASMIDS CANNOT INTEGRATE INTO HOST CHROMOSOMES, episomes can

Bacterial classification—Naming -Bacteria are usually named by the person who discovered it -GENUS - capitalized -SPECIES - common letter -All of the names are usually italicized in formal writing

-Classified into genera and species according to binomial Linnaeus scheme -Neisseria gonorrhoeae OR Escherichia coli -Genus: Neisseria, Escherichia -Species: gonorrhoeae, coli

Lab diagnosis

-Clinical specimen -Laboratory investigation

Complementation contd...

-Co infection with two mutants (not normal) strains of the same virus -Complementation or continuation of virus production indicates that each mutant was defective in a different gene, so there was one functional copy of each gene in the cell to allow production of all proteins (Fig:2) -Lack of viral production indicates that the two strains are defective in the same gene so one protein is missing (Fig: 1)

Coccidioides immitis -Shown in the tissues in the L bottom image -Has SPHERULES with endospores inside of it -In the enviroment you see the fungi reproducing by arthroconidia. *Remember arhtroconidia develop in the hyphae and when it is time to release into the environment the fragments break apart and the arthroconidia are released

-Coccidioides immitis has HYDROPHOBINS in the wall of the conidia that is antiphagocytic. -Coccidioides immitis produces UREASE which converts urea to ammonia. This ammonia helps the organism to survive within the macrophages. -As the hyphae convert to the spherule (tissue) phase, they also become resistant to phagocytic killing because of their size and surface characteristics.

Bacterial gene expression DNA-dependent - RNA polymerase binds to the PROMOTER REGION (which is one of the regulatory regions in the bacteria)

-DNA-dependent RNA pol binds to promoter complex for gene transcription *Ie. Rifampicin, used for TB, binds DdRp with high affinity. This drug is able to freely diffuse into the cell and binds to the RNA polymerase with high affinity preventing transcription -Virulence genes may be organized into 'pathogenicity islands', each island under control of single promoter *Looking at a plasmid in particular, we may have pathogenicity islands whereby a number of genes performing similar functions are grouped together and are under control of a single promoter. If these are all under the control of a single promoter, it makes it much more effecient when the bacteria wants to express it *Ie. Genes coding for T3SS may be part of a PI -Mutations may be deleterious to bacterium or may confer selective advantage *Eg. Base substitution, frame shift mutation *This mutations can be advantageous or not advantageous to the bacteria

Infective endocarditis is a serious infection characterized by colonization or invasion of the heart valves or the mural endocardium by a microbe This is to just introduce to you that there are a lot of features that could show up in the patient, don't have to memorize.

-Diagnosis of Infective endocarditis is based on the presence of DUKE MAJOR and MINOR criteria Definitive diagnosis: 2 major criteria or 1 major + 3 minor criteria or 5 minor criteria Positive blood culture -> Echocardiographic evidence of Infective endocarditis -> Positive blood culture -> Echocardiographic evidence of Infective endocarditis (contineud...)

Conjugation

-Directional transfer of genetic material from a donor cell to a recipient cell through direct cell-to-cell contact -Donor must have a conjugative or mobilisable element such as a plasmid or transposon *Transposon: Mobile genetic element that contains several genes and can move from one site to another within a genome (within chromosome or onto extra-chromosomal DNA). Includes genes necessary for transposition plus others, eg. antibiotic resistance *Insertion sequence: Simplest form of a transposon; short segment of DNA that can translocate within and between genomes -Sex pilus draws donor and recipient together -Protein in donor cell nicks DNA at oriT site (origin of transfer) to produce a single-strand -DNA unwinds and single strand transferred to recipient, 5'3' -Concurrently, a new complementary replacement strand synthesized -A large F plasmid from a donor bacterium (F+) can integrate into a bacterial chromosome of a F- recipient -See F', Hfr

Important Gram Positive organisms causing toxemia or sometimes Toxic shock

-Disease and Organism: Toxic Shock Syndrome: Staphylococcus aureus -Toxin: Toxic shock syndrome toxin -Toxin and Mode of action: Superantigen -Features: Fever, rash, shock, DIC, capillary leakage -Disease and Organism: Diphtheria: Corynebacterium diphtheria -Toxin: Diphtheria toxin -Toxin and Mode of action: Inhibition of protein synthesis by inhibition of EF-2 -Features: Fever, pseudomembrane formation and systemic effects -Disease and Organism: Tetanus: Clostridium tetani -Toxin: Tetanus toxin -Toxin and Mode of action: Dysinhibition of inhibitory neurotransmitters like Glycine and GABA -Features: Severe muscle constractions and spastic paralysis

Pili/Fimbriae Clinical Significance

-Elicit production of Abs (They are antigenic) -Variants in pili protein: ANTIGENIC VARIATION and PHASE VARIATION for escaping immune recognition *If the host immune system doesn't recognize this particular bacterium, it will take a while for it to generate a fresh response to it -Antigenic variation could -> mutated pilus proteins -Phase variation could -> loss of pili (more drastic) *Say you have pili that has caused an initial infection. Later on, with Phase variation, the bacteria re-presents itself without the pilli. Now the body doesn't recognize that it has seen this before and it takes longer to start up the humoral response -Soluble pilin (S-pilin) of N. gonorrhoeae -> secretion of pili fragments *Pillin is usually attached to the bacterial cell wall but species like N gonorrhoeae can secrete the pillin in the form of S-pillin (or soluble pillin) -Typically, the antibody will bind the pillin on the cell wall and the whole bacteria can be destroyed. However, with soluble pili, they antibodies are aiding the killing of just the pilli and the actual bacterium are still floating free

How can we classify them?

-Endoparasites - parasites which live inside the body (can you think of examples?) *Tapeworm, roundworm, Giardia -Ectoparasites - parasites that live outside the body (can you think of examples?) *Lice, ticks, fleas -Parasites can be broadly classified into *Protozoans (unicellular parasites) *Helminths (multicelular parasites)

Examples of Tissue Tropism

-Epstein-Barr virus (EBV) has a very limited host range and tropism because it binds to the C3d receptor (CR2) expressed on human B cells -The B19 parvovirus binds to globoside (blood group P antigen) expressed on erythroid precursor cells. -The HA of influenza A virus binds to sialic acid expressed on many different cells and has a broad host range and tissue tropism.

Some interesting features of fungi

-Eukaryotes -Have a cell wall -No chlorophyll (this is what makes them different from plants) -Produce filamentous structures called HYPHAE -Produce spores (how they reproduce) *These are different from bacterial spores because one fungus can give rise to hundreds of spores while bacteria can only give rise to one spore -Total number of fungal species ~200,000 *About 300 of these are pathogenic to man !

Recombination

-Exchange of genes between two chromosomes through crossing over within regions of significant homology -Occurs commonly within DNA virus; between 2 strains of the same virus *HSV-1 and HSV-2 co-infecting the same cell resulting in a hybrid virus with genes from both parents *Human immunodeficiency virus (HIV) is a diploid virus: each virion carries two complete RNA genomic strands. Homologous recombination can occur when a cell is coinfected with two different but related strains, giving rise to increasing the probability of the creation of high level and multidrug-resistant strains. *Recombination rare among RNA viruses; Sindbis and eastern equine encephalitis = western equine encephalitis virus

Eyes

-Exposed and unprotected site for viral entry -Flow of tears and eyelid barrier -Viral infections of eye normally follow eye injury, ophthalmic procedure, or swimming in unsanitised pools -Adenoviruses, HSV, Enterovirus 70, rhinovirus

Plasmids

-Extra-chromosomal DNA -Small, circular, covalently closed DNA separate from the chromosome -Carry genes coding for some toxins, antibiotic resistance, pili and enzymes -Multiple plasmids in one bacteria -Transferred to other bacteria through conjugation

How will you recognize a case of infective endocarditis? (Duke criteria) INFECTIVE ENDOCARDITIS = DUKE CRITERIA

-Fever with chills -Arthralgia -Myalgia, fatigue, weight loss -Splinter hemorrhages -Janeway lesions (non-tender lesions) -Osler nodes (tender nodes, found in the thenar and hyperthenar eminences) -Roth spots (flammed shaped hemorrhages with pale center seen with opthalmoscope) -Conjunctival hemorrhages -Anemia -Splenomegaly -Heart murmurs -Echocardiographic findings -Positive blood cultures

Minor Criteria -Added to all these features developing in the pt, you also want to evaluate for any predisposed factors in the history. You want to ask the patient about IV drug abuse, which is one of the most important predisposing factor, especially if there are no pre-existing cardiac condition. -You may also want to ask if they have a history of RHEUMATIC HEART DISEASE RHD- Hx of strep throat/pharyngitis, and following pharyngitis after a few weeks, the pt develops joint pain, fever and invalvement of the heart. RHD "licks the throat and bites the heart". This is caused by Streptococcus pyogenes (beta hemolytic). In the process, there can be immunologically mediated damage to the joints or valves. Valvular damage is the perfect example of MOLECULAR MIMICRY because of resemblance of the bacterial component and components of the body, so the antibodies attack. RHD IS ONE OF THE MAJOR CAUSES OF DAMAGE TO THE VALVE in many countries! Does this also cause infective endocarditis? NO! Because streptococcus pyogenes doesnt go itself to infect the heart, the RHD is immunologically mediated, RHD can cause a NON-INFECTIVE ENDOCARDITIS. However once it damages the valve, then that valve is susceptible to a lot of organisms which can lead to infective endocarditis. *STREP PYOGENES does NOT cause Infective endocarditis. It causes RHD -> Damages the heart (valves) -> Valves become susceptible to different organisms

-Fever, Temp >38 deg C -Predisposition *predisposing heart condition or IV drug abuse -Vascular phenomenon *Splinter hemorrhages *Janeway lesions -Immunological phenomenon *Arthralgia *Osler's nodes *Roth spots -Vascular phenomena are due to septic emboli causing vasculitis -Immunological phenomena are due to deposition of immune complexes

Coxsackie virus: General features

-First described in the Coxsackie town of New York state -ss, RNA, non-enveloped virus -Resistant to heat, detergents, pH:3-9, and acids (as against Rhino virus that is acid labile) *This is why it can remain in the gut -Two types: Coxsackie A (Serotypes 1-24) and Coxsackie B (Serotypes 1-6) *Coxsackie B is associated with the heart conditions

Adherence

-First step in infection -The life cycle of a parasite revolves around species and tissue tropisms, and this determines the organs or tissues of the host in which a parasite can survive. -The attachment of the parasite to host cells or tissue *Nonspecific *mechanical or biting mouthparts *Interaction between parasite adhesins and specific glycoprotein or glycolipid receptors found on some cell types -Specific surface structures that facilitate parasite adhesion include *surface glycoproteins, such as glycophorin A and B *complement receptors *adsorbed components of the complement cascade *Fibronectin *N-acetylglucosamine conjugates

Moving on to the viral capsid

-Forms the outermost coat or shell of the naked viruses (non-enveloped viruses) -Composed of protein subunits -Capsid symmetry: Helical or Icosahedral *The protein subunits of most viruses are assembled to form capsids of helical or icosahedral symmetry

Characteristics of Pseudomonas -Some of them produce capsules -Pseudomonas is known to cause repeated pneumonia in cystic fibrosis, and they produce capsules in this case

-GNB with polar flagella *Polar means they are present at the poles -Aerobic and grows over wide temperature ranges. -Produces pigments: Pyocyanin, fluorescein: Green pigmented colonies seen on nutrient agar. -Non-fermenters: NLF on -MacConkey's agar (medium used for gram negative organisms, has lactose. Lactose fermentation results in pink colonies, no lactose fermentation results in pale colonies) -Has a fruity odor (grape-like) -Oxidase positive -Thin Gram Negative Bacilli -Inflammatory cells with Gram Negative Bacilli

Sterilization (cont)

-Gas -Ethylene oxide (EtO) *Alkylating agent - causes irreversible DNA damage -Hydrogen peroxide gas (oxidizing) -Peracetic acid gas (oxidizing) -Ozone (oxidizing)

Lets begin with the genome

-Genome: DNA or RNA -single stranded (ss) or double stranded (ds) -Linear or circular -Number of segments - some viruses like influenza A have a segmented genome (eight pieces) -Number of copies - some viruses like HIV two copies of the genome (diploid) within the capsid -The concept of genome polarity *Seen in ssRNA genomes *'positive polarity' = mRNA sense => the viral genome can directly undergo translation to begin viral replication *'negative polarity' = anti-sense => the viral genome has to undergo transcription to the mRNA which will be translated

A few terms clarified...

-Genotype - genetic constitution of an organism -Phenotype - observable properties of an organism -Mutation - heritable change in the genotype -Genome - sum of the genes of an organism -Wild-type virus - the original virus from which the mutants are derived and with which they are compared -Primary isolates (field isolates) - fresh isolates from the natural host

Staphylococcus : Morphology

-Gram positive cocci in clusters. -Non-motile and non-sporing -S.aureus bear important surface structures called MSCRAMM: Microbial Surface Components Recognizing Adhesive Matrix Molecules ( Protein A, Clumping factor) *Protein A binds to the Fc part of antibodies preventing the phagocytes from binding to it and engulfing it. It does not get easily phagocytes and evades the immune system -Gram stain of a smear made from pus: -Gram positive cocci in clusters with inflammatory (pus) cells. -Gram stain of a smear made from colonies of Staphylococci showing Gram positive cocci in clusters

Neonatal Sepsis -As they are born, they may carry the maternal flora with them (ie. vaginal flora or a vaginal carriage of some of the dangerous pathogens)

-Group B streptococci (S. agalactiae) -E.coli -Listeria monocytogenes (if pregnant mother eats unpasterized cheese or deli meats that are kept in the cold that perhaps got infected, or even cantaloupes).

Invasion Enzymes

-HYALURONIDASE produced by bacteria like streptococci. staphylococci, and clostridia. How does it work? -COLLAGENASE produced by bacteria like Clostridium histolyticum and Clostridium perfringens. How does it work? -STREPTOKINASE and STAPHYLOKINASE produced by streptococci and staphylococci, respectively. How do they work? Kinase enzymes convert inactive plasminogen to plasmin which digests FIBRIN and prevents clotting of the blood. *This allows the bacteria to spread -LECITHINASESs, produced by bacteria like Clostridium perfringens, destroy lecithin (phosphatidylcholine) in cell membranes.

II. Pili / Fimbriae

-Hair-like fibres made of PILLIN (protein) subunits -Tips of fimbriae: adhesins, lectins attach to sugar residues (mannose) on host cells, an essential pathogenic property *This is a significant virulence factor. For the bacteria that do have this regularly, they may not be able to cause disease if these are absent

Diseases associated with Coxsackie A virus Will not be tested on this

-Hand Foot and mouth disease( A 16) -Herpangina -Aseptic meningitis -Vesicular pharyngitis Common cold All of the above can cause: COXSACKIE A

Sterilization

-Heat -Autoclaving: *Steam under high pressure most efficient method *Steam denatures proteins *Exposure to steam at 121°C at 15 psi for 15-20 minutes -Hot air oven at 170°C for 2 hours *Oxidation and denaturation of proteins *Used for sterilizing articles not penetrated by steam—wax, oil, paraffin, powders, glassware, sharp instruments -Incineration: Complete destruction of biological waste -Flaming: exposure to red heat

Gram negative and fastidious organisms: HACEK group & other rare organisms Fastidious = very choosy, pick growth requirements

-Hemophilus spp -Aggregatibacter (earlier Actinobacillus) spp -Cardiobacterium hominis -Eikenella corrodens -Kingella The above organisms are known as the HACEK GROUP! Others: Coxiella Brucella Bartonella

Spores/Endospores So lets say that a bacterium is put in a highly stressful condition and it has the capibility of making spores. A decision is made because it knows it won't be able to survive these intense conditions so it starts preparing. It starts collecting the minimum that is needed for survival (Ie. genetic material, very little water, very few/basic proteins required for survival). Everything else will disintegrate away. In the CORE of the spore is the bare minimum needed for survival. The idea is that once the adverse conditions are no longer present, the bacterial will sense that it is okay to start living again/dividing again. The spore coat will disintegrate and the basics that are in the core will start working again (transcription/translation/etc) and the bacteria basically rebuilds itself to grow/survive again.

-Highly resistant structures formed during environmental stress *Normal dividing bacterial cells do not form spores, only formed in the time of stress, and not all bacterial species can do this -Survival in adverse conditions of heat, chemicals, cold, desiccation, UV rays, antibiotics and boiling -Destroyed by AUTOCLAVING: 1210C, 15 min, 15 pounds pressure -Seen in two Gram-positive genera: CLOSTRIDIUM and BACILLUS -Dehydration stabilizes proteins and nucleic acid -CALCIUM DIPICOLINATE (part of the core of the spore, once conditions are ideal this will leave and germination will happen); resistance -Sporulated bacteria are dormant return to vegetative forms through germination when environmental stress has subsided

Systemic mycosis

-Histoplasma capsulatum -Blastomyces dermatitidis -Coccidioides immitis -Paracoccidioides brasiliensis -These are dimorphic fungi -Therefore they grow in the yeast form at 37 degrees celcius, and the mole form at 25 degrees celcius in the lab -In the lab, if you change the temp from 25 to 37 you are able to see the change from the mold form to the yeast form

Dimorphic Fungi

-Histoplasma capsulatum** -Blastomyces dermatitidis** -Coccidioides immitis** -Paracoccidioides brasiliensis** -Sporothrix schenckii -Penicillium marneffei -Remember: To prove that a fungus is dimorphic you have to demonstrate interconversion between the mold and yeast forms!

Viral Cytopathogenesis

-Syncytium formation: cell surface glycoprotein expression triggers the fusion of neighboring cells into multinucleated giant cells called syncytia; syncytia allows virus to spread from cell to cell, escape antibody detection and cause the cells to die Examples: -HSV -VZV -Paramyxoviruses -HIV

Stages of infectious diseases

-Incubation period: time between infection and occurrence of the first symptoms or signs of disease *Depends on virulence, infectious dose; immune status of the patient, nature of pathogen and site of infection -Prodrome period: short time of generalised, mild symptoms; malaise and muscle ache that precedes illness -Specific illness period -Recovery period (decline and convalescence) -INFECTION : pathogens overcome the body's external defenses, multiply, and become established in the body; successful invasion is infection -Disease: Classification of infectious disease -ACUTE DISEASE: develops rapidly but lasts a short time -CHRONIC DISEASE: develop slowly, less severe symptoms, continual, recurrent; organism is multiplying; no symptoms; tuberculosis, leprosy -Chronic carrier state: example, typhoid Mary

Bacterial Classification Cell arrangement

-Individual -Diplo- (pairs) -Staphylo- "resembling a bunch of grapes"; clusters -Strepto- "resembling a twisted chain"; chains -Filaments

Incubation period:

-Infections maybe symptomatic or asymptomatic; site of disease manifestation and time for damage determine the incubation period -Very short < 1 week: manifest at site of entry -acute respiratory disease (adeno), common cold(rhino), croup (parainfluenza) and bronchiolitis (RSV) , influenza -Manifest in tissue distant from site of entry -1week to several months; rabies -Slow accumulation of tissue damage: HBV, HIV, EBV

Innate cont..

-Inflammation -Bradykinin, histamine, leukotrienes, serotonin, prostaglandins - increase blood flow and permeability -Fibrin - localising -Lysozyme - what does it do? -Complement -Antibodies (if already immune) -Pyrogens -Neutrophils -Macrophages -NK cells -Role of TLRs

Entry Into the Human Body

-Ingestion -Inhalation -Trauma -Needlestick -Arthropod bite -Sexual transmission

Immunity to fungal infections and evasion

-Innate immune system plays an important role in antifungal immunity -Immunity to opportunistic molds is mediated by the professional phagocytes (neutrophils, macrophages, and dendritic cells), the complement system, and pattern recognition receptors. -Phagocytes have DECTIN-1 a lectin-like structure, that binds glucan (which is one of the important components of the fungal cell wall), and Toll-like receptors (TLR2, TLR4). -Neutrophils and alveolar macrophages are able to kill the conidia of fungi if they reach the tissues. -Candida albicans is able to bind complement components in a way that interferes with phagocytosis.

Entry (Penetration)

-Interactions between multiple VAPs and cellular receptors initiate the internalization of the virus into the cell. -The mechanism of internalization depends on the virion structure and cell type.

Protozoans

-Intestine and lumens - example: Entamoeba histolytica -Blood - example: Trypanosoma cruzi

Conventional Slow infections

-Involve neuronal tissue characterised by a slow increase in viral load over a long period of time and gradual progression once symptoms appear -Progressive multifocal leukoencephalopathy (JCV)

Diagnosis, Treatment, Prevention -You look for antibodies in the serum -You look for antigens in the mucous samples/stool? -No specific antiviral treatment for this, the best thing is monitoring the patient. There isn't even a vaccination

-Isolation of virus from Throat, Stool or CSF (if there is meningities). -Antibody detection assays: ELISA, Neutralization tests -RT-PCR : Reverse Transcriptase PCR for viral RNA detection -No specific treatment. -Prevention: no vaccine available -Best prevented by hand washing/ use of disinfectants like formaldehyde, hypochlorite and chlorine

Envelope

-It is a membrane composed of lipids, proteins, and glycoproteins that surrounds some viral nucleocapsids -It is readily disrupted by drying, acidic conditions, detergents, and solvents such as ether -> this leads to inactivation of the virus. -As a result, enveloped viruses must remain wet and are generally transmitted in fluids, respiratory droplets, blood, and tissue. -Most cannot survive the harsh conditions of the gastrointestinal tract -Non enveloped = naked virus -All helical viruses of animals are enveloped -All of the negative-strand RNA viruses are enveloped -Icosahedral viruses of medical importance may be enveloped or naked.

Characteristics of S. Viridans and Enterococci: Morphology, Growth characteristics and identification tests Optochin is CHIN and near the mouth, inside the mouth strep viridins lives. Viridins is resistant to optochin. Viridins refuses to let a neighbor destroy him.

-Microscopy: GPC in chains THEN -Alpha hemolysis on blood agar or on Chocolate agar *The alpha hemolytic ones will show GREEN on the blood agar because it is causing hemolysis -When you are shown a medium with green color, you may confuse it with pseudomonas. However remember that alpha hemolytics turn green on BLOOD AGARS while pseudomonas turns green on NUTRIENT AGAR and is a diffusible pigment THEN -Catalase test: negative THEN -Not lysed by bile - Bile insoluble -Resistant to Optochin For further differentiation between strep viridens vs strep pneumonia (pneumococcus) you can do the following test: -Bile solubility test - grow the organism and then when you add bile, you find the entire culture becomes clear in the case of pneumococcal because it is biosoluble, but in viridans it is not *S. viridans: Not soluble *Pneumococci: Soluble -Optochin disc test *S. viridans: Resistant *Pneumococci: Sensitive -Mouse pathogenicity (remember both are pathogenic in humans, this is just a test to differentiate) *S. viridans: Not susceptible *Pneumococci: Highly susceptible -Both are alpha hemolytic so you have to differentiate between the two. However, viridans is the only one that can cause infective endocarditis. Most of the time when checking for pneumococcus, sputum comes from the mouth, where viridins is. So you must differentiate!

Reduviid / Kissing bug Can lead to CARDIOMEGALY and MEGACOLON

-Mode of infection: Following bite/ scratch and contamination with fecal contents of the Reduwiid bug -Infective form: Metacyclic trypomastigote forms *When these enter they skin, they can get into the bloodstream -Chagoma and Trypanosomal chancre: site of bite/ entry of parasite -Romana's sign: Unilateral eyelid edema. *Many times if these bugs have defecated on the face, and a kid scratches their eye, they can push the bug into the eye causing conjunctiva, or periorbital sign -Blood stream infection with lymphadenopathy, hepatomegaly and splenomegaly *It can infect phagocytes, nerve cells, muscle cells, etc *If the nerves lose their effect, this is not good! Can lead to mega diseases -Chronic stage involves heart, brain and nerve plexus of organs. Effects: Cardiac arrythmias, Heart Blocks, Mega esophagus , mega colon

Characteristics of Candida SDA - medium used to grow fungi, and here you get whitish/cream colonies

-Morphology -Candida exists as budding yeast cells that can produce pseudo-hyphae and true hyphae -Budding yeast cells of Candida. -Yeast cells with elongated buds called Pseudohyphae -GERM TUBE TEST needed to identify it as candida albicans *Germ tubes are true hyphae formed by Candida albicans when incubated with serum *Here you take human or animal serum, incubate it in the germ tubes, and you see outgrowth of the cell known as candida

Gram Stain Gram + = blue Gram - = pink

-Most bacteria classified into 2 staining groups: *Gram-positive, gram-negative -Based on cell wall composition -Most clinically important bacteria can be detected by this stain Exceptions: -Chlamydia, Rickettsia, (intracellular) *Bacteria that survive and due their damage intracellularly. Cannot do a stain for the inside of the cell -Spirochaetes, (thin and delicate) *Causes syphillis *Has a delicate cell wall -Acid-fast bacteria, (waxy cell wall that stain cannot penetrate) -Mycoplasma (wall-less) *No peptidoglycan so the gram stain that is not ideal for finding this

Respiratory

-Most common route of entry -Through aerosol, saliva -Mechanical barriers: *Cilia, mucus *Innate defense *Local macrophages *Secretory IgA -Rhinovirus -Influenza -Parainfluenza -Corona virus -Arena virus -Hanta virus -Adenovirus -Parvo B19 -Epstein-Barr virus -Varicella-zoster virus -HSV

Viral Glycoproteins

-Most viral glycoproteins can be observed as spikes projecting from the viral surface -Functions of viral glycoproteins *Attachment - Virus associated proteins (VAPs), capable of binding to structures on target cells. Antibodies to these may be protective! *Agglutination of RBCs - termed hemagglutinins (HAs) *Neuraminidase (NA) of orthomyxoviruses (influenza) - help in release of influenza from infected cells *Fc receptor and the C3b receptor associated with herpes simplex virus (HSV) glycoproteins - scavenge complement c3b and immunoglobulins

Natural orifices

-Mouth, nose, respiratory tract, ears, eyes, urogenital tract, and anus -How are they protected? -However, many bacteria are unaffected or have the means to evade these defences. *Helicobacter pylori can survive in the acid environment of the stomach *The outer membrane of the gram-negative bacteria makes these bacteria more resistant to lysozyme, acid, and bile. Consequence? The members of the Enterobacteriaceae family can thus colonise the gastrointestinal tract. Any breach in the mucosa can lead to bacteraemia with these organisms

Type III, Type IV Secretion Systems

-Multi-protein complexes that span bacterial cell envelope and extend to cross membrane of host cell *Can be found in both gram positive and gram negative bacteria -T3SS identified in Gram-negative bacteria *Molecular syringe that can inject macromolecules into host cell usually to subvert the host cell so the bacteria can take over without being killed -T4SS found in G -ve and +ve bacteria *Pathogenic bacteria inject macromolecules into host cells *Can transfer DNA between bacteria by CONJUGATION -Injected macromolecules can facilitate uptake and invasion, promote intracellular survival and replication of the bacteria, or the apoptotic death of the host cell *Some bacteria want to enter the host cell (or have to enter) to do their thing, the T3SS and T4SS help with this. -Major virulence factor for some bacteria

Animalia

-Multicellular parasites (think of the worms!) HELMINTHS (worms): 1. Cylindrical worms (Phylum Nemathelminthes, Class: Nematoda) 2. Flat worms (Phylum Platyhelminthes) -Tapeworms or cestodes (Class Cestoda) -Flukes or trematodes (Class Trematoda)

Molds

-Multicellular, filamentous (unlike yeast which are unicellular) -Mycelium - body of the fungus - mass of filaments or hyphae -What are rhizoids? -hypha(e) - are the filaments of the fungus *Septate - having partitions *Aseptate (Coenocytic) (like a tube containing nuclei)

Faeco-oral

-Must survive acid pH of stomach, alkaline pH of intestine, digestive enzymes, bile detergents -Mucosal IgA, local macrophages, lymphocytes -Infecting viruses are mostly naked (non-enveloped) -Enveloped do not survive acid and bile detergents -Enteroviruses -Hepatitis A -Rotaviruses -Caliciviruses -Astroviruses -adenoviruses

Primary pathogens

-What are they? -Those microorganisms that can cause disease in an otherwise normal or healthy individual. -Example - Shigella causing shigellosis, Vibrio cholera causing cholera

Normal flora - Clinical Implications? -When a pt is on broad spectrum abx for a long period of time, they may end up with C-diff which is an anaerobe in the gut which may result in pseudomembranous colitis -Small numbers of C. diff are found in the normal flora however with the abx killing off everyone else, allowing these cdiff to multiply unapposed in the gut

-Pseudomembranous colitis caused by Clostridium difficle -Altered normal flora giving rise to inflammatory bowel disease -Can normal flora cause infectious disease? YES! 1. When in the normal flora are moved to a site in which they are not normally found -Ie. On the skin, there is many normal flora however if there is a cut, these normal flora can get on the deep layers of the tissues and cause infection 2. ???

Gram Negative Bacilli

-Pseudomonas -E. coli -Klebsiella

Burn infections

-Pseudomonas aeruginosa -Staph aureus -Staph epidermidis -Many of these organisms came from the endogenous flora of the person! -Organisms like Pseudomonas can grow in disinfectants, soaps, lurks in the hospital and can cause infection. If you dilute a disinfectant you have to use it right away in the hospital or pseudomonas has the ability to grow in. Sometimes in the hospital you can see bed sheets turn green because of the growth of pseudomonas

Nosocomial pneumonia

-Pseudomonas spp -Acinetobacter -E.coli -Enterococcus

Sterilization (cont..)

-Radiation -Ionizing *Gamma or x-rays used to sterilize heat labile articles like plastic syringes, implants, gloves, pharmaceuticals *Causes irreversible DNA damage

Reassortment

-Random reassortment of gene segments is seen in RNA viruses with segmented genomes e.g., influenza A virus, reoviruses -Which viruses have segmented genomes? (BOAR?) -When a cell is co-infected with 2 different viral strains, gene segments are exchanged and progeny viruses are produced with RNA segments from either parenteral virus -Is responsible for antigenic shift (you should also know what is antigenic drift and what causes it!)

Host range

-Range of cells (or species) that can become a host to a virus or bacteriophage; dictated by the presence or absence of specific receptors that enable active viral infection. -A virus that binds only to a specific receptor may only be able to infect a specific number of species

Entry - Non-enveloped viruses

-Receptor mediated endocytosis (Viropexis): Virus surrounded by plasma membrane and enclosed in a cytoplasmic vesicle (endosomal vesicle) -Low pH in the endosome leads to change in conformation of viral spikes and release of nucleocapsid

More Terms

-Reinfection: Secondary infection that occurs upon recovery from a previous infection and is caused by the same agent. -Coinfection: Concurrent infection of a host (single cell/tissue) by two or more viruses. -Superinfection: Process by which a host previously infected by one virus acquires coinfection with another virus at a later point in time.

Where do we get infection from?

-Reservoirs - sites where pathogens are maintained as source of infection -Animal reservoirs *Zoonotic diseases: diseases that spread naturally from their usual animal hosts **Anthrax, bubonic plague **Direct contact with animals, animal products, through insect vectors -Human carriers *Humans with active disease, convalescing patients important reservoirs *Chronic carrier *Http://www.Pbs.Org/wgbh/nova/typhoid/mary.Html -Non living reservoirs -Soil : tetanus spores -Water: contaminated with feces, urine of humans and animals -Food: meat, vegetables, milk

Ring-form trophozoites of P. vivax in a thin blood smear one of the causative reagents of malaria

-Ring trophozoites inside the RBCs -The nucleus of the trophozoite acts as a dot and the rest of it is just a circumferential ring inside of the RBC

III. Flagella -Much longer than pilli

-Rope-like propellers, help bacteria move from one location to another -Composed of protein FLAGELLIN *Antigenic. Therefore now used for serotyping bacteria (different serotype differentiates those of the same species, ie. diff E. Colis) *H antigen *Ecoli O147:H7 lets us know that the ecoli have an O antigen of that strain and flagellum! -PERITRICHOUS FLAGELLA: Entire cell is surrounded by flagella -MONOTRICHOUS: One flagellum located at one end of the cell -LOPHOTRICHOUS: Two or more flagella attached to one end of the cell -Chemotaxis

Stages of infectious disease (cont)

-SUBACUTE DISEASE: between acute and chronic -LATENT DISEASE: pathogen is not multiplying -COMMUNICABLE DISEASE: from another infected host -NON COMMUNICABLE DISEASE: originate endogenously or from environment; tetanus -CONTAGIOUS DISEASE: easily communicable -Inapparent/subclinical -Local infection: infection confined to a small region of the body -Systemic infection: widespread infection, multisystem; involves blood and lymph -Focal infection: infection that serves as a source of pathogens for infection at other sites of the body -Primary infection: initial infection within a given patient -Secondary infection: infections that follow primary infection; often by opportunistic pathogens -Nosocomial infection *exogenous *endogenous *iatrogenic infections

Growth characteristics and identification tests

-Saboraud's dextrose agar: creamy / pasty colonies -Germ tube test: positive seen in C.albicans: -Incubation of Candida with human/ animal serum at 37deg C for 2 hrs results in germ tube formation

What is the Lab diagnosis for staph?

-Sample collection (blood culture, 15 ml, which is added to blood culture broth and incubated) -You now have an unknown organism THEN -Culture (subculture) and microscopy -Blood agar shows beta hemolytic colonies -(Another media that will support the growth of staph is mannitol salt agar where you will see yellow colonies, usually used for samples like food, vomitus, etc.) -You then do a CATALASE test which will be positive if staph -To further differentiate you do a COAGULASE test which will be positive if it is staph aureus! If negative, it could be epidirmidis or saprophyticus. -You then do a NOVOBIOCIN TEST to differentiate the two. Epirdimidis is sensitive, resistant is saprophyticus THEN Antibiotic susceptibility tests

Laboratory diagnosis

-Sample: What sample is tested? Blood culture -Culture- What are the likely findings? Green pigmented colonies on nutrient agar. Pale colonies on mackonkies agar (because they are non-lactose fermenters). -Identification tests: How is the organism identified? Oxidase test - if positive, it is pseudomonas You will also be able to smell these!

Endotoxin (lipopolysaccharide)

-Seen in the outer leaflet of the outer membrane -The molecular patterns in these structures (pathogen-associated molecular patterns [PAMPs]) bind to Toll-like receptors (TLRs) and other molecules and stimulate the production of cytokines -The lipid A portion of lipopolysaccharide (LPS) produced by gram-negative bacteria is a powerful activator of acute-phase and inflammatory reactions -Endotoxin is released on lysis of gram-negative bacteria. -Endotoxin binds to specific receptors (CD14 and TLR4) on macrophages, B cells, and other cells and stimulates the production and release of acute-phase cytokines, such as IL-1, TNF-α, IL-6, and prostaglandins. -Endotoxin also stimulates the growth (mitogenic) of B cells. -Weaker, endotoxin-like responses may occur to gram-positive bacterial structures, including teichoic and lipoteichoic acids.

Normal flora

-Seen on various body sites like the skin, GIT, etc. -How do they help us? *Digestion *Vitamin K production *Protect host from colonization with pathogens (how?) **Normal flora compete for nutrition with pathogens and they compete for receptors *Activate host immune response **Evidence that shows that the presence of different kind of normal flora may be responsible for many of the autoimmune diseases we see -What factors can disrupt normal flora? *Antibiotics *Changes in diet **Vegan normal flora is different from meat eater normal flora *Stress *Host immune response

Toxic Shock Syndrome -One of the most important causes is S. aureus, usually seen in women keeping tampons in for a long time. Also seen in any other type of packing (like nasal packing) that is kept in for a while) -Another one is S. pyogenes!

-Severe headache, sore throat, fever as high as 105°F (40.6°C), -rashes that spread from the face to the rest of the body, palms and soles and undergo desquamation; dehydration and watery diarrhea leading to shock and death. Predisposing factors: -Infected tampons. -Infected dressings/ surgical packing/ sutures.

Urogenital Tract

-Sexually transmitted viruses -Barriers are *stratified epithelium of the vagina *Acid pH *IgA -Enter mucosal epithelium through tears and abrasions -HSV, HPV, Blood borne viruses, HIV, Hepatitis B, Molluscum contagiosum

Invasion example

-Shigella -https://www.youtube.com/watch?v=CtsQ6lZ3RI4 -Listeria -https://www.youtube.com/watch?v=xaZ0BN-f_fs

Pneumocystis jiroveci

-Silver staining depicting cysts -The black cysts are what is stained

Simple stain

-Simple stain just colors the specimen quickly, and it can be seen what type of microbe we're working with (virus, bacteria, etc). A simple stain from a persons specimen/swab can help identify the microbe -Just one stain used here -It simply allows you to visualize bacteria, DOES NOT CLASSIFY IT AS GRAM NEG OR GRAM POS

Introduction - How are viruses different from other microorganisms

-Small size 10 to 300 nm. In contrast bacteria are 1000 nm and erythrocytes 7500 nm in diameter. -Genome may contain either DNA or RNA, viruses contain only one type of nucleic acid. -Metabolically inert - Viruses have no metabolic activity outside susceptible host cells.

What are Bacteria? -Viruses are small as well but viruses are not independently living

-Smallest, most versatile independently living cells known -Visible only with the aid of microscope -Prokaryotes; no membrane bound organelles

Invasion

-Some bacteria can cross mucosal membranes and other tissue barriers to enter normally sterile sites and more susceptible tissue -These invasive bacteria either destroy the barrier or penetrate into the cells of the barrier. *Enzymes *Other mechanisms

Invasion

-Some fungi are introduced through mechanical breaks in the skin. *For example, Sporothrix schenckii infection typically follows a thorn prick or some other obvious trauma. -Fungi that initially infect the lung must produce conidia small enough to be inhaled past the upper airway defences. *For example, arthroconidia of Coccidioides immitis (2 to 6 μm) can remain suspended in air for a considerable time and can reach the terminal bronchioles to initiate pulmonary coccidioidomycosis. -Invasion directly across mucosal barriers by the endogenous yeast Candida albicans is similarly associated with a morphologic change, the formation of hyphae. -Extracellular enzymes (eg, proteases, elastases) are associated with the advancing edge of the hyphal form of Candida and with the invasive forms of many of the dimorphic and other pathogenic fungi.

Other ways to enter a cell?

-Some herpesviruses and retroviruses fuse with cells at a neutral pH and induce syncytia after replication. -Why should we learn about virus entry? *If we can block virus entry, we can prevent the infection! *Maraviroc - a CCR5 receptor antagonist

Exposure and entry Ingestion Direct penetration Transplancental penetration Organims -directed penetration

-Source typically exogenous -Most common modes of entry *Ingestion *direct penetration through the skin or other surfaces -Transmission of parasitic diseases is frequently facilitated by environmental contamination with human and animal waste -Bites of arthropod vectors -Inoculum size - a person may get malaria from the bite of a single mosquito, however, may require large inocula to produce intestinal amoebiasis

Pathogenesis of Infective Endocarditis

-Source: S.aureus on the skin/ ant. nares of a patient or health care personnel (could also be the person himself pushing in IV drugs) -Localized infection leads to a bloodstream infection -Attachment, colonization, Production of enzymes and toxins -Infective endocarditis: large vegetations on Mitral valve.

Lab diagnosis; Treatment; Prevention

-Spindle shaped, nucleated, curved, flagellate forms called Trypomastigotes in blood films/ CSF. -Xenodiagnosis: feeding lab bred Reduwiid bugs with patient's blood and demonstration of the parasite in the gut of the bug, 4 weeks later. (this is done if the bug count is few - not popular) -ELISA, PCR. -Treatment: Nifurtimox. -Prevention: Protection against bite/ scratch of a Reduviid bug.

Endospores

-Spores are formed within the cell terminally, sub-terminally or centrally and sometimes cause the cell to swell -Clinical challenge as these are highly resistant and infective forms -Botulinum spores survive canning; tetanus spores survive disinfection and boiling (dont have to memorize right now) -Clostridium difficile spores (psedomembranous colitis) survive disinfection by hand sterilizers (dont have to memorize right now) -B. anthracis spores used in biological warfare (don't have to memorize right now)

Growth characteristics

-Staphylococci are non -fastidious bacteria and can grow on several media -Nutrient agar: small, opaque colonies with golden yellow pigment. -Blood agar: Golden yellow colonies with β type of hemolysis, with clear zone around colonies. -Mannitol salt agar: form yellow colonies due to fermentation of mannitol sugar and growth in high salt concentration. *Mannitol usually used to find organisms that cause FOOD POISONING -S. epidermidis forms whitish, opaque colonies -S. epidermidis does not ferment Mannitol, it makes pink colonies -What type of a medium is MSA? What is its use?

Importance of Staphylococci

-Staphylococcus are ubiquitous and part of the normal flora. -Colonize human skin: anterior nares, axilla, perineum etc. Cause wide spectrum of infections including localized and systemic infections. -Notoriously associated with opportunistic and Nosocomial infections. -Associated with drug resistant strains such as Penicillin resistant Staph, MRSA, VISA/VRSA

Name some organisms that are associated with Infective Endocarditis?

-Staphylococcus aureus (however the coagulase negative staph can also cause this) -Pseudomonas -Streptococcus -Candida -Viridans group

Walling off infection

-Staphylococcus aureus can also escape host defences by walling off the site of infection. -It can produce coagulase (an enzyme that promotes the conversion of fibrinogen to fibrin to produce a clot-like barrier) -Pus formation upon the death of neutrophils limits antibody or antibiotic access to the bacteria. -Mycobacterium tuberculosis is able to survive in a host by promoting the development of a granuloma, within which viable bacteria may reside for the life of the infected person.

Pathogenic actions of bacteria (cont.)

-Staphylococcus aureus makes an IgG-binding protein, protein A, which prevents antibody from activating complement or being an opsonin and masks the bacteria from detection. -Bacteria evade complement action by preventing access of the components to the membrane, masking themselves, and by inhibiting activation of the cascade. -The thick peptidoglycan of gram-positive bacteria and the long O antigen of LPS of most gram-negative bacteria prevent the complement from gaining access -Streptococcus pyogenes degrades the C5a component of complement thereby limiting the chemotaxis of leukocytes to the site of infection -To compensate for the lack of O antigen, Neisseria gonorrhoeae attaches sialic acid to its lipooligosaccharide (LOS) to inhibit complement activation.

Structures outside the bacterial cell wall—I. Glycocalyx/Capsules/Slime

-Sticky substance produced by the cell and attached to the cell wall: GLYCOCALYX -When glycocalyx highly organised and firmly adherent: CAPSULE -When glycocalyx loosely attached: SLIME LAYER Made of polysaccharide, except in Bacillus anthracis (glutamic acid) *The Bacillus anthracis capsule is made up of glutamic acid K antigen (refers to the CAPSUE LAYER) found in in E. coli (and there are other bacteria) *The K antigen automatically tells you that that bacterium forms a capsule

Bacterial Superinfection

-Superimposed over existing viral infection -Example: pneumococcal pneumonia and staphylococcal pneumonia in a patient suffering form influenza -NK cell response may be impaired -Staphylococcal protease cleaves viral haemagglutininin, a prerequisite for viral infectivity -Viral neuraminidase facilitates the attachment of Pneumococcus to the respiratory epithelium

Virulence Factors: S.aureus

-Surface components *Protein A - binds Fc part of immunoglobulins evading the immune system because they escape from phagocytosis -Enzymes -Toxins

Cutaneous mycoses

-The cutaneous mycoses are fungal infections of the skin, hair or nails. -No living tissue is invaded, however a variety of pathological changes occur in the host because of the presence of the infectious agent and its metabolic products. *Fungus produces metabolites and these metabolites irritate the host and cause itching -These infections are caused by the DERMATOPHYTES (Ie. ring worm)

Dermatophytes TME

-The disease is called dermatophytoses or tinea 1. Trichophyton - infects skin, hair, nails 2. Microsporum - infects skin, hair 3. Epidermophyton - infects skin, nails

Adhesion

-The first step - adhesion -Requires a receptor on the host cell and a ligand on the bacterial cell (IT TAKES TWO TO TANGLE) -Adhesion is mediated by bacterial ligands called ADHESINS -Consequences? *Tissue tropism *Species specificity

Recognition of and Attachment -> to the Target Cell

-The first step in infection of a cell -The viral attachment protein on the capsid or envelope recognizes specific receptors, which may be protein, carbohydrate or lipid, on the outside of the cell. -Cells without the appropriate receptors are not susceptible to the virus (tissue tropism). -Receptors on host cells maybe proteins glycoproteins or glycolipids which are usual signaling devices or immune system components -Some viruses require interaction with more than one ligand on the cell surface - we call them co-receptors -Antibodies to viral attachment protein may be neutralizing!

Opportunistic mycoses -Seize the opporutinity to invade when the immune system is low

-The opportunistic mycoses are infections due to fungi with low inherent virulence. -The etiologic agents are organisms which are common in all environments. -Examples *Rhizopus *Mucor *Aspergillus

The fungal cell wall

-The plasma membrane is followed by three layers of cell wall material -From inside out these are: 1. a CHITIN LAYER (polymer consisting mainly of unbranched chains of N-acetyl-D-glucosamine) 2. a layer of β-1,3-GLUCAN 3. a layer of MANNOPROTEINS (mannose-containing glycoproteins) which are heavily glycosylated at the outside of the cell

Superficial mycoses

-The superficial mycoses are fungal diseases that are confined to the dead OUTER LAYERS of the skin, nail, or hair, (keratinized layers) rarely invading the deeper tissue or viscera. -Patients usually presenting due to cosmetic reasons. E.g. Malessizia furfur infection

Systemic mycoses

-The systemic mycoses may involve deep viscera and become widely disseminated. -Each fungus type has its own predilection for various organs which will be described as we discuss the individual diseases.

Histoplasma Capsulatum -Has the tissue form that is seen in the human body where the macrophages are filled with the yeast forms (pink image) -In the environmental form, you can see tubecrulate (round asexual spores/conidia) (blue image)

-The tissue yeast form of Histoplasma capsulatum multiplies within macrophages by interfering with lysosomal killing mechanisms. *Modulation of the pH of the Phagolysosome to pH range > 6-6.5 *Iron and calcium uptake *Presence of 1,3-alpha-glucan in the cell wall (also a virulence factor for Blastomyces dermatitidis, Paracoccidioides brasiliensis)

List everything you know about the Coxsackie virus

-There is A & B -B is the one that causes pericarditis and myocarditis -The pt may have arrhytmias/disruptions in EKG, chest pain -The pt got the virus by feco-oral/inhalation transmission but finally by ingestion -It is an SS RNA NON-ENVELOPED VIRUS -It resits acids, detergents, has wide temp variation -Can be detected in stool (by looking for antigens), or in the serum (by looking for antibodies) -If you want to detect it quickly you would do RT-PCR

Taenia saginata

-There is taenia solium and Taenia saginata. -Taenia solium is the PORK tapeworm -Taenia saginata is the BEEF tapeworm -Solium has armed (with whiskers/hooks on the head) -Saginata does not have hooks!

Taenia solium

-There is taenia solium and Taenia saginata. -Taenia solium is the PORK tapeworm -Taenia saginata is the BEEF tapeworm -Solium has armed (with whiskers/hooks on the head) -Saginata does not have hooks!

RNA tumor viruses:

-These are retroviruses that encode oncogene proteins similar to c-oncs (cellular proteins involved in cellular growth control) -This results in the expression of host growth promoting genes. -Human T cell lymphotropic virus *Encodes TAX that transactivates gene expression including those for growth stimulating cytokines *These leukemias develop slowly, 20 to 30 years after infection -Indirect oncogenesis? -Hepatitis B virus and Hepatitis C virus - persistent infection, inflammation and tissue repair, continuous stimulation of liver cell growth may lead to tumor formation -Human Herpes Virus 8, causing Kaposi sarcoma, encodes for growth promoting cytokines.

The capsule

-These slime layers function by shielding the bacteria from immune and phagocytic responses. -Capsules are typically made of polysaccharides, which are poor immunogens. -The Streptococcus pyogenes capsule, is made up of hyaluronic acid, which mimics human connective tissue, thereby masking the bacteria -The capsule also acts like a slimy football jersey, in that it is hard to grasp and tears away when grabbed by a phagocyte. -The capsule also protects a bacterium from destruction within the phagolysosome of a macrophage or leukocyte. (anti-phagocytic)

Look at the image and tell me everything you know about it?

-This is Pseudomonas -Gram - -Fruity/tortilla like odor -Green pigment -AEROBIC -Oxidase positive -Exotoxin A that works by inhibition of protein synthesis *Resembles C. diphtheria toxin -Can grow pale on MacConkey's agar (as it cannot ferment lactose)

How does the heart get infected? Are all vegetations infective? No! They are called marantic vegetations. We will mostly discuss the infective vegitations

-Transient bacteremia/ secondary bacteremia ENDOCARDITIS -Inflammation of the inner lining / endocardium -This is USUALLY THE VALVES! When there is endocarditits it doesn't only affect the native valves but it can also affect PROSTHETIC valves! Prosthetic valves have slightly different organisms associated with them compared to native vilves, though there is a considerable amount of overlap MYOCARDITIS -Inflammation of the myocardium PERICARDITIS -Inflammation of the outer covering of the hear -One of the complications of pericardial fluid accumulating in large amounts is called CARDIAC TAMPONADE therefore the heart can't beat well because there is so much pressure on it

Trypanosoma

-Trypanosoma cruzi trypomastigote in a thin blood smear stained with Giemsa. Note the more anterior location of the nucleus. -The trypanosoma is just outside of the RBC

Toxins

-Two types - exotoxins and endotoxins -Exotoxins (think exo- , outside, so secreted out by the organism) *Cytotoxin *Enterotoxin *Neurotoxin -Exotoxins may be secreted while in the environment outside the human body (preformed toxins) before the person is exposed to them or may be produced within the human body after the organism has gained entry -Endotoxins (think endo- , within, so part of the cell wall of the organism)

Yeasts

-Unicellular fungi -Divide by budding -Some of the buds elongate to form PSEUDOHYPHAE (not true hyphae) which resemble hyphae (pic of pseudohyphae)

Candida -Often described as a yeast like fungus -Reproduces by budding and some of the budding elongates by forming pseudohyphae -Candida produces both hyphae and pseudohyphae

-Unicellular fungi -Yeast like fungus -Divide by budding -Some of the buds elongate to form pseudohyphae (not true hyphae) which resemble hyphae

Plasma Membrane

-Uptake of substrates from small, medium molecular weight compounds and degraded molecules in periplasm -Facilitated diffusion; passive through carrier proteins, permeases -Group translocation - uses phosphoenolpyruvate for energy -Active transport -Uptake of iron: siderophores (chelators) *Bacteria need iron for survival. There may be a lot of iron floating around but a lot of it may not be bioavailable (body not able to uptake that type of iron) *Bacteria secrete or exploit from others siderophores - these kelate/bind to iron with very high affinity. The siderophores are taken up by the bacteria and the iron is released and the bacteria can use this iron

Based on the onset of illness and the severity: ACUTE You are not required to make a diagnosis of whether it is acute or not acute, but this just helps you understand better

-Usually infection of a previously normal heart valve -Usually caused by organisms with higher virulence (staph aureus, pseudomonas), lesions are very destructive -Infection is fulminant with high fever and toxicity, associated with higher mortality -Treatment may need surgery and antibiotics

Based on the onset of illness and the severity: SUBACUTE You are not required to make a diagnosis of whether it is acute or not acute, but this just helps you understand better

-Usually infection of deformed valves -Caused by organisms of lower virulence (strep viridans, strep bovis), lesions are less destructive -Insidious onset, protracted course with low grade fever, night sweats, weight loss, fatigue and better prognosis -Antibiotics often suffice

IV Drug abuse prostheti

-Usually the Tricuspid valve is involved in IVDA cases. -In most other I.E cases, order of involvement is : Mitral valve > Aortic valve > Tricuspid Valve> Pulmonary Valve 1. Staphylococcus aureus(commonest) -S.epidermidis -Gram Positive Cocci in clusters -Infective endocarditis: large vegetations on valves. 2. Pseudomonas aeruginosa -Gram Negative Bacilli 3. Candida albicans Aspergillus -Fungi

A few terms

-Viral pathogenesis - An interaction interaction of viral and host factors that leads to disease production -A virus is considered pathogenic for a specific host if it can infect and cause disease-specific symptoms in that host. -Several key properties determine viral infectivity, the cells/tissues affected, and the outcome of an infection. -Virulence - Ability of the virus to cause disease by avoiding or otherwise overcoming the host's defense mechanisms -Cellular tropism --The specificity of a virus for a given host cell or host tissue, determined by *VAP *Cell surface receptors *Other cellular proteins: cellular transcription factors --Infection by a certain virus tends to affect a specific organ or group of organs (target organs), causing classic symptoms for that particular virus

Direct fusion

-Viral surface protein spikes interact with surface receptor and results in fusion of the viral envelope with the host cell membrane -E.g. HIV, paramyxoviridae

Virus

-Viruses are obligate intracellular parasites *depend on the biochemical machinery of the host cell for replication -Why study viral structure? *Knowledge of the structural (size and morphology) and genetic (type and structure of nucleic acid) features of a virus provides insight into how the virus replicates, spreads, and causes disease.

A few general facts.... (cont...)

-What about the DNA viruses? -Most DNA viruses replicate in the nucleus except Poxviruses *Most DNA viruses use the cell's DNA-dependent RNA polymerase II and other enzymes to make mRNA. *Poxviruses replicate in the cytoplasm and therefore must encode enzymes for replication and to provide deoxyribonucleotides. *Parvovirus requires cells undergoing DNA synthesis to replicate. -Papovavirus stimulates cell growth and DNA synthesis. -Hepadnavirus stimulates cell growth, encodes reverse transcriptase -Adenovirus stimulates cellular DNA synthesis and encodes its own polymerase. -Herpesvirus stimulates cell growth, encodes its own polymerase and enzymes to provide deoxyribonucleotides

Laboratory diagnosis: Specific investigations Why do you need to take 3 blood samples? You want to recover bacteria from the blood stream! When will they go into the bloodstream? You don't know, because they are sitting on the valves and break off when they want, and they are shed into the bloodstream intermittently. This is called INTERMITTENT BACTEREMIA. Therefore you take multiple samples! -After how many hours do you think you'll be able to see the colonies on the plate? 24-48 hours! However, with the HACEK group of organisms will not show any signs of growth in the first 48 hours because they are fastidious. They will start showing up much later! The normal time of holding of the cultures is 48 hours then you discard with report. In blood cultures, you need to incubate the cultures for a longer period of time especially if you are suspecting the HACEK group

-What procedure is carried out for diagnosis? Blood culture -Why is the blood culture carried out for at least two to three times over a 24 hrs period? To document bacteremia which may be intermittent and rule out contamination with normal skin flora Why are large volumes of blood tested? In order to improve the chances of recovery of the bacteria from blood as the number of bacteria is quite low in blood When is a blood culture considered as negative? When the blood has been incubated for a week and shows no signs of growth. If HACEK suspected, retain till 4 weeks.

Reassortment contd...

-When a cell is co-infected with human and avian influenza virus e.g., where farmers live in close contact with their animals, co-infection of pig cells with avian and human influenza virus occurs -If a virus is assembled with the neuraminidase gene segment of one virus and the haemagglutinin of another, the gene assortment results in a dramatically different strain. -This is responsible for the major antigenic changes within the virus termed antigenic shift and lead to pandemics of influenza (spanish flu of 1918 to the latest one H1N1 swine flu in 2009-2010) -http://www.flu.gov/pandemic/history/

Complementation

-When either one or both the virus co-infecting a cell has a mutation that results in a non functional protein, the non mutated virus complements the mutated one by making the functional protein that serves both the viruses -The viral protein (gene product) provide the missing or defective function, but the genotype of the parental and progeny virus are unchanged -e.g., hepatis B virus (helper virus) providing its surface antigen to hepatis D virus (defective virus) which is defective in its ability to produce its own outer protein

lac Operon regulation -The lac operon is an example of an INDUCIBLE OPERON -If lactose is present and other sugars (like glucose) is absent, then there will be induction of transcription of genes on the lac operon. -If glucose and lactose is present, then another process will be turned on -If lactose is the only sugar that is there, then there will be transcription of these genes taking place -Humans have bacteria that are normal flora within the gut that have the lac operon. In the case where there is a meal with no glucose present (ie. someone with celiac who cannot consume regular carbs), this is one example where the normal flora bacteria will help us digest sugars

-When we talk about regulation of gene expression there are other regulatory sequences, such as the OPERATOR, and regulatory proteins, such as a REPRESSOR -In the absence of lactose, the lac repressor binds the operator, and transcription is blocked -In the presence of lactose, the lac repressor is released form the operator, and transcription proceeds at a slow rate

Parasite Pathogenicity

-Wide range of parasites, lots of diversity -Equivalent diversity in pathogenic mechanisms -Infective dose and exposure -Penetration of anatomic barriers -Attachment -Replication -Cell and tissue damage -Disruption, evasion, and inactivation of host defenses

Biological gradient

-Wide range of qualitative responses when a host is exposed to an agent -What determines this? *Virulence of the organism *Age of host *Genetic makeup of the host *Immune response *Portal of entry *Etc. (see next slide)

Classified into two groups based on their appearance in the laboratory

-Yeasts -Produce moist creamy or pasty colonies on culture (Looks like toothpaste on the culture) -Molds -Produce cottony, fluffy, powdery, or woolly colonies on culture (looks like wiffs of cotton on the plate)

Sterilization KILLS SPORES!

-a process that destroys or eliminates all forms of microbial life including spores and is carried out in health-care facilities by physical or chemical methods -Examples: Steam under pressure, dry heat, EtO (ethylene oxide) gas, hydrogen peroxide gas plasma, and liquid chemicals

Disinfection Does NOT kill spores!

-a process that eliminates many or all pathogenic microorganisms, except bacterial spores, on inanimate objects -Examples: liquid chemicals or wet pasteurization

Superantigens

-activate T cells by binding simultaneously to a T-cell receptor and a MHC class II molecule on an antigen-presenting cell without requiring antigen. -activate large numbers of T cells to release large amounts of interleukins (cytokine storm), including IL-1, TNF, and IL-2, causing life-threatening autoimmune-like responses. -Also leads to death of the activated T cells, resulting in the loss of specific T-cell clones and the loss of their immune responses -Examples of superantigens *toxic shock syndrome toxin of Staphylococcus aureus, *staphylococcal enterotoxins *erythrogenic toxin A or C of Streptococcus pyogenes

Blastospores

-buds of yeasts -If these fail to detach and become elongated -> pseudohyphae

A 10 yo boy is brought in to the ER by his concerned parents. They tell you that he was bitten by a fox while they were out camping in the woods, and it has taken them six hours to get to the hospital. On examination, bite marks of the animal are visible on the lower limb of the child. You are concerned about infection with a negative sense RNA virus. 1. Which of the following regarding the infectious agent is true? A. It is enveloped B. It is resistant to organic solvent C. It is spread through fecal-oral route D. It is released from the cell by lysis 2. In order to halt the spread of infection, you immediately administer post-exposure prophylaxis. Which of the following is an appropriate target for passive immunization and why? A. VAP B. Envelope C. Capsid D. Genome

1. A. It is enveloped -We know that ALL NEGATIVE SENSE RNA viruses are ENVELOPED with HELICAL SYMMETRY -All viruses that cause diarrhea are NON-ENVELOPED! Enveloped viruses cannot make it through the rough conditions of the GI tract B - No, it is enveloped C - No, think cruise ship story. Non-enveloped viruses are the ones that mostly cause diarrhea D - This is a characteristic of a non-enveloped virus and the virus they described is enveloped 2. A. VAP

Classification of Streptococci

1. ALPHA HEMOLYTIC STREPTOCOCCI -Alpha hemolysis is a partial lysis of red blood cells in Blood agar, resulting in a greenish discoloration around the colonies -Strep viridans group: *S.mutans *S.salivarius *S.mitis *S. anginosus -Strep bovis-now called Strep gallolyticus -S. pneumoniae (no role in infective endocarditis, will learn about this later) 2. BETA HEMOLYTIC STREPTOCOCCI -Beta hemolysis is a complete lysis of red blood cells. Seen on Blood agar, as total clearance around the colonies. -Eg: Strep pyogenes -Causes RHD but DOES NOT CAUSE INFECTIVE ENDOCARDITIS 3. GAMMA HEMOLYTIC STREPTOCOCCI -No zone of clearance around the colonies. -Eg: Enterococcus fecalis, E. fecium -Also some strains of strep bovis can also be gamma hemolytic

Protozoa

1. AMEBAE - these organisms move by blunt extensions of their protoplasm (pseudopodia) -Pathogenic *Entamoeba histolytica *Entamoeba hartmanni -Non-pathogenic *Entamoeba dispar *Entamoeba nana *Entamoeba coli *lodamoeba butschlii 2. FLAGELLATES - flagella for their locomotion -Pathogenic *Giardia lamblia *Trichomonas vaginalis *Dietamoeba fragilis -Non-Pathogenic *Chilomastix mesnili *Trichomonas hominis 3. CILIATESExposur -Pathogenic *Balantidium coli

After verifying his name age and address, she proceeds to take his temperature with a thermometer. After reading off the thermometer, she cleans the same using a pad containing a chemical. 1. What chemical does the wipe/pad contain? 2. Is it a disinfectant or a steriliant? 3. What other articles/surfaces can this be used to disinfect/sterilize?

1. Alcohol Swabs -Use alcohol hand-rub/soap to clean your hands and use alcohol swabs to clean things like thermometers, stethoscopes, etc. If you go get labs drawn, they will use the same type of wipe to disinfect your skin. 2. This is a DISINFECTANT, (If we're talking about alcohol swabs that go on the skin it is technically an ANTISEPTIC which is a disinfectant that you can apply to your living skin and mucous membranes). The big difference between disinfectants and sterilants is SPORES! Sterilization kills everything including spores, disinfectants does not kill spores. 3. Skin, stethoscopes, etc. Even some surfaces

Conjucation steps

1. F+ cells: male/donor cells Govern production of F pilus 2. F-: female/recipient cells 3. Hfr: F plasmid in F+ cell may integrate into chromosomeHfr -Transfer via conjugation usually incomplete so recipient stays F- 4. F': F plasmid in F+ cell may integrate into chromosome to form Hfr. If subsequent excision inaccurate and chromosomal genes also excised -> F' plasmid. *If transferred, recipient cell becomes F'

Phage Life Cycle—Lytic (Virulent) -Viruses that only infect bacteria

1. Attachment of bacteriophage 2. Injection of nucleic acid 3. Phage genome replication; unfolding and degradation of host genome 4. Tc and Tl of late genes 5. Assembly 6. Host cell lysis, release of phages

Phage Life Cycle—Lysogenic (Temperate)

1. Attachment of bacteriophage 2. Injection of nucleic acid 3. Viral genome integrating with the bacterial chromosome by recombination - PROPHAGE 4. The bacterial cell will go on to live as normal. The bacterial will divide during binary fision, and the viral genome in there will get passed on from one generation to another 5. Occasionally, the prophage may excise from the bacterial chromosome by another recombination event, initiating lytic cycle (ie. cold sore) ........

1. How many chromosomes are harboured in the average bacterial species? 2. What is the size of bacterial ribosomes? 3. Which bacterial genus incorporates a cholesterol derivative in its membrane? 4. Which component of a cell wall is responsible for its maintaining shape? 5. Name the repeating units of this component? 6. Name the specific target of beta-lactam antibiotics

1. Average of ONE 2. 70s -This is significant because the size of bacteria is not the same size of that of eukaryotes, therefore you can have drugs that target these bacterial ribosomes without harming the patient 3. Mycoplasma 4. Peptidoglycan -Rensposible for maintaining cell wall shape, rigidity, etc 5. N-acetylglucosamine (NAG) and N-acetylmuramic acid (NAM) -These repeating units form peptidoglycan. NAM and NAG are linked to together via BETA 1,4 GLYCOSIDIC BONDS. Coming off only the NAM is the PENTAPEPTIDE that connects each NAM/NAG chain. Attached to the PENTAPEPTIDE is also the 5 unit GLYCINE BRIDGE thats attaching both of the pentapeptide chains from each layer and as it does so it takes off the very last D-alanine so you'll have one less D-alanine on the second layer 6. PBPs (Penicillin Binding Proteins) -These play a major role in the formation of the cell wall -These are a family which includes some enzymes like transpeptidases, transglycosylases, etc. -They help the components of the peptidoglycan to bind to each other *Transpeptidases - transfer of peptide bonds between layers with the peptaglycine bridges connecting them

In the OR, suggest and justify the most appropriate method(s) of disenfection or sterlization, in the correct sequence, for the following: 1. Floor w/ blood on it 2. Trolley w/ blood on it 3. Gauze with blood on it 4. Instruments w/ blood on it 5. Kidney Tray w/ blood on it

1. Bleach -When you see blood, you are worried about HIV, Hep B, which are both enveloped viruses. Bleach will kill enveloped viruses. (Side note: "If we could administer IV bleach, we could kill the HIV/Hep B however that would probably not be good for the patient"). You would pour the bleach, wait 10 minutes, then clean it up) 2. High grade disinfectant for prolonged exposure can act as a chemi-steriliant -Trolleys/carts are made up of metal/steel. -You can also do an autoclave if the hospital has an autoclave thats big enough to handle the trolley 3. Place in a bio-hazard bag and send it to the incineration -You would do this for bandages, gauze, dressings, and similar things that have body fluids on it -Sharps go into a hard puncture proof container 4. Put it in a base with disinfectant (where it breaks down organic matter), then send it to a ULTRASONIC WASHERS (where it will further break down and remove the organic matter from the nooks and crevices). Then the autoclave! -Before you put it in the autoclave it must go through these steps first. You must decontaminate first! -The disinfectant placed to soak in the base usually has an enzyme cleaner in it 5. You would treat the kidney tray like the rest of the instruments. place it in a base with disinfectant, send it to the ultrasonic washer, then to the autoclave

A 5 yo child is brought to the clinic by his mother, with complaints of profuse watery diarrhea for the past day. There is no history of passage of blood in the stools. On examination, the child appears mildly dehydrated. 1. Which of the following is a characteristic of the most likely causal agent? A. Enveloped (+) sense RNA virus B. Enveloped (-) sense RNA virus C. Enveloped (+) sense DNA virus D. dsRNA non-enveloped virus E. ssDNA non-enveloped viurs 2. What is the nature of the immune response that is required for protection from infection? 3. A few months down the line the child comes down with a rotaviral (Reoviridae) diarrhea. How could this explained in the context of the immune response? 4. What is the nature of the genetic interaction that may be seen between two rotavirus or reoviridae virions infecting the same host cell? Justify why the virus undergoes the particular genetic interaciton?

1. D. dsRNA non-enveloped virus OR E is also an acceptable answer at this point in time -Next semester you will be expected to know that D is the answer because ROTAVIRUS is a dsRNA that is non-enveloped 2. Humoral Response -Non-enveloped viruses tend to have lytic cycles, therefore they multiply, replicate, lyse the cell, and do this over and over. If you have a strong local antibody response you probably can protect against this viral infection. In the GIT you're looking for secretory IgA (and this is actually what the rotavirus vaccine dones) 3. It happened because of antigenic variation due to the mixing of the segments in the dsRNA! -Remember BOAR! These are the ones that undergo reassortment. *Bunyaviridae *Orthomyoxoviridae *Arenaviridae *Reoviridae 4. Reoviridae has a segmented genome therefore they are capable of REASSORTEMENT.

1. Name a unique feature of the Gram-negative outer membrane not found in Gram-positive bacteria? 2. What specific component of the bacterial cell wall initiates septic shock? 3. Parts of the LPS molecule 4. In E. coli O104:H4, to what does the H refer? What does the O refer to? 5. What is the name given to the compartment between the outer and cytoplasmic bacterial membrane? Gram positive or gram negative? 6. Name the bacterial appendage used for transfer of genetic material between bacteria. What is something special about them? 7. What is another type of variation that is possible?

1. LPS is exclusive to the OUTER MEMBRANE (specifically the outer leaflet of the outer membrane) which is only found on GRAM NEGATIVE bacteria -Gram negative bacteria have both an OUTER and INNER membrane. LPS is on the OUTER. Gram positive bacteria does not have an outer membrane -You will not find LPS on anything but the outer membrane of gram negative bacteria 2. LIPID A portion of LPS/ENDOTOXIN 3. Core, O polysaccharide (O antigen) which is antigenic, and Lipid A 4. H = flagella, O = O polysaccharide/O antigen 5. The PERIPLASM, only found in Gram-negative bacteria between the outer and inner membrane 6. Pilli, they can exhibit ANTIGENIC VARIATION which will elicit antibody production -Pilli on one strain can "look" similar to the pilli on another strain but be different molecularly -Antibodies are specific, so it can recognize one pilli, but not recognize after it undergoes antigenic variation 7. Phasic Variation - where the pilli is completely lost

A patient is brought into the ER with profuse watery diarrhea. He just returned from an overseas trip to Haiti. A gram stain of the stool is given alongside demonstrating the causal organism of his condition. 1. What would be the morphology, Gram staining property of the likely causative agent? 2. Which of the following type of toxin is most likely to be produced by the infecting microorganism? Endotoxin or exotoxin 3. Describe the nature of the toxin 4. What property of the toxin is exploited in devising a strategy to prevent the infection?

1. Rod-shaped, Vibrio, Gram negative 2. At this point in your learning, the answer could be either endotoxin or exotoxin. -Technically this is Vibrio Cholerae (which you do not have to know yet) & Vibrio Cholerae is more famous for its EXOTOXIN. -Exotoxins are always protein in nature and can be turned into a vaccine because proteins are highly antigenic -Exotoxins are the ones that are secreted out into the environment, endotoxins are secreted once there is disruption of the bacteria/cell lysis (because the Lipid A head of the outer leaflet of the outer membrane gets released when the cell lysis - causes release of many cytokines) -Endotoxins are weakly antigenic (as they are not proteins) 3. Exotoxin - protein (see answer 2) 4. Exotoins are AB toxins and AB toxins have an Active portion and a Binding portion. So if you were to produce a vaccine against a toxin, which subunit would you target? B! The Binding part of the toxin is what you would target. If you get rid of binding you get rid of adhesion

A 25 yo male presents to the clinic with complaints of fever, malaise, myalgia, and a loss of appetite for the past two days. The past day, he developed a dry cough and sneezing that has prompted him to seek medical attention. A diagnosis of viral respiratory infection is made and the patient is sent home with acetaminophen, and is advised to take plenty of fluids and rest. From the respiratory tract of the patient, two viruses were isolated. -Virus 1 - ENVELOPED VIRUS -Virus 2 - NON-ENVELOPED VIRUS 1. Which of the following is responsible for the first step of viral infection by either of these viruses? 2. After examining the patient, the physician uses an alcohol based hand sanitizer. Which of the two viruses would be most affected by the hand sanitizer 3. Identify the component/s of the virus that would be affected and why? 4. What is the biological consequence of using the alcohol based hand sanitizer 5. If you were to develop a vaccine against these viruses, which of the following components would be a good candidate for vaccine development and why?

1. Virus 1 - The envelope, Virus 2- The capsule The envelope, Virus 2- The capsule -The first step of viral infection is attachment, and these are the parts of the virus that will mediate in attachement 2. The enveloped virus 3. The virus has a lipid envelope (which is derived from our/host cell membranes which are very sensitive to alcohol ), so when the envelope is destroyed, it disrupts virus infectivity -Even though the envelope is the only thing that is disrupted, the VAPs on enveloped viruses are on the envelope, so if you disrupt the envelope, you disrupt the VAPs, therefore it cannot attach to host cells. 4.Disruption of the lipid membrane (as well as the VAPs) on the lipid membrane which prevent the spread of the virus 5. You would want to target the VAPs on the envelope in enveloped viruses and on the capsid in the non-enveloped viruses. It is the same concept, you want to target attachment! -WIth the vaccine you try to elicit the neutralizing antibody response to bind the VAPs

CLINICAL CASE 2 -A 60 year old lady undergoes a heart valve replacement surgery. Approximately 6 weeks later she experiences malaise, joint pains and a fever of 38.8 degree C. -On examination, splinter hemorrhages of the nail and soft nodular lesions on the palm are observed. Further fundoscopic examination of her eyes reveals hemorrhages. On Chest auscultation, a significant new murmur is heard. 1. Is this acute or slow onset of infective endocarditis? 2. What are features that let you know this is infective endocarditis?

1. Slow onset 2. Splinter hemorrhages of the nail, soft nodular lesions (janeway if not painful), hemorrhages in the eyes on fundoscopic exam (roth spots), new murmur heard

CLINICAL CASE 1 -A 40 -year old male presented to the ER with a 2 week history of low grade fever and fatigue, myalgia and arthralgia. He has recently noted a painful nodule on the tip of his left index finger. -The patient had a past medical history significant for hypertension, and hyperlipidemia. He reported that he had a tooth extracted 4 weeks ago. His echocardiography is positive for an intra-cardiac oscillating mass on the valve. -His blood cultures yield a significant pathogen. 1. What features would suggest Infective Endocarditis? -What are the other organisms that can be spread because of tooth extraction? 3. Suppose this pt had undergone a valvular graft, how would damage to this valve shown up on the echocardiography?

1. The features that would suggest infective endocarditis: Osler nodes (painful node), tooth extracted weeks ago, oscillating mass on the valve, positive blood culture. 2. With tooth extraction, organisms like strep viridans group (inclusive of strep mutans) can enter the blood stream. -The HACEK group 3. Rupture from its implanted site

The life Cycle of Trypanosoma Cruzi

1. Vector bites a human and defecates on the skin (Reduviid/triatomine bug) 2. Metacyclic trypomastigotes are rubbed into the eye or bug's bite 3. Trypomastigotes enter cells and reproduce as amastigotes (smaller forms, they have lost the flagella, easier to get around) 4. Cell dies, amastigotes released. Some amastigotes will infect new cells, some amastigotes will transform into trypomastigotes and remain in the blood stream 5. Vector infected when it ingests amastigotes or trypomastigotes in a blood meal (can go back to 2) 6. The parasite reproduces asexually in the vector's gut. Metacyclic trypomastigotes move to the rectum of the vector

Gram Stain (continued)

2 stains. Primary and component stain. STEPS: -Prepare slide (pt specimen) and allow it to air dry -Apply heat to better fix it to slide. 1. APPLY CRYSTAL VIOLET and allow it to incubate for a while *In gram + bacteria the crystal violet are entering the thick peptidoglycan cell wall and the stain will be trapped in it because it is so thick *In gram - bacteria the crystal violet are entering the peptidoglycan similarly. *This will then appear BLUE - Rinse the slide with water 2. GRAMS IODINE ADDED - forms large complex with the peptidoglycan, and depending on the thickness of the membrane, won't let the crystal violet leave *Both would appear PURPLE at this point 3. DECOLORIZER (ALCOHOL or ACETONE) -Will leach crystal violet out of the bacterial cell wall *Now Gram - is clear because it leaked all the crystal violet out and the wall isn't thick enough to hold it *The Gram + remains purple because it is very thick, the decolorizer can't get rid of the crystal violet with the large complexes with iodine from it 4. SOFRONIN STAIN (Secondary stain) -Safranin retained in both (causing pink) but in gram + still has retained stuff so it stays purple -Safranin turns the colorless pink

Mr. J is recovering from a life threatening infection in the hospital. He was on a prolonged course of multiple, broad spectrum antibiotics. A. Besides drug toxicity or allergy, list at least two possible complications that could possibly result from the broad spectrum antibiotics B. Provide the rationale for each of the complications and the most appropriate interventions to offset the complications

A. 1. Good bacteria are knocked off and the bad bacteria start flourishing leading to pseudomembranous colitis due to C. diff growth leading to antibiotic associated diarrhea. Candida can also cause abx associated diarrhea. B. Drug resistance and infection with drug resistant pathogens. The competition between all the microbes is what keeps all the bacteria numbers down and balanced. If broad spectrum abx is given, there is always going to be something there that is resistant and those begin to multiply

Which of the following represents the first step in initiating infection of the gastrintestinal tract by an enteric pathogen? A. Adhesion B. Colonization C. Invasion D. Systemic spread

A. Adhesion -If you want to colonize Mars, you must land there first!

If one were to take a vaccine for active immunization against an enteric pathogen causing gastroenteritis, which of the following steps would be the best target of the immune response generated by the vaccine in preventing infection of gastrointestinal tract by the enteric pathogen? A. Adhesion B. Colonization C. Invasion D. Systemic spread E. Multiplication

A. Adhesion -The first step of infection is adhesion (interaction between the ligand on the bacteria with the ligand on the host cell). Adhesion may be specific or nonspecific. If you are able to somehow block the receptors or ligands on the bacteria, you would probably get rid of the first stage of infection itself. If you present adhesion you prevent the next step (colonization)

Which of the following represent the first line of defence against fungal spores (linked to previous question)? A. Alveolar macrophages B. Neturophils C. T- cells D. B-Cells E. NK Cells

A. Alveolar Macrophages -They are inhaled, they go to the respiratory track, the alveolar macrophages try to ingest them and kill them. The answer here is alveolar macrophages specifically because you are talking about fungal spores coming into the the respiratory tract where there the first line of defense is the alveolar macrophages. Suppose some of the spores escape and they are able to germinate and produce the fungi and the fungi start to trying to get invasive, thats when the neutrophils come in and line up against the fungal element and they start firing/dumping their granules on the hyphae. -It is not neutrophils! NEUTROPENIA means a decrease number of neutrophils. Neutropenia is a risk factor for systemic fungal infections!

A 58 yo male underwent surgery. Standard preoperative prophylactic protocol with ciprofloxacin antibiotic was administered. The patient also had two previous intervals of ciprofloxacin administration for prostate biopsies. Notably, the patient spent much of his time in an oncology unit amongst a large number of immunocompromised patients. A day after the biopsy, the patient became systemically ill, had to be resuscitated and was given aggressive therapy, including an altered antibiotic regimen that led to his recovery. During this illness, blood cultures grew E. coli that were resistant to ciprofloxacin and four other antimicrobial medications. Which of the following best accounts for the change in outcomes of the patient's second and third biopsies? A. Antibiotic resistance plasmid transferred by conjugation B. Antibiotic resistance plasmid transferred by specialised transduction C. Bacteria produced spores that could withstand the antibiotic then germinate once this treatment was concluded D. Resistant E. coli from his colon released lipopolysaccharides into his bloodstream E. Resistant E. coli from his colon released teichoic acid into his bloodstream

A. Antibiotic resistance plasmid transferred by conjugation

A 56 yo woman recently had surgery after which a urinary catheter was inserted. She later develops symptoms consistent with a urinary tract infection. Although she was treated, the infection persisted. Explain possible reason(s) for this outcome. A. Biofilm produciton B. Fimbriae C. Outer membrane D. Peptidoglycan layer E. S-pillin F. Sterol-containing membrane

A. Biofilm production Why A and not B? Catheters are usually made of plastic. Fimbrae are good at holding on to tissue but not plastic

A 68 yo male was admitted to the hospital from his long-term care facility after 1 week of dyspnea and cough. A BAL sample was sent to the laboratory with COPD exarcerbation topping the differential diagnosis list. In addition to the standard test (which revealed the causative agent), a trainee lab technician decided to perform the Quellung reaction. This test was also positive. Which of the following is most likely regarding the etiological agent? A. Express K antigen B. Forms endospores C. Has flagella D. Has pilli E. Possesses lipopolysaccharide

A. Express K antigen

A 65-year-old diabetic male presents with fever, headache and facial pain and swelling. A CT-scan of the head and paranasal sinuses reveals a mass in the maxillary sinus, and a biopsy shows aseptate hyphae. Which of the following are most likely representative of the fungal elements seen on biopsy? A. Image has no septae - Has no spores B. Image has septae C. Image has no septae - Has spores D. Image has septae

A. Image has no septae - Has no spores -Asexual spores in C are not seen in the body, only in the environment -In the body, YOU ARE ONLY GOING TO SEE HYPHAE, no spores!

A 50-year-old female presents with skin lesions on her hand and arm. Initially there was just one lesion near her wrist, which progressed to multiple lesions occurring along her hand and then arm. Infection with a dimorphic fungus is suspected. Which of the following are you most likely to see in a biopsy from the patient's skin lesions? A. Intracellular Yeast (the yeast are specifically inside of the macrophages) B. Mold with hyphae and mycelium

A. Intracellular Yeast (the yeast are specifically inside of the macrophages) -Humans are only infected by yeast forms!

A gram negative enteric pathogen is grown in the laboratory from the stool sample of a patient with diarrhea. Which of the following components of the organism facilitates its safe passage through the gut? A. Outer Membrane B. Capsule C. Cytoplasmic Membrane D. Pilli/Fimbrae E. Flagella

A. Outer Membrane -How does the pathogen get all the way to the colon? The organism gets ingested, goes down the esophagus then into the stomach acid. If it is able to withstand gastric acid, it goes to the basic/alkaline duodenum. The pH shock from the acidic to the basic environment can again kill alot of the microbes. If they are able to survive this pH shock, they will then get hit by bile which is made of bile salts which are detergents. The detergents help distrupt the microbe however gram negative bacteria have an OUTER MEMBRANE. The outer membrane helps the bacteria resist detergents and disenfectants. The outer membrane also can help resist antibiotics to some extent because the peptidoglycan layer is inside the outer membrane therefore the abx has to first get through the outer membrane in order to be effective. This can be done through PORIN channels but if the antibiotic is too large it cannot get through those channels. The outer membrane acts as a barrier and doesn't allow stuff to get into the organism to kill it. -Also note that between the outermembrane and the peptidoglycan layer is the PERIPLASM! The periplasmic space has many enzymes that can inactivate antibiotic. -The capsule is thick however it is NOT a mechanical barrier!

A patient who has recently emigrated from Egypt presents with painless terminal hematuria. An examination of the urine yields eggs of a parasite. Which of the following infectious agents is it likely to be? A. Schistosoma hematobium B. Taenia solium C. Taenia saginata D. Strongyloides E. Trichuris trichura

A. Schistosoma hematobium -In the pic on the L you see a terminal spine! This gives it away.

Why Genetic Exchange?

Adaptation by bacteria to changing environmental conditions; acquisition of useful genes *Antibiotic resistance, virulence factors DNA repair—uptaken DNA can serve as template for repair *Processes of homologous and nonhomologous recombination (review from med1) Source of Nitrogen, Carbon, Phosphorous

Bacterial Growth (continued...) -Generation time = the length of time it takes a bacterial cell to replicate (Ie. Ecoli doubles in 20 minutes) -Incubation time = the length of time it takes the bacteria to get to a certain mass/number to where you can identify symptoms

At optimum temperature and nutrients, E. coli doubles itself in 20 minutes (generation time), M. tuberculosis (20 hours) Average 30 to 60 minutes Growth rate depends on species of bacteria, temperature of medium, chemical composition of medium

Which of the following would be most appropriate to prevent the above side effect? A. A second antibiotic B. A probiotic C. A large bowel enema D. A mineral supplement E. A vitamin supplement F. A fecal transplant

B. A Probiotic -This is preventative and this is what the question is asking A - a second abx can help but there is no abx that can kill everything F - used for more than 3 recurrent C. diff infections

A 30 yo man with a prosthetic valve replacement experiences malaise, joint pain and fever after a few weeks. O/E the physician notes a temp of 39 deg C and hemorrhagic streaks on his nails. An echocardiography shows the presence of an intra-cardiac mass on the implant. Two blood cultures yield gram positive cocci that grow in high salt concentrations and are catalase negative. 1. Which of the following is associated with the most likely etiological agent? A. Pale colonies on Mac Conkeys agar B. Black colonies on Bile esculin agar C. Green pigmented colonies on Nutrient agar D. Whitish colonies on Saborauds dextrose agar E. Beta hemolytic colonies on Blood agar 2. What underlying condition mostly predisposed for this infection?

B. Black colonies on Bile esculin agar Dx = Infective Endocarditis -The list of organisms that can grow in high salt agar so far: S. aureus, S. epidirmidis, Enterococcus -However the vignettes says it is catalase negative, therefore now this just leaves you with enterococcus -Enteroccocus grows can produce black colonies on bile esculin agar (along with Strep bovis) A - This agar is for gram - bacteria C - This is pseudomonas D - This is fungi like candida E - S. aureus 2. Gut manipulation or cystoscopy

A 46 yo woman presents with fever, nausea, painful and frequent micturition. Urine and blood samples were collected and sent to the laboratory for analysis. She was diagnosed with recurrent UTI due to co-infection with two strains of E. coli. Which of the following would confer the best advantage for maintenance of these bacteria in the urinary tract? A. Flagella B. Fimbriae C. Plasmids D. Outer membrane E. S-pillin F. Thick peptidoglycan layer

B. Fimbrae (pilli) -This helps it hold on to the urinary tract. They can hold on despite the rate at which urine is being passed -E coli is a common cause for UTI

Four weeks after returning from a hiking trip, a 35-year-old male presents with complaints of bloating and abdominal discomfort. A stool examination reveals the presence of the cysts of a flagellate. Which of the following is most likely to be the infectious agent? ​ A. Entamoeba histolytica B. Giardia lamblia C. Balantidium coli D. Ascaris lumbricoides E. Schistosoma hematobium F. Taenia solium

B. Giardia lamblia Be aware of answers of amoeba, ciliate, flagellates, etc. Know which ones are protozoans and which are helminths Parasites can be broadly classified into unicellular and multicellular parasites. -Unicellular = Protozoans -Multicellular = Helminths -Among the protozoans you have your LUMINAR protozoan = Amoeba, flagellates & ciliates. Then you also have your TISSUE protozoans like Leishmania, Malaria, Hemoflagellates, etc -Among the Helminths you have the FLATWORMS which includes A. tapeworms (tinias), and B. Flukes. Then you also have the NEMATODES which include the roundworm, hookeworm.

In the laboratory a virologist is studying the properties of HIV. She infected the same cell with both HIV and the rabies virus. (HIV can infect only human CD4 cells whereas Rabies virus can infect both human and dog cells.) Some of the progeny virions were able to infect dog cells within which they found HIV specific RNA. Which of the following describe the ability of the progeny virions to infect dog cells? A. Complementation B. Phenotypic mixing C. Reassortment D. Recombination E. Lysogeny

B. Phenotypic mixing -The point is that you do not normally get HIV in dog cells because the receptor is not there. However when you put them in a cell with rabies (both, by the way, are enveloped viruses), now the dog cell is able to get the HIV virus. If phenotypic masking were it, this could also be a likely answer.

Following 3rd degree burns, a 45-yr-old-female is admitted to the hospital for intensive care. A few days later, she develops high fever and discharge from the extensive skin lesions. O/E her temperature is 39 C, HR: 120/min, B.P: 100/54 mmHg and RR is 24/min. She has leukocytosis and her serum CRP is high. Skin swabs and her blood samples yield a significant gram negative pathogen which forms green pigments. Which of the following is a characteristic of the most likely pathogen? A. Presence of Lipoteichoic acid B. Presence of Lipid A C. Formation of Superantigens D. Thick peptidoglycan E. Formation of coagulase

B. Presence of Lipid A -Pseudomonas aeruginosa is the likely organism which is a gram negative organism. Lipid A is the portion is a part of the endotoxin that is released when pseudomonas aeruginosa dies.

The formation of a mating pair during the process of conjugation in Escherichia coli requires which of the following? A. Lysis of the donor B. Sex pilus C. Transfer of both strands of DNA D. Restriction endonuclease E. Integration of a transposon

B. Sex Pilus A - This happens in the LYTIC cycle of TRANSDUCTION C - Happens in TRANSFORMATIOn E - Can happen in TRANSFORMATION, but this is not correct because we're talking about conjugation. This can happen in conjugation also however for them to mate in the first place you need a sex pillus *THIS IS NOT GENERALIZED TRANDUCTION. Generalized transduction is going to require a phage.

A research studying viral replication in RNA viruses notes that the viral mRNA produced is complementary to the parent genome. Which of the following represents the virus under investigation? A. ssRNA (+) enveloped B. ssRNA (-) enveloped C. ssRNA (+) non-enveloped

B. ssRNA (-) enveloped -Most (+) RNA Viruses are already in their sense mRNA and the moment you stick them in the cytoplasm they can start translation -Our cells have NO IDEA how to replicate RNA viruses so therefore the (-) RNA viruses need to come in with their OWN ENZYMES or they cannot make copies of themselves. So if you have a (-) sense RNA and their polymerase starts replicating the genome, will the daughter strand be (+) or (-)? It will be complimentary to the parent genome, therefore it will be (+) RNA sense strand. -Humans have enzymes to replicate our own DNA (DNA dependent DNA polymerase) and to make RNA we have (DNA dependent RNA polymerase). In our cytoplasm we don't have any of these enzymes available to the viruses. Imagine you are a DNA virus and you have 2 DNA strands, you get into a cell and you want to start replicating but you don't have an enzyme. You need DNA Dependent DNA polymerase but where is the enzymes? It's in the nucleus! This is why all the DNA viruses replicate in the nucleus with exception of the POXVIRIDAE FAMILY (which replicate in the cytoplasm, therefore it brings its own enzyme). Bigger DNA viruses code for their own polymerases but they all start off with the host DNA dependent DNA virus. Then they have their own made because they are more efficient, making much more!

Differentiate the clinical presentations of Bacteremia, Septicemia/ Sepsis and Toxemia

Bacteremia: presence of circulating bacteria in the blood. -Give examples of conditions leading to bacteremia *Brushing your teeth vigorously, flossing your teeth, straining at defecation *-Many times you have bacteria in your blood but the load is not huge. These bacteria are there transiently - TRANSIENT/INSIGNIFICANT BACTEREMIA. These organisms get mopped up by immune system and they are not significant enough to cause infection. -What will be the signs and symptoms? *You will have no signs and symptoms from this usually if just transient bacteremia Septicemia( Sepsis): A systemic response that is caused by the presence of pathogenic microorganisms/microorganisms and their toxins in the bloodstream. -Give examples of conditions leading to sepsis *Gangrene, pneumonia, toxic shock syndrome -What could be the signs and symptoms? *fever/chills, hypovolemia, hypotension, tachycardia, palpitations, cellular hypoxia, anoxia, malaise. If you do a lab exam, you will have increased WBC then after some time they WBC can decrease because the endothelial becomes leaky causing the WBC to leave to site. Toxemia: presence of circulating toxins in the blood (could be from bacteria dying and releasing its toxins in the blood) -Give examples of conditions leading to Toxemia. *Gram negative organisms *They produce mostly endotoxins, but there are some that can produce exotoxins *Diphtheria toxin doesnt present like sepsis but is still a toxemia. -What could be the possible signs and symptoms?

Pathogenic actions of bacteria (cont)

Bacteria can evade antibody responses by antigenic variation, by inactivation of antibody or by intracellular growth. Neisseria gonorrhoeae can vary the structure of surface antigens (pili) to evade antibody responses and also produces a protease that degrades immunoglobulin A (IgA).

Quorum sensing (aka CROSS TALK)

Bacterial biofilm: multi-layered aggregate of 1 or more spp, ENCASED IN A CAPSULAR MATRIX, attached to biological or inanimate surface that co-exist through cross-talk -Very few biofilms are composed of a single species Marked resistance to antibacterial agents -Lets say you have a patient that has an airway biolfilm and rx them abx and tell them to take it for 7 days. The abx may be successful during that 7 days as it may kill most of the layers of the biofilm however by the time the course of the abx is complete, there are some layers that could not be accessed by the abx therefore when the abx are no longer being taken, these few layers that are still viable will start growing and dividing again and building back up the biofilm layer QS (cross-talk): signalling mechanism whereby regulation of gene expression in response to population density leads to exchange of small -Lets say the above pt started off with just a few bacteria in the airway. Overtime as the bacteria grow and divide, they produce SMALL MOLECULES that are now identified by scientist. The more bacteria present, the more of these small molecules there are. When a QUORUM (number of bacteria) are present then the concentration of the small molecules is at a certain level and the bacteria are able to sense the presence of these small molecules and this leads the bacteria to turn on the expression of certain genes that may confer a virulence advantage for the bacteria -So now all the bacteria within this biofilm that are in the process of growing will begin to produce capsule (for example) because of the genes that have been turned on due to the high concentration of small molecules. This helps evasion of the immune system. Different type of virulence genes can be turned on. *Ie. Biofilm on lettuce years ago. Biofilms hard to get rid of

BOAR

Bunyaviridae Orthomyoxoviridae Arenaviridae Reoviridae

Epidemiologists at the CDC are investigating an outbreak of a yet unidentified virus. The pattern of manifestation of the disease and illness caused by the virus is plotted as shown in the following figure. Which of the following infection pattern is likely caused by the virus being investigated? X: Time Y: Infection Event Disease is Varicella (infects once, is latent, and comes back again later) A. Acute infections followed by chronic infection B. Acute infection followed by clearing C. Acute infection followed by latent infection and periodic reactivation D. Slow chronic infection E. Acute infection followed by persistent infection

C. Acute infection followed by latent infection and periodic reactivation -Suppose this was HIV instead? Here you would have an acute HIV infection, it may decrease a little but it remains a chronic infection and does not ever go latent (so its infectivity never comes down to zero)

Gram-Positive Summary

Capsule -Polysacharide except B.anthracis Surface Protein -Staph A, M protein GAS Teichoic Acid -lipoteichoic acid Cell wall; peptidoglycan -thick layer, cross linked, pentaglycine bridge, small and moderate molecules diffuse in Cytoplasmic membrane -Hydrophobic phospholipid bilayer -Antiphagocytic and immunogenic except for hyaluronic acid of GAS -Antiphagocytic, immunogenic -Attachment to epithelial surfaces, chunks from lysis, trigger alt C pathwy and cytokines -Rigid support; osmotic protection, chunks of PG & TA trigger alt C pathway, cytokine, shock -Membranous matrix for enzyme pathways, respiration, final stages of CW syntheis, PBP and transpeptidases

Clinical significance of bacterial cell wall (continued...)

Cell wall associated structures Teichoic acids -Mediate attachment to host cells -Used as vaccines Lipoteichoic acids -Produce inflammatory response in host M protein of Streptococcus pyogenes -Helps evade phagocytosis -Immunogenic *Streptococcus pyogenes is one of the common causes for streptococcal pharyngitis causing sore throat. Some pts get a sore throat, recover, then later end up with REUMATIC FEVER! This is because some of the bacterial cell wall traveled through the bloodstream and antibodies were generated against it. What happens later is that ab against the M protein could cross react with cardiac tissue because the protein profile of the cardiac valves resemble that of M protein. The cross reactivity of ab that were initially generated against M protein that have continued to circulate can cause problems later in life Group carbohydrate antigen of streptococci -Impedes entry of hydrophobic compounds -Resistant to bile salts

Types of virus mutants contd...

Conditional lethal mutants - like temperature-sensitive (ts) or cold-sensitive mutants -Have a mutation in a gene for essential proteins that allows virus production only at certain temperatures. -ts mutants generally grow well or relatively better at 30 to 35 deg. C (permissive conditions); they cannot grow at 38 to 40 deg. C (non-permissive conditions)

Pili/Fimbriae (Continued..)

Clinical significance -Adherence: E. coli: uroepithelium, intestinal tract; Neisseria gonorrhoeae: mucosa of genital tract -Only piliated N. gonorrhoeae initiates infection *The Pilli here are an essential virulence factor -Sex (F) pili used for CONJUGATION —transfer of genetic material such as plasmids

Coagulase enzyme -This create fibrin barriers -Whenever the organism is flowing through the bloodstream or in the skin or tissues, the proteins that leak out from the capillaries will accumulate around the organism (fibrinogin), and coagulase enzyme will act on that -Ie. Abscess or fasciculitis there is a walling off of the organisms -When compared to Streptococci they do not produce coagulase, they produce Hyaluronidase instead. Staph can also produce hyaluronidase, however streptococci produces it much more! If hyaluronidase is made in large amounts, it spreads! -Clinically, if you see a pt with a well circumcised abscess like lesion with redness in the periphery then this is caused by STAPH! On the other hand, with strep you will get more of a spreading type fasciitis, cellulitis, spreading type of manifestations. -When staph produces hyaluronidase (which is in small amounts), it helps the vegetations spread, but again this is found more in streptococci

Coagulase enzyme plays an important role in producing a fibrin barrier and walling off lesions, protecting it from phagocytosis Staphylococcal localizing lesions are usually well circumscribed (abscesses) as against streptococcal lesions that are spreading in nature (cellulitis) -Well cirumscribed pustules in case of folliculitis of the skin -Staphyloccoal vegetations found on the valve

Intravascular bloodstream infection

Common organisms: -S.aureus -S.epidermidis -E.coli, Klebsiella, Candida

Factors which influence the resistance of microorganisms to various procedures employed (continued)

Concentration and potency -With other variables constant, and with one exception (iodophors), the more concentrated the disinfectant, the greater its efficacy and the shorter the time necessary to achieve microbial kill. -Length of disinfection time is inversely proportional to the potency of the agent

A 46 yo woman presents with fever, nausea, painful and frequent micturition. Urine and blood samples were collected and sent to the laboratory for analysis. She was diagnosed with recurrent UTI due to co-infection with enteroaggregative E. coli and extended spectrum B-lactamase (ESBL)-producing uropathogenic E. coli. Which of the following regarding the ESBL strain is most likely? A. B-1,4-glycosidic linkage in cell wall cannot form B. B-lactam antibiotics cannot access peptidoglycan layer C. B-lactam antibiotics exceed size exclusion limit of outer membrane porins D. B-lactam antibiotics will be inactivated by the ESBL E. B-lactam antibiotics will lead to a full recovery F. Transpeptidation in bacteria cell wall prevented

D. B-lactam antibiotics will be inactivated by the ESBL -Beta Lactamase is produced in the PERIPLASM, therefore to inactivate something thats in the periplasm, the antibiotic must have gained access to the periplasm. B - they can access the peptidoglycan layer C - They can get in E - wrong F - transpeptidation is inactivated by inactivating transpeptidase which is not happening here. The transpeptidase and the Beta lactamase belongs to the bug (not the host) this helps inactivate the abx rx'd to the pt

A 28 yo man presents with high fever with petechiae on the mucous membranes and conjunctiva. On examination, the patient is febrile, and has tenderness over his joints. In addition to a few maculo-papular lesions on his hands, a tricuspid murmur is detected on chest auscultation. Two blood cultures grow Gram positive bacteria. Which of the following underlying conditions most likely predisposed him for this infection? A. Valvular surgery B. Dental extraction C. Cytoscopy for bladder stones D. IV drug abuse E. Carcinoma of the colon

D. IV drug abuse -Tricuspid valve is usally associated with IV drug abuse -2 blood cultures showing gram positive: could be S. aureus or S. epidermidis -If you wanted to describe it further and ask about staph epidermidis, you could talk about how it is catalase positive and coagulase negative -What does valvular surgery predispose people for? Almost anything but most likely S. epidermidis, S. aureus, Pseudomonas, Candida -If it were a dental extraction you could have thought of S. mutans (in the S. viridans group) or HACEK! -In cystoscopy and bladder stones you would think of strep enterococcus -In colonoscopy you could think of strep. bovis or enterococcis -Which one causes carcinoma of the colon? S. bovis!

Scientist have engineered a bacterial cell to harbor a stretch of bacteriophage DNA. within its chromosome. Which of the following best explains how this was accomplished? A. Conjugation B. Excision of bacterial DNA, lytic replication of bacteriophage C. Infected the cell with virulent phage D. Site-specific recombination with DNA from a temperate phage E. Uptake of bacterial DNA from the environment

D. Site-specific recombination with DNA from a temperate phage -This is SPECIALIZED TRANSDUCTION, the DNA described is a PROPHAGE. Site specific restriction endonucleases are needed here which cleave the DNA B - We're excising the VIRAL DNA not the bacterial DNA C - This happens only in generalized transduction (when this is definitely specialized transduction) E - Describes TRANSFORMATION

A 64 yo male diagnosed with colorectal cancer presents with fever and breathlessness of 5 day duration. Patient says his hands are "more red than usual" and there are painful lesions on his fingers. CVS examination is remarkable for a holosystolic, "blowing" murmur best heard at the mitral area which was not present during prior examination. His blood cultures are positive for gram positive cocci. Which of the following organisms is most likely responsible for patient's disease? A. Staphylococcus aureus B. Streptococcus viridans C. Staphylococcus epidermidis D. Streptococcus gallolyticus E. Eikenella corrodens

D. Streptococcus gallolyticus Aka. Strep bovis E - is a part of the HACEK!

A 46 yo woman presents with fever, nausea, painful and frequent micturition. Urine and blood samples were collected and sent to the laboratory for analysis. Culture of the urine grew bacteria that were subsequently gram-stained. The results are shown in the photo below. Which of the following can be inferred from the data? A. Cell walls contain teichoic acids B. Cells contain antibiotic resistance plasmids C. Cells have a waxy cell wall D. Cells express K antigen E. Cells have an outer membrane F. Primary stain retained after gram staining

E. Cells have an outer membrane -This gram stain is it is gram negative because they are pink A - this is only for gram positive B - Not enough info for this answer C - this is for Acid Fast bacteria. You cannot stain waxy cell walls D - K antigen is referring to capsule. However you can't tell from gram stain if it has a capsule F - Primary stain here is crystal violet and it is only retained in gram positive bacteria

A 35 yo man is admitted to the ER with severe headache, vomiting, and fever. He had recently undergone corrective surgery for his deviated septum. O/E, he has a temp of 39.2 deg C, B.P: 90/54 mmHg and has extensive severe burn-like rashes on his body and scaling of his palms and soles. Which of the following is the most likely diagnosis of this condition? A. Gram negative sepsis B. Gas Gangrene C. Drug reaction D. Gastroenteritis E. Toxic Shock Syndrome

E. Toxic Shock Syndrome -Why is this Toxic Shock Syndrome and not a Gram negative sepsis? You would not get a burn-like rash on the body or a scaling of palms and soles in a gram-negative sepsis! Also, you know he had nasal septum surgery which he most likely had to have packs in his nostrils which can result in staph aureus toxic shock syndrome if left too long. -Toxemia is MOSTLY done by gram positives. Here we're focusing on the toxins that are actively being released into the blood. The nature of action of the toxin is very specific based on the organism that it is producing. Ie. Tetanus toxin acts at the neuromuscular junction. In staphylococcus which produces toxic shock syndrome, it is so potent and is a superantigen, and clinically can look like gram negative sepsis -Sepsis can be caused by both gram positive and gram negative -With gram negative, sepsis can only be caused when the organisms are dying in the bloodstream releasing their endotoxin. This is called ENDOTOXIC SHOCK/GRAM NEGATIVE SEPSIS -A gram positive organism can also get into the bloodstream causing a much milder sepsis via its lipoteichoic & teichoic acids acting as PAMPS. We just call this sepsis by a gram positive organism -Manifestations of all gram negative organisms causing sepsis will be similar. Manifestations of different toxemias produced by gram positive organisms will be different. Some toxemias (like toxic shock syndrome) can resemble a gram negative sepsis in its manifestations but the mechanism is totally different

Which mechanism of bacterial horizontal gene transfer is most sensitive to extracellular nucleases during the DNA uptake process? A. Conjugation B. Generalized Transduction C. Homologous Recombination D. Specialized Transduction E. Transformation

E. Transformation -This is because here there is naked DNA in the environment, it can easily be taken up by nucleases

Enzymes and Toxins

ENZYMES -Coagulase: Converts fibrinogen to fibrin -Fibrinolysin: Dissolves fibrin clots -Lipases: Hydrolyzes lipids -Phospholipases: Hydrolizes phospholipids -Hyaluronidase: Hydrolyzes hyaluronic acids in connective tissue, promoting the spread of staphylococci in tissue TOXINS -Cytotoxins: Hemolysin, Leukocidins, Panton Leukocidins: Pore forming toxins which affect cells including erythrocytes and leukocytes -Exfoliative toxins: Split the intercellular bridges in the Stratum granulosum of epidremis -Exterotoxins (A-E): Responsiblle for food poisoning -Toxic hock Syndrome Toxin -1 (TSST-1): Super-antigen (stimulates proliferation of T cells and release of cytokines); produces leakage or cellular destruction of endothelial cells

Viral cytopathogenesis

Efficiency of viral replication in the cell depends on 1. Optimum temperature for replication, and 2. Permissiveness of cell for replication. This results in cytotoxic viral proteins, inhibition of cell's macromolecular synthesis, accumulation of viral proteins and structures (inclusion bodies), and altered cell metabolism (Ie. Cell immortalization)

Enatmoeba histolytica Trophozoite

Enatmoeba histolytica consist of both trophozoite and cyst form

Pathogenesis For infective endocarditis you would take blood samples. -However for myocarditis or pericarditis you would take stool samples since COXSACKIE B is the most common cause and it multiples in the gut/is shed in the feces

Entry through feco-oral route/ inhalation of aerosols *Here it is not about it going into the lungs, it is about getting trapped in our oropharynx and then we swallowing it THEN Attach to ICAM/ CD55/ Coxsackie-Adenovirus Receptor THEN Entry thru pharyngeal mucosa/ GIT mucosa Multiply in lymphoid tissue THEN Reach target organ Restricted initially by secretory antibodies and then serum antibodies that block viremic spread THEN Shed in feces for a long time

Cestodes

Examples of intestinal cestodes include: -Diphyllobothrium latum (fish tapeworm) -Taenia solium (pork tapeworm) -Taenia saginata (beef tapeworm) -Hymenolepis nana (dwarf tapeworm) -Hymenolepis diminuta (rat tapeworm) -Dipylidium caninum (dog tapeworm) Examples of tissue cestodes include: -Echinococcus granulosus (unilocular hydatidosis or hydatid disease; dog tape worm) -Taenia solium ("cysticercosis")

Nematodes

Examples of intestinal nematodes include: -Enterobius vermicularis (pinworm) -Trichuris trichiura (whipworm) -Ascaris lumbricoides (roundworm) -Strongyloides stercoralis (threadworm) -Necator americanus and Ancylostoma duodenale (hookworm) Examples of blood nematodes include: -Brugia malayi (Malayan filariasis or elephantiasis) -Loa loa (African eye worm) -Onchocerca volvulus (onchocerciasis or river blindness) -Wuchereria bancrofti (Bancroft's filariasis or elephantiasis) Examples of tissue nematodes include: -Trichinella spiralis (trichinosis) -Toxocara canis (visceral larva migrans) -Ancylostoma caninum (cutaneous larva migrans)

How do they compare with bacteria?

FUNGI -Domain: Eukarya -Cell wall: CHITIN, MANNAN, GLUCAN (serve as targets for antibacterial agents) -Cell membrane: Ergosterol -Capsule: Polysaccharide, mannan, glucan -Spores: Used for sexual and asexual reproduction -Thermal dimorphism: Exhibit by thermally dimorphic fungi different morphology at different temperatures -Metabolism: No obligate anaerobes BACTERIA -Domain: Eubacteria -Cell wall: Peptidoglycan -Cell membrane: No sterols (any exceptions?) *Mycopalsma -Capsule: Polysaccharide (any exceptions) *Bacillus antrhasis (has a polypeptide capsule) -Spores: Used to tide over unfavourable conditions -Thermal dimorphism: None -Metabolism: Entire spectrum from aerobic to anaerobic

Virus Structure

From inside to outside -Genome composed of Nucleic Acid -Capsid composed of protein *Protein coat for the virus *If the virus is a naked/non-enveloped virus, the capsid is the outer layer and all the ligands that the virus needs to bind to the host cell will be present on the capsule -Envelope composed of lipids, proteins and glycoproteins (only seen in enveloped viruses) *In this case it is the envelope that contain the ligands to attach to host cells

CLASSIFICATION OF GRAM NEGATIVE BACILLI

Gram Negative Bacilli 1. Facultative Anaerobes & Fermenters a. Entero-bacteriaceae -E.coli, Klebsiella Citrobacter, Salmonella, Shigella, Proteus, Yersinia -All of the above are oxidase negative 2. Vibrio - Oxidase positive 2. Aerobes & Non fermenters -Pseudomonas - Oxidase positive

Comparison of Gram-positive and Gram-negative bacteria

Gram Positive Bacteria -Sporulation: Some strains -Capsule: Sometimes present -Lysozyme: Sensitive -Exotoxin: Some strains Gram Negative Bacteria -Sporulation: Absent -Capsule: Sometimes present -Lysozyme: Resistant -Exotoxin: Some strains

Clinical significance of bacterial cell wall

Gram-negative bacterial OM -Strong negative charge (due mainly to the LPS molecule); This helps it to repel certain agents, evade phagocytosis and complement -Permeability barrier against lysozyme, bile salts, digestive enzymes, other antibacterial agents -Endotoxic shock: during infection release of endotoxin can cause fever and shock -Makes the bacteria less sensitive to some antibiotics: lactamase enzymes in the periplasmic space; porins exclude large MW antibiotics *Ie penicillins wont be able to have access to the peptidoglycan layer therefore won't be effective in the gram neg bacteria because it is blocked by an outer layer

Helical vs. Icosahedral symmetry

Helical symmetry e.g. Paramyxoviridae -Consists of repeated units of a single polypeptide species (protomer) -Each polypeptide unit (protomer) is hydrogen bonded to its neighbouring protomer -Self assemble into a helical cylinder -Associated with the viral nucleic acid -complex of protomers & nucleic acid - nucleocapsid Icosahedral symmetry e.g. Adenoviridae -An icosahedron is a structure with twelve vertices and twenty triangular faces -Several different polypeptides are grouped into structural assemblies called capsomeres -Capsomeres are hydrogen bonded to each other to form the icosahedron -The nucleic acid is located in the empty space created by the rigid icosahedral structure -Unlike helical particles, "empty" icosahedral particles can form, where the protein shell contains no nucleic acid.

So when do I sterilize and When do I Disinfect??? A Risk Based Approach!

High Risk (Critical) -Direct contact with a break in the skin or mucous membrane or entering sterile bodv lumen -Surgical instuments, needles, syringes, parenteral fluids, arthroscopes -Must be sterile ! Heat sterilized or irradiated Intermediate Risk (semi critical) -In contact with mucous membrane -ET tubes, Gastroscopes, Anaesthesia equipment -should be free from all microorganisms; however, small numbers of bacterial spores are permissible, thorough cleaning with chemical disinfectants. High-level disinfection Low risk (non critical) -In contact with intact skin -Bed pans, urinals, furniture -Clean/wash with disinfectants and detergents

Treatment of Enterococcal I.E and drug resistance

I.E due to Enterococcus spp: -Enterococci have intrinsic resistance to a number of antibiotics such as B lactam agents. -Therefore combination therapy is used. -First line treatment: Penicillin G/ Ampicillin + Gentamicin (Synergistic action of the two drugs). Ampicillin acts on cell wall and weakens it while Gentamicin inhibits protein synthesis. -In resistant cases, Vancomycin + Gentamicin -In case of Vancomycin resistant enterococci (VRE), drugs of choice are Linezolid and Quinupristin/ Dalfopristin.

As the patient is wheeled into the OR, the nurse anaesthesist notes that the patient has an IV line and a urinary catheter. An anaesthetic premedication is administered using a disposable syringe. Suggest and justify the most appropriate method of disinfection or sterilization for each of the items, at the time of manufacture.

IV lines, Urinary Catheters, and isposable Syringes come pre-packaged. You want to sterilize this at the time of manufacture, so what do you do? GAMMA RADIATION! -Radiation causes irreversible DNA damage, damaging the DNA beyond repair -Ethynyl oxide is a good answer but no longer used because it is carcinogenic. It is an alkylating agent. (Just fyi, there are many cancer drugs that are alkylating agents because they cause damage to the cancer drugs) -The similarities between gamma radiation and ethynyl oxide is that they have NO HEATING involved in their processes. The temperature of the article to be sterilized is not raised! They are useful methods of steirlization for HEAT LABILE OBJECTS! -These are made up of plastic and rubber. Autoclaving would not be a good answer because they wouldn't be able to withstand 120 degrees celcius/centegrade at 15 lbs per square inch pressure. -There is also something called a PLASMA GAS STERILIZER these days. A plasma is a highly ionized cloud of gas. This is kind of named weird because the plasma is only present for a fraction of a second and cannot account for the sterilization process however this is another common method of COLD STERILIZATION

Pathogenesis of sepsis Eg: Gram Negative sepsis induced by endotoxins sepsis Lysed gram negative bacterial cell -> LPS released -> LPS binds with LPS binding protein -> LPS/LPS binding protein complex attacks macrophage via various binding proteins (CD14, CD11/CD18, TLR02/TLR04 LPS receptors) -> release of cytokines from the macrophage (TNF, IL-1, IL-12, IL-6, IFN gamma) -> Activation of coagulation cascade (which can lead to Adult respiratory distress syndrome and disseminated intravascular coagulation), prostaglandins and leukocytes, activation of complement cascade -> Endothelial cell damage -> Multiple organ system failure

In gram positive organisms, other PAMPs such as teichoic acid and lipo-teichoic acids can induce septic shock. Some exotoxins formed by gram positive organisms also lead to shock. Eg: Toxic Shock Syndrome

The mattress cover is then inspected, and found to be worn out in places. Removing the mattress cover reveals the condition of the mattress as indicated in the image alongside. Suggest and justify the most appropriate method of disinfection or sterilization for the mattress with blood stains.

Incineration - destructive sterilization -It's done, you must burn it!

Inclusion bodies & Ribosomes

Inclusion bodies -Deposits, sometimes surrounded by a polypeptide membrane -Deposits of lipids, starch, phosphates (metachromatic granules) *The cell degrades inclusion bodies when it needs it for things like nucleic acid synthesis, etc Ribosomes -Sites of protein synthesis -70S; Composed of 2 subunits: 30S and 50S *This feature is exploited in the development of abx

Superficial mycoses cont

Infection : Aetiological agent -Black piedra: Piedraia hortae -White piedra: Trichosporon asahii -Pityriasis versicolor: Malassezia furfur -Tinea nigra: Hortaea werneckii (pic of Tinea nigra) *Just a black patch. Doesn't bother the pt much other than maybe looking a little unsightly

Live Virus Vaccines

Live virus vaccines (like the nasal influenza vaccine, FluMist) are often conditional or host range mutants and attenuated for human disease.

Diseases caused by S.aureus

Localized supurative diseases -Impetigo, Folliculitis (styes in the eye), Abscess, mastitis, wound infections Systemic Diseases: -Bacteremia and Endocarditis -Pneumonia and Empyema. -Osteomyelitis and septic arthritis Toxin mediated diseases -Food poisoning -Staphylococcal Scalded Skin Syndrome -Toxic Shock Syndrome

Lymphocutaneous nocardiosis Acid Fast example

Lymphocutaneous nocardiosis caused by traumatic implantation of N. brasiliensis in the middle finger

Clinical significance of bacterial cell wall (Continued...)

Lysozyme -Found in tears, saliva, mucus, lysosomes -Disrupts beta 1,4 linkage of peptidoglycan Cell wall acting antibiotics -Bind to transpeptidase (PBP) -β-lactams like penicillin inhibit murein synthesis Persisters -Stop growing, not lysed, become tolerant -A persister is a bacterium that may be effected by an anti-bacterial agent, but it wont be killed. It may be effected to the point where it is not able to grow (where at this point it is not useful) but it will not die. However, when the pt stops taking the abx, they will begin growing again Protoplasts -Gram-positive bacteria that lose peptidoglycan layer. May be able to re-synthesise in very few bacterium. However in most, once it loses this peptidoglycan layer, it dies. Spheroplasts -Gram-negative bacteria that lose peptidoglycan layer. OM retained -These will become very spherical (because they have lost the layer that helps confer shape) L forms -Some wall-less bacteria that are able to grow and replicate -These are not very common, but can do damage without a cell wall Autolysin -Enzyme required to nip and trim peptidoglycan -Bacteria with mutant forms of autolysin are resistant to lysis with penicillin

2. Spectrum of diseases caused by Candida -Most of them are MUCOCUTANEOUS infections

MUCOCUTANEOUS lesions -Cutaneous lesions -Oral thrush -Vaginal thrush -Candida lesions are typically described as creamy-white curd-like patches on the mucosal surfaces. SYSTEMIC illnesses -Blood stream infections -UTI

Diagnosis of Myocarditis/ Pericarditis -Its hard to differentiate between the two however: -PERICARDITIS: If you lean forward, the pain relieves, and if you lie down the pain comes back. All the pain is coming from the inflammed pericardium, so if you lean forward, the pericardium is kind of getting pushed away. Mostly caused by bacteria! This can also be caused by parasites (CHAGAS DISEASE). -MYOCARDIUM: Associated with the musculature AND the conduction system of the heart because it is there in the myocardium! Therefore you can also get tachycardia, arrhythmias, etc. Mostly caused by viruses! This can also be caused by parasites (CHAGAS DISEASE). -You can also see EKG changes: DIFFUSE ST SEGMENT ELEVATIONS!

MYOCARDITIS -A sharp or stabbing chest pain or pressure, which may spread to your neck and shoulders -Rapid or abnormal heart rhythms (arrhythmias) -Shortness of breath, at rest or during physical activity -Fluid retention with swelling of legs, ankles and feet Fatigue PERICARDITIS -sharp, stabbing chest pain in the middle or left side of the chest which may travel into the left shoulder and neck -Pain relieved by leaning forward -Fever, weakness, coughing, -Other signs and symptoms of a viral infection (this is the most important cause of infective myocarditis or pericarditis), such as a headache, body aches, joint pain, fever, a sore throat or diarrhea

Non-enveloped (naked) vs. Enveloped viruses

NON-ENVELOPED -Susceptibility to organic solvents, bile salts (detergents), drying, chlorination: More resistant -Infectivity: Retained after drying or exposure to organic solvents -Release of progeny: By lysis or host cell death -Transmission:Faeco-oral, fomites, dust; survives in the gastrointestinal tract -Immune response: Humoral ENVELOPED -Susceptibility to organic solvents, bile salts (detergents), drying, chlorination: Less resistant -Infectivity: Lost after drying or exposure to organic solvents (Why?) - because drying disrupts the envelope made of lipid (which contains the virus needs to unlock the cell) -Release of progeny: Released from the living cell by budding. Can produce chronic infections. -Transmission: Large droplet, secretions, blood transfusion, organ transplant; does not survive in the gastrointestinal tract -Immune response: Humoral and Cell mediated

Underlying conditions and possible etiology

Native Valve (NORMAL) -I. V. drug abuse By: -Organisms with high virulence -Staphylococcus aureus -Enterococcus spp -Gram negative bacilli -Staph epidirmidis Native Valve (DAMAGED) -Rheumatic heart disease -Mitral Valve Prolapse -Degenerative valvular calcific stenosis -Bicuspid aortic valves -Congenital valvular defects By: -Streptococcus viridans -Staph aureus -Staphylococcus epidermidis -Gram negative bacilli -Fungi: Candida, Aspergillus

Epidemiological/ predisposing factors for candida infection

Predisposing factors -Antibiotic therapy -Diabetes mellitus -Neutropenia -Chronic mucocutaneous candidiasis -Prolonged corticosteroid or immunosuppressive therapy -Prolonged exposure to moisture and maceration of skin THEN -Invasive techniques- Eg: Intravenous line/ -Urinary Catheterization Intravenous drug user -Disseminated infection from a focus THEN Blood stream infection CANDIDEMIA -Ability to produce biofilms THEN -Infective endocarditis: Vegetations produced -Ability to adhere to host cells and invade with hyphae and enzymes

Pathogenesis of Infective Endocarditis You must give prolonged treatment IV for this!

Prior valve damage or turbulent blood flow (Eg/:Congenital valvular diseases/ IV catheterization). Results in Endothelial damage and formation of Platelet-fibrin thrombi *Imagine all these platelet-fibrin thrombi sitting on the damaged valves. It creates a beautiful nest in which organisms can settle down. THEN Microorganisms lodge onto and adhere to the Platelet-fibrin thrombi *Bacteria gain access into blood stream. *Adherence aided by some virulence factors: Eg :extracellular dextran produced by S.viridans group The organisms start lodging into the nest and start multiplying and producing their on colonies there. This is known as a VEGETATION! THEN Formation of INFECTED VEGETATIONS *Vegetation is nothing but a web of platelets and fibrin put together which gets colonized by microbes. It almost becomes a relatively avascular site because it is like a cocoon there. It's so hard for antibiotics to reach there which is why YOU MUST GIVE IV ANTIBIOTICS OVER A PROLONGED TIME PERIOD (4-6 weeks) in order for them to have its effect. Thrombolytics will also help in the long run to dislodge. THEN Seeding of bacteria into blood at a slow and constant rate *The infected vegetation gets broken off from time to time getting into the bloodstream, which can later on cause sepsis! What are Infected Vegetations? -Web of platelets and fibrin that get colonized by microbes. A relatively avascular site that promotes bacterial growth shielded from serum bactericidal factors. -Friable vegetations shower bacteria into the circulation as SEPTIC EMBOLI

COURSE OF EVENTS IN SEPSIS Bacteremia leading to MODS Bacterimia -> SIRS/Sypsis -> Severe sepsis -> Septic shock -> MODS SIRS= Systemic Inflammatory Response syndrome MODS= Multiple Organ Dysfunction Syndrome Gram positive organisms also have LIPOTEICHOIC ACID and TEICHOIC ACID on their cell wall that can also lead to septic shock. These are PAMPS that can cause the septic shock

SIRS: -Early stages in which there is an immune response. -Any two of : *Altered temperature *Tachycardia *Tachypnoea *Altered WBC counts(<4000 cells/mm3 or > 12,000/mm3; *> 10% immature neutrophils/ band forms *Bandemia - when significant called LEFT SHIFT Sepsis: 2 signs and symptoms of SIRS + a documented site of infection. Septic shock: Hypotension and hypo perfusion despite resuscitation MODS: Multiple Organ Dysfunction Syndrome Altered organ functions that cannot be normalized without intervention

Gram Positive Cocci -Note that STREP BOVIS can be ALPHA HEMOLYTIC and some can be GAMMA HEMOLYTIC. Biochemically they look very similar to ENTEROCOCCUS (which is also gamma hemolytic) so you need to differentiate between the two. Strep bovis is found in the colon associated with colonic carcinoma, and enterococcus is usually found in the gut! If you have something going on in the gut, if cancerous, it is usually streptococcus bovis (which is now also called streptococcus gallolyticus)

STAPHYLOCOCCI (clusters) -Staphyle-Bunch -Coccus: Berry -CATALASE POSITIVE STREPTOCOCCI (chains) -Streptos-twisted chain -Coccus: Berry -Catalase negative 1. ALPHA HEMOLYTIC STREPTOCOCCI (partial hemolysis of blood - produce green colonies) -Strep viridans group -Strep bovis -Strep pneumoniae 2. BETA HEMOLYTIC STREPTOCOCCI (complete degradation of blood) -Group A- V Eg. Gp A S.pyogenes 3. GAMMA HEMOLYTIC STREPTOCOCCI (no hemolysis of blood) -Enterococci: *E.fecalis *E.fecium

Gram positive cocci What differentiates staphylococcus from streptococcus is the CATALASE TEST! Catalase acts on H2O2 and breaks it to release oxygen. Staph: Catalase + Strep: Catalase - To further specify the types of staph, you can do the COAGULASE TEST (which converts fibrinogen to fibrin - helping to wall off lesions very well)

STAPHYLOCOCCI (clusters) -Staphyle-Bunch -Coccus: Berry -CATALASE POSITIVE -Staphylococcus aureus: COAGULASE POSITIVE -Staphylococcus epidermidis/saprophyticus: Coagulase negative *Staphylococcus epidermidis: Sensitive to Novobiocin Sensitivity Test *Staphylococcus saprophyticus: Resistant to Novobiocin Sensitivity Test STREPTOCOCCI (chains) -Catalase negative

Simple conidia

Seen in Penicillium -See a string of beads arranged at the ends of the condiophore

Interactions among viruses

Seen when two or more virus particles infect the same host cell -Recombination -Reassortment -Complementation -Phenotypic mixing

Bacterial Classification Cell morphology Morphology=shape

Shapes -Rod *BACILLUS *Do not confuse the adjective bacillus with the bacterial genus for Bacillus *Vibrio - curved rods -Coccobacillus - Small rod -Coccus (pl. cocci) -Spirochete - spiral shape - (treponema pallidum - agent of SYPHILLIS -Pleomorphic - indefinite shape (Mycoplasma)

Factors which influence the resistance of microorganisms to various procedures employed (cont)

Species and strain of the organisms -Intrinsic resistance varies *Spores are resistant to disinfectants (why? the spore coat and cortex act as a barrier) *Mycobacteria have a waxy cell wall that prevents disinfectant entry *Gram-negative bacteria possess an outer membrane that acts as a barrier to the uptake of disinfectants *Resistance to chemical germicides Prions>bacterial spores>coccidian (example, Cryptosporidium)>Mycobacteria>non-enveloped viruses>fungi>vegetative bacteria>enveloped viruses

The alpha hemolytic streptococci

Strep viridans group: -S. mutans -S. salivarius -S. mitis -S. anginosus -Strep bovis-(Strep gallolyticus) ( biochemically resembles Enterococci) -Streptococcus pneumoniae (Pneumococci)

Pathogenesis???

Structural and biochemical mechanisms by which microorganisms cause disease!

LPS, a.k.a endotoxin

Structure: 3 components: 1. LIPID A 2. CORE 3. O SIDE CHAIN 1. Lipid A/endotoxin *Glycolipid that anchors the entire LPS to the outer leaflet of OM bilayer *LPS is released upon death of the cell (and a small amount during division of the cell), functions as endotoxin -Can trigger endotoxic shock/septic shock - caused by the LIPID A portion of the LPS GRAM NEG BACTERIA! 2. Core *Composed of primarily Polysaccharides 3. O polysaccharide *Hydrophilic side chains *Varies from one bacterial species to another *Antigenic (O antigen); induces production of Abs in the host **Used as diagnostic aid in identification, eg. in E. coli O157:H7 (the 157 is a specific identifier of the O polysaccharide character in that strain, different strains will have different identifying numbers) -Mutants that do not make O Ag become sensitive to bile and antibiotics *This just tells us that this portion of the molecule helps the LPS confer a protective feature on the bacteria that have them -LOS (lipo oligosaccharide) seen in Neisseria and Haemophilus ***Don't have to remember genus names *Here the O antigen is not completely missing but it is truncated in these species

What is the mechanism of resistance of intact skin when exposed to viral agents in the environment?

The outer layer of the skin is dead and viruses need living cells to multiply -This is why the mucosae (Conjunctiva, oral cavity, genital mucosa) are susceptible and vulnerable to viral infections because you don't have dead cells over top to protect them -Then how do viruses like chickenpox/herpes infect the skin causing lesions/etc? Because they are actually infecting the ENDOTHELIAL CELLS of the blood vessles of the skin, and not the skin itself

Banana shaped gametocyte of Plasmodium falciparum

They are present EXTRACELLULARLY

CPE, Human Papilloma Virus

ThinPrep pap smear with group of normal cervical cells on left and HPV-infected cells showing features typical of koilocytes: enlarged (x2 or x3) nuclei and hyperchromasia.

Infectious causes of Myocarditis/ Pericarditis

Type of infection: Organism involved -Viral pericarditis: Coxsackie virus, Echovirus -Purulent pericarditis: Staphylococcus aureus -Chronic pericarditis: Mycobacterium tuberculosis -Myocarditis: Coxsackie virus B -Myocarditis, Cardiac arrythmias, Cardiomyopathy: Trypanosoma cruzi

A 46 yo woman presents with fever, nausea, painful and frequent micturition. What should be your next recourse and why?

Urine sample. You want to take a specimen from that region.

Warthin-Starry Silver stain

Used for visualization of spirochetes in tissue -Walls too thin to be reliably stained in Gram reaction Bacterial cell wall binds silver ions -> reduced to elemental silver (which is a heavy silver) *Ie. would be used to find syphillis agents

What is the basis for development of splinter hemorrhages?

Vasculitis! Vegetations can develop on the heart valves. They can then break off from time to time and go into the bloodstream (these are called septic/infective emboli). They can lodge into various part of the body which has vasculature because of vasculitis. A lot of the manifestations, like Janeway lesions and splinter hemorrhages can be explained because of vasculitis. What is the reason for an Oslers node or arthritis? Because of the deposition of immune complexes. A lot of organisms circulating in the body will generate a good mass of antigens to which specific antibodies will attach and those get deposited in various sites in the body. One of the preferred sites is the joints. Similarly, they can get deposited on the vascular endothelia. They also bring their complements there so there could be an immune complex mediated manifestation so patients may get different kind of manifestations like athralgia, arthritis, oslers nodes, rod spots, etc.

Viral Oncogenesis

Viruses immortalize cells -activating or providing growth-stimulating genes, -removing the inherent braking mechanisms that limit DNA synthesis and cell growth -preventing apoptosis -Providing or inducing growth stimulating cytokines DNA tumor viruses: establish persistent infections -Prevent activity of normal growth suppressor proteins such as p53 and RB (HPV, adenovirus) -Epstein-Barr Virus (EBV) acts as mitogen to stimulate B cell proliferation -Inhibit apoptosis; EBV

Colonization and disease

What happens when we encounter a microorganism? -Transient colonization *Ie. If you touch a surface that has microbes, the microbes come from that surface to your hands (which is why you wash your hands) -Permanent colonization -Disease

A patient is brought into the ER after a road traffic accident. After stabilizing him, he is moved to the OR for emergency surgery. The nurse and nursing attendant survey the scene. Suggest and justify the most appropriate method of disinfection or sterilization for the bed linen with blood on it.

What you would do is bag the soiled laundry and send it to the laundry section of the hospital where they wash it in hot water and bleach). -Before we start thinking about disinfecting or sterilizing anything, the first thing we need to do is DECONTAMINATION! You have some organic matter on a bed sheet or bed pan, etc., you have to get rid of the organic matter or the disinfection or sterilization process wont work. Say you go to a dinner, and notice something on the plate from the last persons meal -> This is what happens when you disinfect or sterilize before you decontaminate. The organic matter wont leave, but it will get plastered there from whatever process it went through. This like putting dirty dishes in the dish washer without removing/scrubbing off the organic matter.

What do you mean by major crietria?

When you have a conclusive evidence of the presence of the disease. Dukes criteria takes into consideration, typical clinical, laboratory and echocardiographic findings. Positive echocardiogram for IE defined as : • oscillating intracardiac mass on valve or supporting structures, in the path of regurgitant jets, or on implanted material in the absence of an alternative anatomic explanation, or • abscess, or • new partial dehiscence of prosthetic valve or New valvular regurgitation (worsening or changing of preexisting murmur not sufficient). New regurgitant murmurs suggestive of valvular damage or ruptured chordae

Sporangiospores

produced within a sac called sporangium; e.g. Rhizopus , Mucor -These are the bread molds

Arthroconidia

rectangular spores which arise from fragmentation of hyphae. E.g. Coccidioides immitis -You would have a fungal hyphae and the arthroconidia develop inside the fungal hyphae and the hyphae breaks up to release the arthroconidia in the environment


Kaugnay na mga set ng pag-aaral

Exploring Creation with Chemistry Mod 1 OYOs

View Set

Other Health Insurance Concepts Practice Questions

View Set

Sololearn Python for Beginners #6 Functions

View Set

Torts - Invasion of Right to Privacy

View Set

Genetics Final Exam Homework Review

View Set